Вы находитесь на странице: 1из 89

Taller de resolucin

de problemas de concurso
Universidad de Puerto Rico
Colegio Universitario de Cayey
Dr. David A. SANTOS

Versin del 2 de febrero de 2006

ndice general

Prefacio

1. Tcnicas elementales
1.1. Contradiccin . . . . . . . . . . . . . .
Tarea . . . . . . . . . . . . . . . . . . . . . .
1.2. Principio de las pichoneras de Dirichlet
Tarea . . . . . . . . . . . . . . . . . . . . . .
1.3. Paridad . . . . . . . . . . . . . . . . .
Tarea . . . . . . . . . . . . . . . . . . . . . .
1.4. Induccin . . . . . . . . . . . . . . . .
Tarea . . . . . . . . . . . . . . . . . . . . . .
1.5. Buen orden . . . . . . . . . . . . . . .
Tarea . . . . . . . . . . . . . . . . . . . . . .
1.6. Condiciones extremas . . . . . . . . . .
Tarea . . . . . . . . . . . . . . . . . . . . . .

.
.
.
.
.
.
.
.
.
.
.
.

.
.
.
.
.
.
.
.
.
.
.
.

.
.
.
.
.
.
.
.
.
.
.
.

.
.
.
.
.
.
.
.
.
.
.
.

.
.
.
.
.
.
.
.
.
.
.
.

.
.
.
.
.
.
.
.
.
.
.
.

.
.
.
.
.
.
.
.
.
.
.
.

.
.
.
.
.
.
.
.
.
.
.
.

.
.
.
.
.
.
.
.
.
.
.
.

.
.
.
.
.
.
.
.
.
.
.
.

.
.
.
.
.
.
.
.
.
.
.
.

.
.
.
.
.
.
.
.
.
.
.
.

.
.
.
.
.
.
.
.
.
.
.
.

.
.
.
.
.
.
.
.
.
.
.
.

.
.
.
.
.
.
.
.
.
.
.
.

.
.
.
.
.
.
.
.
.
.
.
.

.
.
.
.
.
.
.
.
.
.
.
.

.
.
.
.
.
.
.
.
.
.
.
.

.
.
.
.
.
.
.
.
.
.
.
.

.
.
.
.
.
.
.
.
.
.
.
.

.
.
.
.
.
.
.
.
.
.
.
.

.
.
.
.
.
.
.
.
.
.
.
.

.
.
.
.
.
.
.
.
.
.
.
.

.
.
.
.
.
.
.
.
.
.
.
.

.
.
.
.
.
.
.
.
.
.
.
.

.
.
.
.
.
.
.
.
.
.
.
.

.
.
.
.
.
.
.
.
.
.
.
.

.
.
.
.
.
.
.
.
.
.
.
.

.
.
.
.
.
.
.
.
.
.
.
.

.
.
.
.
.
.
.
.
.
.
.
.

.
.
.
.
.
.
.
.
.
.
.
.

.
.
.
.
.
.
.
.
.
.
.
.

.
.
.
.
.
.
.
.
.
.
.
.

.
.
.
.
.
.
.
.
.
.
.
.

.
.
.
.
.
.
.
.
.
.
.
.

.
.
.
.
.
.
.
.
.
.
.
.

.
.
.
.
.
.
.
.
.
.
.
.

.
.
.
.
.
.
.
.
.
.
.
.

3
3
4
4
6
6
8
8
9
10
11
11
12

2. lgebra y aritmtica
2.1. Identidades algebraicas
Tarea . . . . . . . . . . . . .
2.2. Los enteros . . . . . .
Tarea . . . . . . . . . . . . .
2.3. Aritmtica modular . .
Tarea . . . . . . . . . . . . .

.
.
.
.
.
.

.
.
.
.
.
.

.
.
.
.
.
.

.
.
.
.
.
.

.
.
.
.
.
.

.
.
.
.
.
.

.
.
.
.
.
.

.
.
.
.
.
.

.
.
.
.
.
.

.
.
.
.
.
.

.
.
.
.
.
.

.
.
.
.
.
.

.
.
.
.
.
.

.
.
.
.
.
.

.
.
.
.
.
.

.
.
.
.
.
.

.
.
.
.
.
.

.
.
.
.
.
.

.
.
.
.
.
.

.
.
.
.
.
.

.
.
.
.
.
.

.
.
.
.
.
.

.
.
.
.
.
.

.
.
.
.
.
.

.
.
.
.
.
.

.
.
.
.
.
.

.
.
.
.
.
.

.
.
.
.
.
.

.
.
.
.
.
.

.
.
.
.
.
.

.
.
.
.
.
.

.
.
.
.
.
.

.
.
.
.
.
.

.
.
.
.
.
.

.
.
.
.
.
.

.
.
.
.
.
.

.
.
.
.
.
.

.
.
.
.
.
.

.
.
.
.
.
.

13
13
17
18
21
22
25

3. Combinatoria
3.1. Las reglas de la multiplicacin y la suma
Tarea . . . . . . . . . . . . . . . . . . . . . .
3.2. Mtodos combinatorios . . . . . . . . .
3.2.1. Permutaciones sin repeticin . .
3.2.2. Permutaciones con repeticin .
3.2.3. Combinaciones sin repeticin .
3.2.4. Combinaciones con repeticin .
3.3. Principio de inclusin-exclusin . . . .
Tarea . . . . . . . . . . . . . . . . . . . . . .

.
.
.
.
.
.
.
.
.

.
.
.
.
.
.
.
.
.

.
.
.
.
.
.
.
.
.

.
.
.
.
.
.
.
.
.

.
.
.
.
.
.
.
.
.

.
.
.
.
.
.
.
.
.

.
.
.
.
.
.
.
.
.

.
.
.
.
.
.
.
.
.

.
.
.
.
.
.
.
.
.

.
.
.
.
.
.
.
.
.

.
.
.
.
.
.
.
.
.

.
.
.
.
.
.
.
.
.

.
.
.
.
.
.
.
.
.

.
.
.
.
.
.
.
.
.

.
.
.
.
.
.
.
.
.

.
.
.
.
.
.
.
.
.

.
.
.
.
.
.
.
.
.

.
.
.
.
.
.
.
.
.

.
.
.
.
.
.
.
.
.

.
.
.
.
.
.
.
.
.

.
.
.
.
.
.
.
.
.

.
.
.
.
.
.
.
.
.

.
.
.
.
.
.
.
.
.

.
.
.
.
.
.
.
.
.

.
.
.
.
.
.
.
.
.

.
.
.
.
.
.
.
.
.

.
.
.
.
.
.
.
.
.

.
.
.
.
.
.
.
.
.

.
.
.
.
.
.
.
.
.

.
.
.
.
.
.
.
.
.

.
.
.
.
.
.
.
.
.

.
.
.
.
.
.
.
.
.

.
.
.
.
.
.
.
.
.

.
.
.
.
.
.
.
.
.

.
.
.
.
.
.
.
.
.

.
.
.
.
.
.
.
.
.

.
.
.
.
.
.
.
.
.

.
.
.
.
.
.
.
.
.

27
27
31
31
32
33
34
37
39
42

4. Sumas y recurrencias
4.1. Progresiones aritmticas . . . . . . . . . . . . . . . . . . . . . . . . . . . . . . . . . . . . . . . . . . . . . .
Tarea . . . . . . . . . . . . . . . . . . . . . . . . . . . . . . . . . . . . . . . . . . . . . . . . . . . . . . . . . . . .
4.2. Progresiones geomtricas . . . . . . . . . . . . . . . . . . . . . . . . . . . . . . . . . . . . . . . . . . . . . .

44
44
46
46

.
.
.
.
.
.

.
.
.
.
.
.

.
.
.
.
.
.

.
.
.
.
.
.

.
.
.
.
.
.

.
.
.
.
.
.

.
.
.
.
.
.

.
.
.
.
.
.

II

NDICE GENERAL

Tarea . . . . . . . . . . . . . . . . . . . . .
4.3. Cancelacin telescpica . . . . . . . .
Tarea . . . . . . . . . . . . . . . . . . . . .
4.4. Recursiones y ecuaciones funcionales
Tarea . . . . . . . . . . . . . . . . . . . . .

.
.
.
.
.

.
.
.
.
.

.
.
.
.
.

.
.
.
.
.

.
.
.
.
.

.
.
.
.
.

.
.
.
.
.

.
.
.
.
.

.
.
.
.
.

.
.
.
.
.

.
.
.
.
.

.
.
.
.
.

.
.
.
.
.

.
.
.
.
.

.
.
.
.
.

.
.
.
.
.

.
.
.
.
.

.
.
.
.
.

.
.
.
.
.

.
.
.
.
.

.
.
.
.
.

.
.
.
.
.

.
.
.
.
.

.
.
.
.
.

.
.
.
.
.

.
.
.
.
.

.
.
.
.
.

.
.
.
.
.

.
.
.
.
.

.
.
.
.
.

.
.
.
.
.

.
.
.
.
.

.
.
.
.
.

.
.
.
.
.

.
.
.
.
.

.
.
.
.
.

.
.
.
.
.

.
.
.
.
.

.
.
.
.
.

49
50
55
56
59

.
.
.
.

.
.
.
.

.
.
.
.

.
.
.
.

.
.
.
.

.
.
.
.

.
.
.
.

.
.
.
.

.
.
.
.

.
.
.
.

.
.
.
.

.
.
.
.

.
.
.
.

.
.
.
.

.
.
.
.

.
.
.
.

.
.
.
.

.
.
.
.

.
.
.
.

.
.
.
.

.
.
.
.

.
.
.
.

.
.
.
.

.
.
.
.

.
.
.
.

.
.
.
.

.
.
.
.

.
.
.
.

.
.
.
.

.
.
.
.

.
.
.
.

.
.
.
.

.
.
.
.

.
.
.
.

.
.
.
.

.
.
.
.

.
.
.
.

.
.
.
.

.
.
.
.

60
60
63
64
70

6. Geometra
6.1. Razones de reas de tringulos . . . . .
Tarea . . . . . . . . . . . . . . . . . . . . . .
6.2. Teoremas de Ceva y de Menelao . . . .
6.3. Puntos y rectas notables de un tringulo
Tarea . . . . . . . . . . . . . . . . . . . . . .

.
.
.
.
.

.
.
.
.
.

.
.
.
.
.

.
.
.
.
.

.
.
.
.
.

.
.
.
.
.

.
.
.
.
.

.
.
.
.
.

.
.
.
.
.

.
.
.
.
.

.
.
.
.
.

.
.
.
.
.

.
.
.
.
.

.
.
.
.
.

.
.
.
.
.

.
.
.
.
.

.
.
.
.
.

.
.
.
.
.

.
.
.
.
.

.
.
.
.
.

.
.
.
.
.

.
.
.
.
.

.
.
.
.
.

.
.
.
.
.

.
.
.
.
.

.
.
.
.
.

.
.
.
.
.

.
.
.
.
.

.
.
.
.
.

.
.
.
.
.

.
.
.
.
.

.
.
.
.
.

.
.
.
.
.

.
.
.
.
.

.
.
.
.
.

.
.
.
.
.

.
.
.
.
.

.
.
.
.
.

71
71
73
73
77
79

A. Indicaciones y respuestas
Indicaciones y respuestas . . . . . . . . . . . . . . . . . . . . . . . . . . . . . . . . . . . . . . . . . . . . . . . . .

80
80

5. Polinomios y ecuaciones
5.1. Ecuaciones . . . .
Tarea . . . . . . . . . . .
5.2. Polinomios . . . .
Tarea . . . . . . . . . . .

.
.
.
.

.
.
.
.

.
.
.
.

.
.
.
.

.
.
.
.

.
.
.
.

.
.
.
.

.
.
.
.

.
.
.
.

.
.
.
.

Prefacio

Escrib estas notas en el verano del 1996, en un taller de resolucin de problemas de concurso para maestros de secundaria
en Puerto Rico.
El hacerlas accesibles en la red me ha hecho percatar cun tiles han sido para estudiantes de habla castellana, ya que
muchos lectores se han comunicado conmigo. Por eso decidido el revisarlas frecuentemente.
Quisiera pues pedir a los lectores el que me comunicasen errores que hallaren, etc.
Bastante del material que aqu aparece es traduccin de material que he escrito en lengua gringa. De semana en semana, de
mes en mes, ir aadiendo material y las respuestas de varios ejercicios. Todava necesito aadir material en geometra, teora
de grafos, enumeracin y anlisis. Invito tambin a los lectores a contribuir material.
David A. SANTOS
dsantos@ccp.edu

Captulo

Tcnicas elementales
1.1. Contradiccin
1 Ejemplo Sean x, y, z, w enteros satisfaciendo

1 1 1 1
+ + + = 1.
x y z w
Demustrese que al menos uno de ellos es par.
Resolucin: Presmase que todos x, y, z, w son nones. Luego
yzw + xzw + xyw + xyz = xyzw.
El lado siniestro es par, por ser la suma de cuatro enteros nones. El lado diestro es non, siendo el producto de cuatro enteros
nones. Esto resulta en una contradiccin.
2 Ejemplo El producto de 34 enteros es igual a 1. Demustrese que la suma de stos no puede ser 0.

Resolucin: Forzozamente los enteros debern ser 1. Ya que el producto es 1, debe de haber un nmero par de 1s. Si la
suma de estos enteros fuese 0, entonces debera haber tantos +1s como 1s. Luego as habrn de haber diecisiete 1s y
diecisiete +1s, lo que conlleva a una contradiccin.
3 Ejemplo Demustrese, sin recurrir a una calculatriz, que 6

1
35 < .
10

1
1
Resolucin: Presmase que 6 35 . Entonces 6
35, o sea, 59 10 35. Al cuadrar uno y otro lado, 3481
10
10

1
3500, lo que no tiene sentido. Luego entonces se concluye que 6 35 < .
10
4 Ejemplo Sea a1 , a2 , . . . , an una permutacin arbitraria de los enteros 1, 2, . . . , n, donde n es non. Demustrese que el producto

(a1 1)(a2 2) (an n)


es par.
Resolucin: Obsrvese primero que la suma de un nmero impar de enteros impares impar es. Slo tiene que demostrarse
que al menos una de las diferencias ak k es par. Presmase al contrario, que todas las diferencias ak k impares son. Es
evidente que
S = (a1 1) + (a2 2) + + (an n) = 0,
3

Captulo 1

ya que las ak s son un reordenamiento de 1, 2, . . . , n. S es, por suposicin, la suma de un nmero impar de enteros impares,
resultando en el entero par 0. Esto es imposible, as que nuestra suposicin inicial es falsa y por lo tanto al menos una de las
diferencias ak k es par, lo que por consiguiente, hace par al producto.
5 Ejemplo Demustrese que

2 es irracional.

a
2 = , con enteros positivos a, b. Esto conlleva a 2b2 = a2 . Ahora bien, tanto a2 como b2
b
tienen un nmero par de primos en su factorizacin (contando repeticiones). Luego 2b2 tiene un nmero impar de primos en
su factorizacin y a2 tiene un nmero par de primos en su factorizacin. Esto contradice el hecho de que todo entero positivo
mayor que 1 puede descomponerse en factores primos de forma nica.
Resolucin:

Presmase que

6 Ejemplo Demestrese que 2003 no es la suma de dos cuadrados.

Resolucin: Primero se demostrar que la suma de dos cuadrados nunca deja residuo 3 al ser dividida por 4. De esto se
obtiene el resultado de inmediato. Cada entero es o bien par, (de la forma 2k) o non (de la forma 2k + 1). Se tiene que
(2k)2
(2k + 1)2

=
=

4(k2 ),
4(k2 + k) + 1.

Luego, el cuadrado de cada entero o bien deja residuo 0 o bien deja residuo 1 al ser dividido por 4. La suma de dos enteros
entonces dejar residuo 0, 1, o 2 al ser dividida por 4.
7 Ejemplo Si a, b, c son enteros impares, demustrese que la ecuacin ax2 + bx + c = 0 no posee una solucin racional.

Resolucin: Si la ecuacin poseyere la solucin racional


2

p
q

p
, con p, q relativamente primos, entonces
q

+b

p
+ c = 0 = ap2 + bpq + cq2 = 0.
q

Si ambos p y p fuesen nones, entonces ap2 + bpq + cq2 sera tambin non, y por lo tanto 6= 0. De manera semejante, si uno entre
p y q fuese impar y el otro par, luego o bien ap2 + bpq o bien bpq + cq2 sera par, y ap2 + bpq + cq2 impar, otra contradiccin.
Luego, tal raz racional d f racpq es ficticia.

Tarea
8 Problema En ABC, A > B. Demustrese que BC > AC.

9 Problema Sea 0 < < 1. Demustrese que

> .

10 Problema Sea =
0,999 . . . en donde hay al menos 2000 nueves. Demustrese que

la expansin decimal de

tambin comienza con al menos 2000 nueves.

11 Problema Demostrar que no existen enteros a, b, c, d tales que

x4 + 2x2 + 2x + 2 = (x2 + ax + b)(x2 + cx + d).

1.2. Principio de las pichoneras de Dirichlet


12 Ejemplo Las nueve casillas de un cuadrado 3 3 son llenadas aleatoriamente por 1s, 0s, o 1s. Demustrese que entre

las ocho sumas resultantes (tres columnas, tres filas y dos diagonales), hay al menos dos de ellas idnticas.

Resolucin: Hay siete sumas posibles, cada una un entero del conjunto {3, 2, 1, 0, 1, 2, 3}. Por el principio de las pichoneras, dos de las ocho sumas del cuadrado debern de coincidir.

Principio de las pichoneras de Dirichlet

13 Ejemplo Cincuenta y un puntos se distribuyen sobre un cuadrado 1 1. Demustrese que hay al menos tres puntos que

pueden ser cubiertos por un cuadrado

1 1
.
5 5

1 1
Resolucin: Divdase al cuadrado en veinticinco subcuadrados , cada uno de ellos con sus lados paralelos al cuadrado
5 5
51
original. Uno de estos subcuadrados posee V W = 3 al menos.
25
14 Ejemplo (Putnam 1978) Sea A cualquier conjunto de veinte enteros escogidos de la progresin aritmtica 1, 4, . . . , 100.

Demustrese que hay al menos dos enteros diferentes en A cuya suma es 104.
Resolucin: Agrpese los treinticuatro enteros de esta progresin en los dicienueve grupos
{1}, {52}, {4, 100}, {7, 97}, {10, 94}, . . . , {49, 55}.
Como se habr de escoger veinte y se tiene diecienueve conjuntos, se habr de tomar dos enteros, al menos, perteneciendo al
mismo conjunto, y estos sumarn a 104.
15 Ejemplo Demustrese que entre siete enteros positivos distintos 126, siempre se puede conseguir dos de ellos a y b

satisfaciendo

b < a 2b.
Resolucin: Divdase el conjunto {1, 2, 3, . . . , 126} en los seis subconjuntos
{1, 2}, {3, 4, 5, 6}, {7, 8, . . . , 13, 14}, {15, 16, . . . , 29, 30},
{31, 32, . . . , 61, 62} y {63, 64, . . . , 126}.
Dos de los siete enteros yacern en el mismo subconjunto y satisfacern las desigualdades mencionadas.
16 Ejemplo No importa cuales cincuenta y cinco enteros se seleccionen de

{1, 2, . . . , 100},
demustrese que habr dos de ellos cuya diferencia ser 10.
Resolucin: Obsrvese primero que si elegimos n + 1 enteros de cualquiera ristra de 2n enteros consecutivos, entonces habr
dos cuya diferencia ser n. Esto es patente al aparear los 2n enteros consecutivos
{a + 1, a + 2, a + 3, . . . , a + 2n}
en los n pares
{a + 1, a + n + 1}, {a + 2, a + n + 2}, . . . , {a + n, a + 2n}.
Agrpese pues los cien enteros como sigue:
{1, 2, . . . 20}, {21, 22, . . . , 40},
{41, 42, . . . , 60}, {61, 62, . . . , 80}
y
{81, 82, . . . , 100}.
Si seleccionamos cincuenta y cinco enteros, entonces, forzozamente habremos de seleccionar once del mismo grupo. Del dicho
grupo, por la observacin anterior (con n = 10), habr dos cuya differencia ser 10.

Captulo 1

17 Ejemplo (AHSME 1994) Mrquese a un disco con la etiqueta 1, a dos discos con la etiqueta 2, a tres discos con la

etiqueta 3, . . . , a cincuenta discos con la etiqueta 50. Pngase a estos 1 + 2 + 3 + + 50 = 1275 discos en una caja. Se
sacan luego discos de la caja, al azar y sin remplazo. Cul es el nmero mnimo de discos que se debe sacar para garantizar al
menos diez discos con la misma etiqueta?
Resolucin: Si se saca todos los 1+2+ +9 = 45 discos con etiquetas 1, . . . , 9 y cualquiera nueve discos con etiquetas
10, . . . , 50, se habr sacado 45 + 9 41 = 414 discos. El 415-avo disco sacado garantizar que haya al menos diez discos
con la misma etiqueta.
18 Ejemplo Dado cualquier subconjunto A de diez enteros del conjunto {1, 2, . . . , 98, 99} demustrese que siempre habr dos
subconjuntos disjuntos de A cuyos elementos tienen la misma suma.

Resolucin: Hay 210 1 = 1023 subconjuntos no nulos que se pueden formar con un conjunto de diez elementos. A cada uno
de estos subconjuntos le asociamos su suma. La mxima suma que puede ser obtenible es 90 + 91 + + 99 = 945 < 1023.
Luego, hay dos subconjuntos, digamos S, T (no necesariamente disjuntos) cuya suma de elementos es idntica. Luego, S\(ST )
y T \ (S T ) tambin tienen suma idntica de elementos.
19 Ejemplo Dados cualesquiera 9 enteros cuyos factores primos yagan en el conjunto {3, 7, 11}, demustrese que habr dos

cuyo producto es un cuadrado perfecto.

Resolucin: Para que un entero sea un cuadrado, todos los exponentes de los primos de su factorizacin en primos deben ser
pares. Todo entero cuyos factores yagan en el conjunto dado es de la forma 3a 7b 11c . Los tros (a, b, c) yacen en exactamente
uno de los 8 patrones de paridad (par, par, par), (par, par, non), (par, non, par), (par, non, non), (non, par, par), (non, par, non),
(non, non, par), (non, non, non). En un grupo de nueve tales enteros, habr pues dos cuyo patrn de paridad sea idntico.
Luego el producto de estos dos enteros es un cuadrado, ya que la suma de cada exponente ser par.

Tarea
20 Problema Si se toman n + 1 enteros del conjunto {1, 2, . . . , 2n}, demustrese que

siempre habr dos que son relativamente primos.

21 Problema Si se toman n + 1 enteros del conjunto {1, 2, . . . , 2n}, demustrese que

siempre habr dos tales que el menor dividir (sin dejar residuo) al mayor.

23 Problema (AHSME 1991) Una mesa circular tiene exactamente sesenta sillas en
torno. Hay N personas ya sentadas de manera que la prxima persona a sentarse por
fuerza se sentar al lado de alguien. Cul es el valor mnimo de N?

24 Problema Cinco puntos cualesquiera son colocados sobre un


cuadrado de lado 1.
Demustrese que dos de ellos estn a una distancia de a lo sumo 2/2.

22 Problema Pruebe que entre n + 1 enteros, siempre habr dos cuya diferencia ser

divisible por n.

1.3. Paridad
25 Ejemplo Dos esquinas diametralmente opuestas son cortadas de un tablero de ajedrez, que como se recordar, tiene 64

casillas. Demustrese que es imposible recubrir totalmente a las 62 casillas restantes de 31 domins.
Resolucin: Cada domin cubre cuadrados de diferente color. Al eliminar dos casillas diametralmente opuestas, se eliminan
dos casillas del mismo color. Por lo tanto quedan 32 casillas de un color y 30 de otras y luego los 31 domins no las pueden
cubrir a todas.
26 Ejemplo Los 28 domins de un juego se enfilan observando las reglas del domin. Si al principio de la cadena se observa
un 6 qu entero se observar al final de la cadena?

Resolucin:

Se observar tambin a un 6. Cada nmero debe ocurrir un nmero par de veces de manera que se puedan

Paridad

enfilar. De los ocho 6s: se tiene uno al principio de la cadena, seis de ellos se aparearn entre s en medio de la cadena y
finalmente, el restante quedar al final de la cadena.
27 Ejemplo Demustrese que para ninguna seleccin de signos en

1 2 10,
se obtendr una suma 0.
Resolucin: La suma 1 + 2 + + 10 = 55, un entero impar. Ya que la paridad no es afectada por la eleccin de signo, para
cualquier seleccin de signo 1 2 10 nunca ser par, y en particular, nunca ser 0.
28 Definicin Llmase punto reticular en el plano al punto coordenado (m, n) en el plano cuyas coordenadas m y n son ambas

enteras.
29 Definicin El punto medio del segmento de recta que comienza en (x, y) y termina en (x1 , y1 ) es el punto
x+x

y + y1 
.
2

30 Ejemplo Se seleccionan cinco puntos reticulares en el plano, al azar. Demostrar que existe dos de entre ellos que forman

un segmento de recta cuyo punto medio es tambin un punto reticular.


Resolucin: Hay cuatro patrones de paridad posibles para cada punto reticular en el plano: (par, par), (par, non), (non, non),
(non, par). Por el principio de las pichoneras, dos de los cinco puntos reticulares compartirn el mismo patrn de paridad, y
luego, su punto medio ser tambin reticular.
Para los ejemplos siguientes necesitaremos algunas definiciones de tetromins, las cuales se daremos en las figuras al calce.

Figura 1.1: L-tetromin

Figura 1.4: Tetromin torcido

Figura 1.2: T-tetromin

Figura 1.3: Tetromin recto

Figura 1.5: Tetromin cuadrado

31 Ejemplo Se posee tan slo una copia de los cinco tetromins arriba mostrados, pudiendo as cubrir 20 cuadrados. Demustrese que es imposible arreglarlos de tal manera que se cubra a un rectngulo.

Resolucin: Si tal rectngulo existiese, tendra 20 cuadrados. Colorese el rectngulo a la manera de un tablero de ajedrez,
con diez cuadrados rojos y diez negros. El T-tetromin siempre cubre un nmero impar de cuadrados rojos y los otros siempre
cubren un nmero par de cuadrados rojos. As pues el nmero de cuadrados rojos cubiertos es impar, contradiccin.

Captulo 1

32 Ejemplo Demustrese que un tablero de ajedrez (8 8) no se puede recubrir totalmente con 15 tetromins rectos y un

L-tetromin.

Resolucin: Colorese las filas 1, 3, 5, 7 en blanco y las filas 2, 4, 6, 8 en azul. Un tetromin recto siempre cubrir un nmero
par de cuadrados blancos y un L-tetromin siempre cubrir un nmero impar de cuadrados blancos. Si el recubrimiento fuese
posible entonces se cubrir tan slo un nmero impar de cuadrados blancos, contradiccin.

Tarea
33 Problema Veinticinco nios y veinticinco nias son sentados alrededor de una mesa
circular. Demustrese ambos vecinos de al menos una persona sern nias.

36 Problema Demustrese que un tablero 10 10 nunca se podr recubrir totalmente


con 25 tetromins rectos.

34 Problema Se sueldan 2001 varillas (rectas) formando un camino. Demostrar que no


existe ninguna lnea rectano pasando por un punto de soldadura del caminoque intersecte a todos los 2001 segmentos del camino.

37 Problema Demustrese que un tablero 8 8 nunca se podr recubrir totalmente con


15 T-tetromins y un tetromin cuadrado.

35 Problema Se escribe los 1, 2, . . . , 2001 enteros en una pizarra. Se borran de dos en

dos, remplazndolos con el valor absoluto de su diferencia. Demostrar que el ltimo nmero obtenido nunca ser 0.

38 Problema Una urna tiene 900 boletas, numeradas del 100 al 999. Se sacan boletas al
azar y sin remplazo, y se suman sus dgitos. Cul es el nmero menor de boletas que se
necesitar sacar para garantizar que al menos tres de estas boletas tengan la misma suma
de dgitos?

1.4. Induccin
El principio de induccin matemtica resta en la siguiente observacin intuitiva. Supongamos que tenemos que efectuar
una tarea que requiere cierto nmero de pasos sucesivos. Supongamos que siempre lograremos completar el paso n si ya hemos
completado el paso n 1. As pues, si acaso pudisemos comenzar (completando un paso base), entonces podramos completar
todos los pasos a partir del paso base.
As pues, en el principio de induccin matemtica, tratamos de comprobar la veracidad de una asercin P(n) estableciendo
primero su validez en un caso base k0 (usualmente k0 = 1). Luego tratamos de establecer si informacin sobre la validez de
P(n 1) conlleva a informacin favorable sobre P(n).
39 Teorema (Principio de induccin matemtica) Si un conjunto S de enteros positivos posee al 1, y tambin se verifica

que el entero n + 1 est toda vez que el entero n est, entonces S = N.


40 Corolario Si el conjunto A de enteros positivos contiene al entero m y tambin contiene al entero n + 1 siempre que

contenga a n, donde n > m, entonces A es el conjunto de todos los enteros positivos mayores o iguales a m.
41 Corolario (Induccin robusta) Si el conjunto A de enteros positivos contiene al entero m y tambin contiene a n + 1

siempre que contenga a m + 1, m + 2, . . . , n, donde n > m, entonces A es el conjunto de todos los enteros positivos mayores o
iguales a m.
42 Ejemplo Demostrar que 2n > n, n N.

Resolucin: La asercin es cierta para n = 0, ya que 20 > 0. Presmase que 2n1 > n 1 para n > 1. Ahora bien,
2n = 2(2n1 ) > 2(n 1) = 2n 2 = n + n 2.
Pero n 1 > 0 = n 2 0, ya que n + n 2 n + 0 = n y entonces,
2n > n.
Esto establece el resultado por induccin.

Tarea

43 Ejemplo Demostrar que todo cuadrado se puede descomponer en n subcuadrados, no necesariamente del mismo tamao,

para toda n = 4, 6, 7, 8, . . ..
Resolucin: Al dividir al cuadrado en cuatro, como en la figura 1.6, se incrementa el nmero de cuadrados por tres. As pues,
si n es asequible, tambin lo es n + 3. As pues, si se demuestra que n = 6, n = 7 y n = 8 son factibles, entonces se conseguir
toda descomposicin en n 6 cuadrados. Pero esto se deduce de las figuras 1.7 y 1.8 (para n = 7, se descompone uno de los
subcuadrados de la figura 1.6), terminando la demostracin.

Figura 1.6: Ejemplo 43.

Figura 1.7: Ejemplo 43.

Figura 1.8: Ejemplo 43.

44 Ejemplo Demostrar que

33n+3 26n 27
es un mltiplo de 169 para todo nmero natural n.
Resolucin: Sea P(n) la asercin T N with 33n+3 26n 27 = 169T . Demostrarase que P(1) es cierta y que P(n
1) = P(n). Para n = 1 se asevera que 36 53 = 676 = 169 4 es divisible por 169, lo cual es evidente.
Ahora bien, P(n 1) se traduce en la existencia de un N N tal que 33(n1)+3 26(n 1) 27 = 169N, i.e., para n > 1,
33n 26n 1 = 169N
para algn entero N. Luego
33n+3 26n 27 = 27 33n 26n 27 = 27(33n 26n 1) + 676n
lo que simplifica a
27 169N + 169 4n,
que claramente mltiplo de 169 es. Esto establece el resultado mediante induccin.

Tarea
45 Problema Demostrar que para todo entero n 1, la cantidad

47 Problema Si se toman n + 1 enteros del conjunto {1, 2, . . . , 2n}, demustrese que


siempre habr dos tales que el menor dividir (sin dejar residuo) al mayor.

es un entero par y que

48 Problema La sucesin de Fibonacci est dada por

(1 + 2)2n + (1 2)2n

(1 + 2)2n (1 2)2n = b 2

para algn entero b.

esto es, cada nmero luego del segundo es la suma de los dos precedentes. As la sucesin
de Fibonacci comienza por

46 Problema Si k es impar, demostrar que 2n+2 divide a

k
para todo natural n.

f0 = 0, f1 = 1, fn+1 = fn + fn1 , n 1,

2n

0, 1, 1, 2, 3, 5, 8, 13, 21, . . . .
Demustrese mediante induccin matemtica , que para entero n 1, se tiene
fn1 fn+1 = fn2 + (1)n .

10

Captulo 1

49 Problema En el pas Pesimista, las monedas slo vienen en cantidades de 3 y 5 pe-

sos. Demostrar que toda cantidad de 8 o ms pesos se puede pagar con estas monedas.

50 Problema Utilcese induccin para demostrar que para todo nmero natural n > 0

1
4

1
9


1

1
(n + 1)2


=

n+2
.
2(n + 1)

53 Problema Sea n 1 un entero y sea C un conjunto constituido de 2n + 1 enteros


positivos no nulos, no necesariamente distintos. Supngase que C tiene la propiedad siguiente: Si x C , entonces existe particicin de C \ {x} en dos subconjuntos A y B de
n elementos cada uno tal que la suma de los elementos en A es igual a la suma de los
elementos en B. Demostrar que todos los elementos de C son idnticos.

54 Problema Sean a1 , a2 , . . . , an y b1 , b2 , . . . , bn enteros naturales no nulos. Supngase

que
a1 + a2 + + an = b1 + b2 + + bn < mn.

51 Problema Utilcese induccin para demostrar que para entero natural n, la candidad

n3 + (n + 1)3 + (n + 2)3 es siempre divisible por 9. Puede avalarse de la identidad


(a + b)3 = a3 + 3a2 b + 3ab2 + b3 .

55 Problema Mediante induccin, demuestre que todo tringulo equiltero puede ser
descompuesto en n subtringulos equilteros (no necesariamente del mismo tamao) para toda n 6.

52 Problema Demostrar que para todo entero n 1,

1+

Demostrar que siempre es posible suprimir unos cuantos trminos de uno y otro lado
(pero no todos!) y conservar la igualdad de la suma de los trminos restantes.

1
1
1
++ 2 2 .
n
22
n

56 Problema Sea s un entero estrictamente positivo. Demostrar que en el intervalo cerrado [s; 2s] hay una potencia de 2.

1.5. Buen orden


57 Axioma (Axioma del buen orden) Todo conjunto no vacuo S de nmeros naturales posee un elemento mnimo.
58 Ejemplo Demustrese que no existe ningn entero natural en el intervalo ]0; 1[.

Resolucin: Presmase al contrario que el conjunto S de enteros naturales en ]0; 1[ no es nulo. Por el axioma del buen
orden, este conjunto debe poseer un elemento mnimo, llammosle m. Ahora bien, 0 < m2 < m < 1, y por tanto m2 S . Pero
esto quiere decir que S tiene un entero positivo m2 estrictamente menor que su mnimo elemento m, contradiccin, y por lo
tanto S = .
59 Ejemplo Si a, b, c son enteros tales que a6 + 2b6 = 4c6 , demustrese que a = b = c = 0.

Resolucin: Es evidente que podemos restrinjirnos al caso donde todas las incgnitas son mayores o iguales a 0. Escjase
un tro a, b, c satisfaciendo la ecuacin y con
max(a, b, c) > 0
tan pequeo como fuere posible. Si a6 + 2b6 = 4c6 entonces a deber ser par, a = 2a1 . Esto conlleva a 32a61 + b6 = 2c6 .
Luego tambin b = 2b1 y as 16a61 + 32b61 = c6 . Finalmente esto d c = 2c1 , por lo cual a61 + 2b61 = 4c61 . Pero max(a1 , b1 , c1 ) <
max(a, b, c) lo que contradice la minimalidad de max(a, b, c). As pues se debe tener que max(a, b, c) = 0 y todas las incgnitas
son 0.
60 Ejemplo (IMO 1988) Si a, b son enteros positivos tales que

a2 + b2
a2 + b2
es entero, entonces demuestre que
cuadrado es.
1 + ab
1 + ab

a2 + b2
= k es un contra-ejemplo de un entero que no es cuadrado, con max(a, b) tan pequeo
1 + ab
como fuere posible. Sin prdida de generalidad presmase que a < b, ya que si a = b entonces then

Resolucin: Supngase que

0<k=

2a2
< 2,
a2 + 1

forzando k = 1, un cuadrado.
Ahora bien, a2 + b2 k(ab + 1) = 0 es una ecuacin cuadrtica en b, con suma de races ka y producto de races a2 k.
Sean b1 , b sus races, de modo que b1 + b = ka y b1 b = a2 k.

Tarea

11

Si a, k son enteros positivos, el suponer que b1 < 0 es incompatible con a2 + b21 = k(ab1 + 1). Como k no es cuadrado, el
suponer b1 = 0 es incompatible con a2 + 02 = k(0 a + 1). Adems
b1 =

a2 k b2 k
<
< b.
b
b

Entonces hemos encontrado otro entero b1 para el cual


pues k cuadrado es.

a2 + b21
= k y que es menor que max(a, b), contradiccin. Entonces
1 + ab1

Tarea
62 Problema Demostrar que cuartetos de enteros estrictamente positivos (x, y, z, w) satisfaciendo la ecuacin x2 + y2 = 3(z2 + w2 ) no existen.

61 Problema Demostrar que la serie infinita de cuadrados

1, 4, 9, 16, . . . ,
no contiene ninguna progresin aritmtica infinita.

1.6. Condiciones extremas


63 Ejemplo (Problema de Sylvester) Un conjunto de n puntos en el plano posee la propiedad que toda lnea pasando por dos

de ellos siempre pasa por un tercero de ellos. Demustrese que los puntos estn alineados.
Resolucin: Si los puntos no estuviesen alienados, entre todos los pares (p, L) de puntos p no sobre la lnea L habr uno
minimizando la distancia d entre p y L. Sea f el pie de la perpendicular de p a L, como en la figura 1.9. Por hiptesis hay al
menos tres puntos a, b, c sobre L. Dos de stos, digamos a y b, estn del mismo lado que f y uno de ellos, digamos b, es ms
cercano a f . La distancia de b a la recta ap es menor que d, contradiccin.

A
b

c f

b
b

b
b

Figura 1.9: Ejemplo 63


Figura 1.10: Ejemplo 64.

64 Ejemplo De 2n puntos en el plano, n son rojos, n son azules y ningn tro de entre ellos es colineal. Sortanse los puntos
en n pares de tal manera que cada par tiene un punto rojo y otro azul y se forman n segmentos uniendo cada par de puntos.
Existir un apareamiento para el cual ningn par de segmentos se intersecte?

Resolucin: S existe. El nmero de apareamientos es finito, luego ha de existir una manera de parear los puntos de modo que
la distancia total de los segmentos sea mnima. Sostenemos que bajo estas condiciones ningn par de segmentos se intersecar.
Supngase que bajo las condiciones de mnima distancia arriba estipuladas existe un par de segmentos AB y CD que se
intersecan en el punto P, como en la figura 1.10. En virtud de la desigualdad del tringulo
AD + BC < AP + PD + BP + PC = AB +CD,

12

Captulo 1

minimizando la ya minimizada distancia, contradiccin.

Tarea
65 Problema (BMO 1975) Siete puntos se encuentran sobre un disco cerrado de radio

1. Si las distancias mutuas entre todos estos puntos son 1, demostrar que el centro del

disco pertenece a esta coleccin de puntos.

Captulo

lgebra y aritmtica
2.1. Identidades algebraicas
Una de las identidades ms tiles en la resolucin de problemas es la diferencia de cuadrados
x2 y2 = (x y)(x + y).
Muchas expresiones se pueden factorizar si se convierten en diferencias de cuadrados. Por ejemplo
x4 + x2 y2 + y4

Tambin

a4 + 4b4

= x4 + 2x2 y2 + y4 x2 y2
= (x2 + y2 )2 (xy)2
= (x2 xy + y2 )(x2 + xy + y2 ).

= a4 + 4a2 b2 + 4b4 4a2 b2


= (a2 + 2b2 )2 (2ab)2
= (a2 2ab + 2b2 )(a2 + 2ab + 2b2 )

Otra identidad til es la de diferencia de cubos


x3 y3 = (x y)(x2 xy + y2 ).
Si n es un entero positivo tenemos en general el siguiente teorema.
66 Teorema Sea n un entero positivo. Entonces

xn yn = (x y)(xn1 + xn2 y + xn3 y2 + + xyn2 + yn1 ).


Demostracin: Primero demostraremos que si a 6= 1, entonces
1 + a + a2 + an1 =

1 an
.
1a

Pngase
S = 1 + a + a2 + + an1 .
Entonces
aS = a + a2 + + an1 + an .
Luego
S aS = (1 + a + a2 + + an1 ) (a + a2 + + an1 + an ) = 1 an ,
13

14

Captulo 2
y al ver que
(1 a)S = S aS = 1 an ,
x
se obtiene el resultado. Poniendo ahora a = , se ve que
y
2

x
x
1+ +
y
y
de donde se obtiene

1
lo que equivale a

x
y

++

x
x
1+ +
y
y

x
y

n1
x

++

n1 
x

x x2
xn1
1 + + 2 + + n1
y y
y

x
y
1 xy

= 1

= 1

n
x

xn
.
yn

Multiplicando por yn uno y otro lado

y 1

x n1
x x2
xn1
y
1 + + 2 + + n1
y
y y
y

= yn 1

xn
,
yn

lo que da
yn xn = (y x)(yn1 + yn2 x + + yxn2 + xn1 ),
el resultado pedido.

El segundo factor tiene n trminos y cada trmino tiene grado (peso)

n 1.

67 Corolario Sean x, y enteros con x 6= y y sea n un entero positivo. Entonces x y divide a xn yn .

Por ejemplo, sin necesidad de hacer clculos, el corolario anterior nos dice que 781 = 1996 1215 divide a 19965 12155 .
Otros resultados tiles son los siguientes
68 Teorema Si n es un entero positivo impar

xn + yn = (x + y)(xn1 xn2 y + xn3 y2 xn4 y3 + + x2 yn3 xyn2 + yn1 ).


69 Corolario Sean x, y enteros con x 6= y y sea n un entero positivo impar. Entonces x + y divide a xn + yn .

Por ejemplo 129 = 27 + 1 divide a 2861 + 1 y 1001 = 1000 + 1 = 999 + 2 = = 500 + 501 divide a
11997 + 21997 + + 10001997 .
70 Ejemplo Si a2 + b2 = 1 y ab = 2, halle (a + b)2 , (a b)2 , a4 + b4 .

Resolucin: Tenemos
(a + b)2 = a2 + b2 + 2ab = 5,
(a b)2 = a2 + b2 2ab = 3,
y
a4 + b4 = (a2 + b2 )2 2a2 b2 = 7.
71 Ejemplo Hallar todos los primos de la forma n3 1, donde n es un entero positivo.

Identidades algebraicas

15

Resolucin: Como n3 1 = (n 1)(n2 + n + 1) y como n2 + n + 1 > 1, deberemos tener n 1 = 1. Luego el nico primo de
la forma deseada es 23 1 = 7.
72 Ejemplo Demostrar que el nico primo de la forma n4 + 4 es el 5.

Resolucin: Podemos restrinjirnos a enteros positivos. Vemos que


n4 + 4 = n4 + 4n2 + 4 4n2 = (n2 + 2)2 (2n)2 = (n2 2n + 2)(n2 + 2n + 2).
Si este producto es un nmero primo entonces el factor ms pequeo debe ser igual a 1. As n2 2n + 2 = 1, o sea (n 1)2 = 0,
esto es n = 1. As, el nico primo de esta forma es 14 + 4 = 5.
73 Ejemplo Dado que 1979 es primo, demostrar que si

1 1
1
u
= 1+ + ++
.
b
2 3
1978
entonces 1979 divide a u.
Resolucin: Rearreglemos la suma de la siguiente manera

1
1
1
+
+
1978
2 1977

1
1
1
1
+
++
+
+
3 1976
989 990

1+

1979
1979
1979
+
++
.
1 1978 2 1977
989 990

Al sumar todas las fracciones arriba en la derecha , vemos que el denominador divide a 1978!. Como 1979 es primo, ningn
factor de 1978! cancela al 1979 del numerador. Luego, 1979 divide al numerador de la fraccin.
74 Ejemplo Demostrar la siguiente identidad de Cataln:

1
1
1
1
1
1 1 1
+ ++

=
+
++ .
2 3 4
2n 1 2n n + 1 n + 2
2n

Resolucin: La cantidad de la izquierda es

1
1
1 1 1
+ + ++
+
2 3 4
2n 1 2n
1
1 1 1
+ + ++
2
2 4 6
2n

1+

1
1
1 1 1
+ + ++
+
2 3 4
2n 1 2n
1
1
1 1 1
2
1+ + + ++
2
2 3 4
n

1 1 1
1
1
1+ + + ++
+
2 3 4
2n 1 2n
1
1 1 1
1+ + + ++
2 3 4
n
1
1
1
+
++ ,
n+1 n+2
2n
1+

como queramos demostrar.


75 Ejemplo Si tan x + cot x = a, exprese tan3 x + cot3 x como un polinomio en a.

16

Captulo 2

Resolucin: Primero observemos que


a2 = (tan x + cot x)2 = tan2 x + cot2 x + 2,
de donde a2 2 = tan2 x + cot2 x. As
tan3 x + cot3 x = (tan x + cot x)(tan2 x tan x cot x + cot2 x) = a(a2 3).
76 Ejemplo Factorizar

1 + x + x2 + + x80 .
Resolucin: Pongamos S = 1 + x + x2 + + x80 . Entonces xS = x + x2 + x3 + + x80 + x81 = S 1 + x81 . De aqu
1 + x + x2 + + x80 =

x81 1
.
x1

Luego
x81 1 x81 1 x27 1 x9 1 x3 1
= 27

.
x1
x 1 x9 1 x3 1 x 1
Por lo tanto
1 + x + x2 + + x80 = (x54 + x27 + 1)(x18 + x9 + 1)(x6 + x3 + 1)(x2 + x + 1).
77 Ejemplo Hallar la raz cuadrada de

Resolucin: Observe que


Luego

5 + 2 6.

5 + 2 6 = 3 + 2 2 3 + 2 = ( 2 + 3)2 .

5 + 2 6 = 2 + 3.

78 Ejemplo Simplificar

1
1
1
1
+

++

+
.
99 + 100
1+ 2
2+ 3
3+ 4

Resolucin: Como 1 = n + 1 n = ( n + 1 n)( n + 1 + n), entonces

= n + 1 n.

n+ n+1
Por lo tanto

y as

1+ 2
1

2+ 3
1

3+ 4
..
.
1

99 + 100

=
=
=
..
.
=

2 1

3 2

4 3
..
.

100 99,

1
1
1
1

+
++

+
= 100 1 = 9.
99 + 100
1+ 2
2+ 3
3+ 4

Tarea

17

79 Ejemplo Demostrar que para todo entero positivo n, la expresin

2903n 803n 464n + 261n


es siempre divisible por 1897.
Resolucin: Por el Teorema 66, 2903n 803n es divisible por 2903 803 = 2100 = 7 300 y 261n 464n es divisible por
203 = (29)7. Por lo tanto, la expresin es divisible por 7. Adems 2903n 464n es divisible por 2903464 = 2439 = 9271
y 803n + 261n es divisible por 803 + 261 = 542 = 2 271. As pues, como la expresin es divisible por 7 y por 271 y
como estos son relativamente primos, la expresin es pues divisible por 7 271 = 1897.

Tarea
80 Problema Dado que 9877892 = 975727108521, halle el valor de 9877902 .
81 Problema Calcule (123456789)2 (123456791)(123456787) mentalmente.

82 Problema Halle a6 + a6 dado que a2 + a2 = 4.

83 Problema Demostrar que el entero

95 Problema Evale la suma

1+

1
1
1

.
+
3
3
3
2+ 3 4
4+ 3 6+ 3 9
9 + 3 12 + 3 16

96 Problema Factorice 1 + x + x2 + x3 + + x624 .


97 Problema Expandir el producto

. . . 11
|11{z
}
221 1 s

(1 + x)(1 + x2 )(1 + x4 )(1 + x8 ) (1 + x1024 ).

es compuesto.
98 Problema Demostrar que si 2n 1 es un nmero primo, entonces n es un nmero
primo. Primos de esta forma se llaman primos de Mersenne.

84 Problema Demostrar que 7 divide a

22225555 + 55552222 .
85 Problema Demostrar que 100 divide a 1110 1.
86 Problema Demostrar que 271958 108878 + 101528 es exactamente divisible por

99 Problema Demostrar que si 2n + 1 es un nmero primo, entonces n es una potencia


de 2. Primos de esta forma se llaman primos de Fermat.

100 Problema Demuestre que

26460.
a2 + b2 + c2 ab bc ca =


1
(a b)2 + (b c)2 + (c a)2 .
2

87 Problema Demostrar que si k es un entero positivo impar

1k + 2k + + nk
es divisible por
1 + 2 + + n.
88 Problema Demostrar que 1492n 1770n 1863n +2141n es divisible por 1946 para
todo entero positivo n.

101 Problema Demostrar que

a3 + b3 + c3 3abc = (a + b + c)(a2 + b2 + c2 ab bc ca).

102 Problema Demostrar que

(x + y)5 x5 y5 = 5xy(x + y)(x2 + xy + y2 ).


89 Problema Dividir x128 y128 por

(x + y)(x2 + y2 )(x4 + y4 )(x8 + y8 )(x16 + y16 )(x32 + y32 )(x64 + y64 ).

103 Problema Demostrar que

(x + a)7 x7 a7 = 7xa(x + a)(x2 + xa + a2 )2 .


90 Problema Halle la suma de los factores primos de 216 1.
104 Problema Demostrar que
91 Problema Dado que 1002004008016032 tiene un factor primo p > 250000, hllelo.

A = x9999 + x8888 + x7777 + + x1111 + 1


92 Problema Si a3 b3 = 24, a b = 2, halle el valor de (a + b)2 .

93 Problema Hallar

94 Problema Hallar

p
p

11 + 72.

10 + 4i 6.

es divisible por B = x9 + x8 + x7 + + x2 + x + 1.
105 Problema La diferencia

p
es un entero. Hllelo.

57 40 2
57 + 40 2

18

Captulo 2

2.2. Los enteros


Una de las propiedades ms tiles de los enteros es la expresada por el algoritmo de divisin:
106 Teorema (Algoritmo de divisin) Sean a, b enteros con a > 0. Entonces existen enteros q y r con

b = aq + r, 0 r < a.
Por ejemplo, 39 = 4 9 + 3. Vemos pues que el algoritmo de divisin discrimina a los enteros segn el residuo que dejan al ser
divididos por a. Por ejemplo, si a = 2, descomponemos a los enteros en las dos familias
A0 = {. . . 4, 2, 0, 2, 4, . . .},
A1 = {. . . , 5, 3, 1, 1, 3, 5, . . .}.
As pues todo entero es de la forma 2k o 2k + 1. Observe que todo entero de la forma 2k + 1 es tambin de la forma 2t 1.
Si a = 4 entonces descomponemos a los enteros en las cuatro familias
B0 = {. . . , 8, 4, 0, 4, 8, . . .},
B1 = {. . . , 7, 3, 1, 5, 9, . . .},
B2 = {. . . , 6, 2, 2, 6, 10, . . .},
B3 = {. . . , 5, 1, 3, 7, 11, . . .}.
As pues, los enteros son de la forma 4k, 4k + 1, 4k + 2 o 4k + 3. Observe que todo entero de la forma 4k + 1 es tambin de la
forma 4t 3 y que todo entero de la forma 4k + 3 es tambin de la forma 4t 1.
107 Ejemplo Sea r el residuo cuando 1059, 1417 y 2312 se dividen por d > 1. Halle el valor de d r.

Resolucin: Por el algoritmo de divisin, existen enteros q1 , q2 , q3 con 1059 = dq1 + r, 1417 = dq2 + r y 2312 = dq3 + r.
Restando obtenemos 1253 = d(q3 q1 ), 895 = d(q3 q2 ) y 358 = d(q2 q1 ). Como 7 179, 895 = 5 179, 358 = 2 179, vemos
que d = 179. Como 1059 = 5 179 + 164, r = 164. Finalmente, d r = 15.
108 Ejemplo Demostrar que el cuadrado de todo entero es de la forma 4k o de la forma 4k + 1.

Resolucin: Si el entero es par, es decir de la forma 2a, su cuadrado es (2a)2 = 4a2 , que es de la forma 4k. Si el entero es
impar, digamos 2t + 1, entonces (2t + 1)2 = 4(t 2 + t) + 1, que es de la forma 4k + 1.
109 Ejemplo Demostrar que ningn entero en la sucesin

11, 111, 1111, 11111, . . .


es el cuadrado de un entero.
Resolucin: Como es obvio que 11 no es un cuadrado, nos ocuparemos de los dems enteros en la sucesin. Para n > 2,
. . 11} 00 + 12 1 = 100 11
. . 11} +12 1.
. . . 1} = |11 .{z
|11{z
| .{z
n 1 s

n2 1 s

n2 1 s

As pues, todo nmero en esta sucesin es de la forma 4k 1. Pero por el ejercicio anterior, 4k 1 no puede ser el cuadrado de
ningn entero. Esto completa la demostracin.
110 Ejemplo Demuestre que n2 + 23 es divisible por 24 para un nmero infinito de nmeros n.

Los enteros

19

Resolucin: Tenemos que n2 + 23 = n2 1 + 24 = (n 1)(n + 1) + 24. Luego, las familias n = 24m 1, m = 0, 1, 2, 3, . . .


producen infinitos valores de n2 + 23 que son divisibles por 24.
111 Ejemplo Demostrar que todos los enteros en la sucesin

. . 88} 9
49, 4489, 444889, 44448889, |44 .{z
. . 44} 88
| .{z
n 4 s

n1 8 s

son cuadrados.
Resolucin: Observe que
44
. . 44} 88
. . 88} 9
| .{z
| .{z
n 4 s

44
. . 44} 10n + 88
. . 88} 10 + 9
| .{z
| .{z
n 4 s

n1 8 s

=
=
=
=

n1 8 s

8
4
(10n 1) 10n + (10n1 1) 10 + 9
9
9
4
4
1
102n + 10n +
9
9
9
1
(2 10n + 1)2
9

2 10n + 1 2
3

Nos falta demostrar que esta ltima cantidad es entera, esto es, que 3 divide a 2 10n + 1 = 2 00
. . 00} 1. Pero la suma de los
| .{z
dgitos de esta ltima cantidad es 3, y por lo tanto este entero es divisible por 3.

n1 0 s

112 Ejemplo Demostrar que el cuadrado de todo primo mayor que 3 deja residuo 1 al ser dividido por 12.

Resolucin: Si p > 3 es primo, entonces p es de la forma 12k 1, 12k 5. Ahora bien


(12k 1)2 = 12(12k2 2k) + 1
y
(12k 5)2 = 12(12k2 10k + 2) + 1.
Esto demuestra la asercin.
113 Ejemplo Demostrar que si ambos p y 8p 1 son primos, entonces 8p + 1 es compuesto.

Resolucin: Si p = 3, 8p 1 = 23 y 8p + 1 = 25, luego la aseveracin se cumple para p = 3. Si p > 3, p es de la forma


3k + 1 o 3k + 2. Si p = 3k + 1, 8p 1 = 24k 7 y 8p + 1 = 24k 6, que es divisible por 6 y por lo tanto no es primo. Si
p = 3k + 2, 8p 1 = 24k 15 no es primo.
114 Ejemplo Demostrar que si n es un entero positivo tal que 2n+1 es un cuadrado, entonces n+1 es la suma de dos cuadrados

consecutivos.
Resolucin: Como 2n + 1 es un cuadrado impar, tenemos 2n + 1 = (2t + 1)2 para algn entero t. Resolviendo para n,
n=

(2t + 1)2 1
= 2t 2 + 2t.
2

Luego n + 1 = t 2 + (t + 1)2 , la suma de dos cuadrados consecutivos.


115 Ejemplo Demostrar que si 3n + 1 es un cuadrado, entonces n + 1 es la suma de tres cuadrados.

20

Captulo 2

Resolucin: Es claro que 3n + 1 no es un mltiplo de 3, luego 3n + 1 = (3k 1)2 . De aqu


n+1 =

(3k 1)2 1
+ 1 = 3k2 2k + 1 = k2 + k2 + (k 1)2 ,
3

como queramos demostrar.


116 Ejemplo Hallar todos los enteros con dgito inicial 6 tales que si se les suprime este dgito incial,el nmero resultante es

1/25 del nmero original.


Resolucin: Sea x el entero buscado. Entonces x = 6 10n + y donde y es un entero positivo. La condicin del problema estipula
que
1
(6 10n + y) ,
y=
25
o sea,
10n
y=
= 25 10n2 .
4
Esto requiere n 2 y por lo tanto y = 25, 250, 2500, 25000, etc.. Luego x = 625, 6250, 62500, 625000, etc..
117 Ejemplo Sea A un entero positivo y A sea el entero positivo resultante de alguna permutacin especfica de los dgitos de

A. Demostrar que si A + A = 1010 entonces A es divisible por 10.

Resolucin: Claramente, A y A debern tener 10 dgitos cada uno. Pongamos pues


A = a10 a9 a8 . . . a1
y
A = b10 b9 b8 . . . b1 ,
donde ak , bk , k = 1, 2, . . . , 10 son los dgitos de A y A respectivamente. Ahora, como A + A = 10000000000, deberemos tener
que a1 + b1 = a2 + b2 = = ai + bi = 0 y
ai+1 + bi+1 = 10, ai+2 + bi+2 = = a10 + b10 = 9,
para algn subndice i, 0 i 9. Note que si i = 9 no hay ninguna suma de las ai+2 + bi+2 , ai+3 + bi+3 , . . . y si i = 0 no hay
ninguna suma de las a1 + b1 , . . . , ai + bi .
Sumando,
a1 + b1 + a2 + b2 + + ai + bi + ai+1 + bi+1 + + a10 + b10 = 10 + 9(9 i).
Ahora bien, si i es par, 10 + 9(9 i) es impar y si i es impar 10 + 9(9 i) es par. Pero como
a1 + a2 + + a10 = b1 + b2 + + b10 ,
tenemos
a1 + b1 + a2 + b2 + + ai + bi + ai+1 + bi+1 + + a10 + b10 = 2(a1 + a2 + + a10 ),

un entero par. Colegimos que i es impar, lo que necesariamente implica a1 = b1 = 0, esto es, A y A son ambos divisibles por
10.
118 Ejemplo Cuntos ceros hay al final de 999!?

Resolucin: El nmero de ceros est determinado por la potencia mayor de 10 que divide a 999!. Como hay menos mtiplos
de 5 en {1, 2, . . . , 999} que mltiplos de 2, el nmero de ceros est pues determinado por la potencia mayor de 5 que divide a
999!. Esta es

999
999
999
999
+
+
+
= 199 + 39 + 7 + 1 = 246.
5
52
53
54
Por lo tanto, 999! termina en 246 ceros.

Tarea

21

119 Ejemplo La suma de enteros positivos es 1996. Cul es el valor mximo de su producto?

Resolucin: Tenemos enteros positivos a1 , a2 , . . . , an con a1 + a2 + + an = 1996. Es claro que para maximizar a1 a2 an ,
ninguna de las ak s puede ser igual a 1. Demostraremos que para obtener un producto mximo deberemos tener la mayora de
las ak = 3 y a lo sumo dos a j = 2. Supongamos que a j > 4. Si substituimos a j por los dos trminos a j 3 y 3 la suma no se
afecta, pero el producto incrementa pues a j < 3(a j 3). As pues las ak s son iguales a 2, 3 4. Pero como 2 + 2 + 2 = 3 + 3
y 2 2 2 < 3 3, si hay tres o ms 2s, los podemos substituir con 3s. Como 1996 = 3(665) + 1 = 3(664) + 4, el producto
mximo es pues 3664 4.
120 Ejemplo Demostrar que el producto de cuatro enteros consecutivos es siempre divisible por 24.

Resolucin: Sean n 1, n, n + 1, n + 2 los cuatro enteros consecutivos. Uno de ellos es divisible por 3, uno de ellos es de la
forma 4k (y por lo tanto divisible por 4) y otro de ellos es de la forma 4a + 2 (y por ende divisible por 2). Luego el producto es
divisible por 3 4 2 = 24.
121 Ejemplo Demostrar que el producto de cuatro enteros consecutivos, diferentes de 0, jams es un cuadrado.

Resolucin: Sean n 1, n, n + 1, n + 2 cuatro enteros consecutivos. Entonces su producto P es


P = (n 1)n(n + 1)(n + 2) = (n3 n)(n + 2) = n4 + 2n3 n2 2n.
Ahora bien,
(n2 + n 1)2 = n4 + 2n3 n2 2n + 1 = P + 1 > P.
Como P 6= 0 y P es 1 ms que un cuadrado, P no puede ser un cuadrado.
122 Ejemplo Hallar todos los enteros positivos de la forma

1
r+ ,
r
donde r es un nmero racional.
Demostraremos que la expresin r + 1/r es entero slo cuando r = 1, en cuyo caso r + 1/r = 2. Sea pues
r+

1
= k,
r

k un entero positivo. Luego

k k2 4
r=
.
2
Como k es un entero, r puede ser entero si y slo si k2 4 es un cuadrado de la misma paridad que k. Ahora, si k 3,
(k 1)2 < k2 4 < k2 ,

esto es, k2 4 est entre dos cuadrados consecutivos y por lo tanto no puede ser un cuadrado. Si k = 1,
k = 2, k2 4 = 0. Luego, r + 1/r = 2, esto es, r = 1. Esto termina la demostracin.

k2 4 no es real. Si

123 Ejemplo Para cuntos enteros n en {1, 2, 3, . . . , 100} es el dgito de las decenas de n2 impar?

Resolucin: En el subconjunto {1, 2, . . . 10} hay slo dos valores de n (4 y 6) para los cuales el dgito de las decenas de n2 es
impar. Ahora bien, (n + 10)2 = n2 + 20n + 100 tiene la misma paridad en su dgito de las decenas que el dgito de las decenas
de n2 . Luego, hay 2 10 = 20 enteros n para los cuales se verifica la condicin prescrita.

Tarea

22

Captulo 2
130 Problema Demostrar que la suma de todos los enteros de n dgitos, n 3, es

124 Problema Sea a el entero

a = 111 . . . 1

| {z }

494 99 . . . 9 55 00 . . . 0 .

|{z} |{z}

m 1 s

n3 9 s

y sea b el entero

n2 0 s

b = 1 000 . . . 0 5.

| {z }
m1 0 s

131 Problema Demostrar que para todo entero positivo n,

Demostrar que ab + 1 es un cuadrado perfecto.


11 . . . 1 22 . . . 2

|{z} |{z}
2n 1 s

125 Problema Demostrar que el cuadrado de un entero es de la forma 3k o 3k + 1. Lue-

go demostrar que si los lados de un tringulo rectngulo son enteros, entonces 3 divide a
alguno de los lados.

n 2 s

es un cuadrado.

126 Problema Hallar la suma

132 Problema Demostrar que para todo nmero a 6= 0, a 6= i 3 se verifica la frmula

de Reyley (1825):
5 + 55 + 555 + + 5 . . . 5 .

|{z}

n 5 s

a=

127 Problema Qu dgitos aparecen en el producto

3 
+

a2 + 30a2 9
6a(a2 + 3)

3 
+

6a3 + 18a
(a2 + 3)2

3
.

Si a es racional, esto demuestra que todo nmero racional puede expresarse como la suma
de tres cubos de nmeros racionales.

3...3 6...6 ?
|{z}
|{z}
666 3 s

a6 + 45a5 81a2 + 27
6a(a2 + 3)2

666 6 s

133 Problema Demostrar que para n 2, la expresin


128 Problema Demostrar que no existe ningn entero con la propiedad de que si su

n3 + (n + 2)3
4

dgito inicial se suprime, el entero resultante es 1/35 del entero inicial.


129 Problema Cul es la potencia mayor de 7 que divide a 1000!?

es un entero compuesto.

2.3. Aritmtica modular


Comenzaremos primero con la siguiente definicin. Si a 6= 0 es un entero, decimos que a divide al entero b (escrito a|b) si
existe un entero k con ak = b. Por ejemplo, 11|99 porque 11 9 = 99.
Las siguientes propiedades de divisibilidad son obvias. Sean a, b, c, x, y enteros. Entonces
ab 6= 0, a|b, b|c = a|c

(2.1)

a 6= 0, a|b, a|c = a|(xb + yc)

(2.2)

Por ejemplo, 11|99 y 33|330 implica que 11|330.

Por ejemplo, 7|21 y 7|49 implica que 7 divide a 3 21 2 49 = 35.


Si a no divide a b escribimos a 6 |b. Note adems que a|c, b|c no necesariamente implica que ab|c. Por ejemplo, 2|6, 6|6
pero claramente 12 = 2 6 6 |6.
Dado un entero n 2, el algoritmo de divisin distribuye los enteros en una de n clases dependiendo del residuo que deje el
entero al ser dividido por n. Si u y v dejan el mismo residuo al ser divididos por n, o de manera equivalente, si u v es divisible
por n, entonces decimos que u y v son congruentes mdulo n y escribimos u v mod n. Por ejemplo, 3 13 26 7 mod
10.
Notamos de paso que si u v mod n, entonces u = v + an para algn entero a. Por ejemplo, 3 24 mod 7 y 3 = 24 + (3)7.
El siguiente teorema es de suma utilidad.
134 Teorema Sea n 2 un entero. Si x y mod n y u v mod n entonces

ax + bu ay + bv mod n.
Demostracin Como n|(x y), n|(u v) entonces hay enteros s,t con ns = x y, nt = u v. Luego
a(x y) + b(u v) = n(as + bt),
es decir,
n|(ax + bu ay bv).

Aritmtica modular

23

Esto ltimo es equivalente a


ax + bu ay + bv mod n.

135 Corolario Sea n 2 un entero. Si x y mod n y u v mod n entonces

xu yv mod n.
Demostracin Pongamos a = u, b = y en el teorema anterior.
136 Corolario Sea n > 1 un entero, x y mod n y j un entero positivo. Entonces x j y j mod n.
137 Ejemplo Hallar el residuo cuando 61987 es dividido por 37.

Resolucin: 62 1 mod 37. As pues, 61987 6 61986 6(62 )993 6(1)993 6 31 mod 37.
138 Ejemplo Hallar el residuo cuando

12233 455679 + 876533


es dividido por 4.
Resolucin: 12233 = 12200 + 32 + 1 1 mod 4. De manera semejante, 455679 = 455600 + 76 + 3 3, 87653 = 87600 +
52 + 1 1 mod 4. As
12233 455679 + 876533 1 3 + 13 4 0 mod 4.

O sea, que 12233 455679 + 876533 es divisible por 4.


139 Ejemplo Hallar el ltimo dgito de 3100 .

Resolucin: Queremos hallar 3100 mod 10. Observemos que 32 1 mod 10. Luego, 3100 = (32 )50 (1)50 1 mod 10.
As, el ltimo dgito es el 1.
140 Ejemplo Demostrar que 7|(22225555 + 55552222 ).

Resolucin: 2222 3 mod 7, 5555 4 mod 7 y 35 5 mod 7. Ahora bien, 22225555 + 55552222 35555 + 42222 (35 )1111 +
(42 )1111 51111 51111 0 mod 7, lo que demuestra la asercin.
7

141 Ejemplo Hallar el dgito de las unidades de 77 .


7

Resolucin: Tenemos que hallar 77 mod 10. Ahora bien, como 72 1 mod 10, entonces tenemos 73 72 7 7 3 mod
10 y 74 (72 )2 1 mod 10. Adems, 72 1 mod 4 y por lo tanto 77 (72 )3 7 3 mod 4, lo que quiere decir que hay un
entero t tal que 77 = 3 + 4t. Ensamblando todo esto,
7

77 74t+3 (74 )t 73 1t 3 3 mod 10.


As el ltimo dgito es un 3.
142 Ejemplo Demostrar que 7 divide a 32n+1 + 2n+2 para todo nmero natural n.

Resolucin: Observemos que 32n+1 3 9n 3 2n mod 7 y 2n+2 4 2n mod 7. Luego


32n+1 + 2n+2 7 2n 0 mod 7,

24

Captulo 2

para todo nmero natural n.


143 Ejemplo Qu dgitos debe substituirse por a y b en 30a0b03 de tal manera que el entero resultante sea divisible por 13 ?

Resolucin: Como 30a0b03 = 3 + 100b + 10000a + 3000000, observamos que 30a0b03 3 + 9b + 3a + 3 6 + 9b + 3a mod
13. Para que 30a0b03 sea divisible por 13 necesitamos 9b + 3a 7 mod 13. Aqu claro est, debemos tener 0 a, b 9. Por
inspeccin vemos que a = 8, b = 1; a = 5, b = 2; a = 2, b = 3; a = 9, b = 5; a = 6, b = 6; a = 3, b = 7; a = 0, b = 8 Luego 3080103,
3050203, 3020303, 3090503, 3060603, 3030703, 3000803 son todos divisibles por 13.
144 Ejemplo Hallar los cuadrados mod 13.

Resolucin: Observemos primero que slo necesitamos cuadrar los enteros hasta 6, porque r2 (13 r)2 mod 13. Cuadrando
los enteros no negativos hasta el 6, obtenemos 02 0, 12 1, 22 4, 32 9, 42 3, 52 12, 62 10 mod 13. Por lo tanto, los
cuadrados mod 13 son 0, 1, 4, 9, 3, 12, y 10.
145 Ejemplo Demostrar que la ecuacin x2 5y2 = 2 no tiene soluciones enteras.

Resolucin: Si x2 = 2 5y2 , entonces x2 2 mod 5. Pero 2 no es un cuadrado mod 5.


146 Ejemplo Demostrar la siguiente observacin de Euler: 232 + 1 es divisible por 641.

Resolucin: Observemos que 641 = 27 5 + 1 = 24 + 54 . Luego 27 5 1 mod 641 y 54 24 mod 641. Ahora bien,
27 5 1 mod 641 nos da 54 228 = (5 27 )4 (1)4 1 mod 641. Esta tima congruencia y 54 24 mod 641 nos da
24 228 1 mod 641, lo que significa que 641|(232 + 1).
147 Ejemplo Hallar un nmero infinito de enteros n tal que 2n + 27 sea divisible por 7.

Resolucin: Observemos que 21 2, 22 4, 23 1, 24 2, 25 4, 26 1 mod 7 y as 23k 1 mod 3 para todos los enteros
positivos k. Luego 23k + 27 1 + 27 0 mod 7 para todos los enteros positivos k. Esto produce una familia infinita de enteros
n = 3k, k = 1, 2, . . . tal que 2n + 27 es divisible por 7.
148 Ejemplo Existen acaso enteros positivos x, y tales que x3 = 2y + 15?

Resolucin: No. Los cubos mod 7 son 0, 1, y 6. Ahora bien, cada potencia de de 2 es congruente a 1, 2, 4 mod 7. As pues,
2y + 15 2, 3, or 5 mod 7. Esto es imposible.
149 Ejemplo Demostrar que 2k 5, k = 0, 1, 2, . . . nunca deja residuo 1 cuando es dividido por 7.

Resolucin: 21 2, 22 4, 23 1 mod 7 y este ciclo de tres se repite. As pues, 2k 5 deja residuos 3, 4, 6 al ser dividido
por 7.
150 Ejemplo (USAMO 1979) Determine todas las soluciones no negativas

(n1 , n2 , . . . , n14 )
de la ecuacin diofntica
n41 + n42 + + n414 = 1599
de haberlas.

Tarea

25

Resolucin: No hay tales soluciones. Todas las cuartas potencias mod 16 son o bien 0 o bien 1 mod 16. Esto significa
que
n41 + + n414
es a lo sumo 14 mod 16. Pero 1599 15 mod 16.
Usando congruencias y el sistema de numeracin decimal podemos obtener varias reglas de divisibilidad. La ms famosa
es quizs la siguiente.
151 Teorema Regla de los 9s Un nmero natural n es divisible por 9 si y slo si la suma de sus dgitos es divisible por 9.

Demostracin Sea n = ak 10k + ak1 10k1 + + a1 10 + a0 la expansin en base-10 de n. Como 10 1 mod 9, tenemos
10 j 1 mod 9. Colegimos que n = ak 10k + + a1 10 + a0 ak + + a1 + a0 , de donde resulta la asercin.
152 Ejemplo (AHSME 1992) Los enteros de dos dgitos desde el 19 hasta el 92 se escriben consecutivamente para obtener el

entero
192021222324 89909192.
Cul es la potencia mayor de 3 que divide a este nmero?
Resolucin: Por la regla de los 9s este nmero es divisible por 9 si y slo si
19 + 20 + 21 + + 92 = 372 3
lo es. Por lo tanto, el nmero es divisible por 3 pero no por 9.
153 Ejemplo (IMO 1975) Cuando 44444444 se escribe en notacin decimal, la suma de sus dgitos es A. Sea B la suma de los

dgitos de A. Hallar la suma de los dgitos de B. (A y B se escriben en notacin decimal.)


Resolucin: Tenemos que 4444 7 mod 9, y por lo tanto 44443 73 1 mod 9. As 44444444 = 44443(1481) 4444 17 7
mod 9. Sea C la suma de los dgitos de B.
Por la regla de los 9s, 7 44444444 A B C mod 9. Ahora bien, 4444 log10 4444 < 4444 log10 104 = 17776. Esto
significa que 44444444 tiene a lo sumo 17776 dgitos, as la suma de los dgitos de 44444444 es a lo sumo 9 17776 = 159984,
de aqu A 159984. Entre los nmeros naturales 159984 el que tiene la suma mximal de sus dgitos es 99999, de donde
colegimos que B 45. De todos los enteros naturales 45, 39 tiene la mxima suma dgitoal, es decir 12. As la suma de los
dgitos de B es a lo sumo 12. Pero como C 7 mod 9, se sigue que C = 7.
Las congruencias mod 9 a veces pueden ser usadas para verificar multiplicaciones. Por ejemplo, 8759612753 6= 2410520633,
ya que si esto fuese cierto entonces
(8 + 7 + 5 + 9 + 6 + 1)(2 + 7 + 5 + 3) 2 + 4 + 1 + 0 + 5 + 2 + 0 + 6 + 3 + 3 mod 9.
Pero esto dice que 0 8 8 mod 9, que es patentemente falso.
Se puede establecer un criterio de divisibilidad por 11 de una manera semejante. Sea n = ak 10k + ak1 10k1 + + a1 10 +
a0 . Como 10 1 mod 11, tenemos 10 j (1) j mod 11. Por lo tanto n (1)k ak + (1)k1 ak1 + a1 + a0 mod 11,
o sea, n es divisible por 11 si y slo si la suma alternante de sus dgitos es divisible por 11. Por ejemplo, 912282219
9 1 + 2 2 + 8 2 + 2 1 + 9 7 mod 11 y as 912282219 no es divisible por 11, mientras que 8924310064539 8 9 +
2 4 + 3 1 + 0 0 + 6 4 + 4 3 + 9 0 mod 11, y as 8924310064539 es divisible por 11.

Tarea

26

Captulo 2

154 Problema Si 62ab427 es un mltiplo de 99, hallar los dgitos a y b.

169 Problema Demostrar que si 7|a2 + b2 entonces 7|a y 7|b.

155 Problema Demostrar que un nmero natural es divisible por 2n , n = 1, 2, 3, . . . si y


slo si el nmero formado por sus ltimos n dgitos es divisible por 2n .

170 Problema Demostrar que no hay enteros con

800000007 = x2 + y2 + z2 .
156 Problema Hallar el ltimo dgito de

2333333334 9987737 + 12 21327 + 12123 99987.


157 Problema Demostrar que la ecuacin
2

171 Problema Demostrar que la suma de los dgitos, en notacin decimal, de un cuadrado, no puede ser igual a 1991.

172 Problema Demostrar que


2

x + 3xy 2y = 122

7|42 + 22 + 1

no posee soluciones enteras.

para todos los nmeros naturales n.

158 Problema Hallar cuntas n, 1 n 25 poseen la propiedad que n2 + 15n + 122 es

divisible por 6.

Pista: n2 + 15n + 122 n2 + 3n + 2 = (n + 1)(n + 2) mod 6.


159 Problema Demostrar que en cualquier subconjunto de 55 elementos tomado del

173 Problema Cuntos cuadrados hay mod 2n ?

174 Problema Demostrar que los no-mltiplos de 3 son potencias de 2 mod 3n .

175 Problema (USAMO 1986) Cul es el menor entero n > 1, para el cual

conjunto {1, 2, 3, . . . , 100}, siempre se encontrarn dos elementos que diferirn por 9.
160 Problema (AIME 1994) La sucesin creciente

12 + 22 + + n2
n

1/2

es un entero?

3, 15, 24, 48, . . . ,


consiste de aquellos mtiplos de 3 que son uno menos de un cuadrado. Cul es el residuo
cuando el 1994avo trmino de esta sucesin se divide por 1000?

176 Problema Hallar todos los enteros a, b, a > 1 y todos los primos p, q, r que satisfa-

cen la ecuacin
pa = qb + ra

161 Problema Demostrar que para cualesquiera a, b, c Z, n N, n > 3, existe un en-

(a, b, p, q, r no son necesariamente diferentes).

162 Problema (AIME 1983) Sea an = 6n + 8n . Determine el residuo cuando a83 se di-

(n 1)2 .

tero k tal que n 6 |(k + a), n 6 |(k + b), n 6 |(k + c).

vide por 49.


163 Problema Demostrar que si 9|(a3 + b3 + c3 ), entonces 3|abc, para enteros a, b, c.

177 Problema Si n > 1 es un entero, demostrar que nn n2 + n 1 es divisible por

178 Problema (Putnam 1952) Sea


n
X

10

164 Problema Describa todos los enteros n tal que 10|n

f (x) =

+ 1.

ak xnk

k=0

165 Problema Demostrar que si


2

a b, a b , a b , a b , . . .

un polinomio de grado n con coeficientes enteros. Si a0 , an y f (1) son todos nones, demostrar que f (x) = 0 no tiene races racionales.

son todos enteros, entonces a y b son tambin enteros.

179 Problema (AHSME 1991) Un entero de n dgitos es lindo si sus n dgitos son una

166 Problema Hallar los ltimos dos dgitos de 3100 .

permutacin del conjunto {1, 2, . . . , n} y sus primeros k dgitos forman un entero que es
divisible por k para toda k, 1 k n. Por ejemplo, 321 es lindo de tres dgitos ya que 1
divide a 3, 2 divide a 32 y 3 divide a 321. Cuntos enteros lindos de seis dgitos hay?

167 Problema (AHSME 1992) Cul es el tamao del subconjunto mayor S de


{1, 2, . . . , 50} tal que ningn par de elementos distintos de S tenga una suma divisible
por 7?

180 Problema Un viejo recibo est algo borroso. Dice que 88 pollos costaban un total
de $x4,2y, donde x, y son dgitos ilegibles. Cunto costaba cada pollo?

168 Problema Demostrar que la ecuacin x2 7y = 3 no tiene soluciones enteras.

por 3n .

181 Problema Demostrar que un entero que consiste de 3n dgitos idnticos es divisible

Captulo

Combinatoria
3.1. Las reglas de la multiplicacin y la suma
182 Definicin (Cardinalidad de un conjunto) Si S es un conjunto, entonces su cardinalidad es el nmero de elementos que
ste tiene. Se denotar la cardinalidad de S por card (S).

Comenzaremos nuestro estudio de la combinatoria con los siguientes dos principios fundamentales.
183 Regla (Regla de la suma: Forma disyuntiva) Sean E1 , E2 , . . . , Ek , conjuntos finitos mutuamente disjuntos, esto es Ek

E j = si k 6= j. Entonces

card (E1 E2 Ek ) = card (E1 ) + card (E2 ) + + card (Ek ) .


184 Regla (Regla del producto) Sean E1 , E2 , . . . , Ek , conjuntos finitos. Entonces

card (E1 E2 Ek ) = card (E1 ) card (E2 ) card (Ek ) .


185 Ejemplo Cuntos pares ordenados de enteros (x, y) hay tales que 0 < |xy| 5?

Resolucin: Pngase Ek = {(x, y) Z2 : |xy| = k} para k = 1, . . . , 5. El nmero deseado es


card (E1 ) + card (E2 ) + + card (E5 ) .
Entonces
E1
E2
E3
E4
E5

=
=
=
=
=

{(1, 1), (1, 1), (1, 1), (1, 1)}


{(2, 1), (2, 1), (1, 2), (1, 2), (1, 2), (1, 2), (2, 1), (2, 1)}
{(3, 1), (3, 1), (1, 3), (1, 3), (1, 3), (1, 3), (3, 1), (3, 1)}
{(4, 1), (4, 1), (2, 2), (2, 2), (1, 4), (1, 4), (1, 4), (1, 4), (2, 2), (2, 2), (4, 1), (4, 1)}
{(5, 1), (5, 1), (1, 5), (1, 5), (1, 5), (1, 5), (5, 1), (5, 1)}

Por lo tanto 4 + 8 + 8 + 12 + 8 = 40 es el nmero deseado.


186 Ejemplo Se escribe los divisores positivos de 400 en manera creciente:

1, 2, 4, 5, 8, . . . , 200, 400.
Cuntos enteros hay en esta sucesin? Cuntos de stos son cuadrados?
27

28

Captulo 3

Resolucin: Como 400 = 24 52 , cualquier divisor de 400 habr de ser de la forma 2a 5b en donde 0 a 4 y 0 b 2.
Luego hay 5 maneras de elegir a a y 3 maneras de elegir a b, dando un total de 5 3 = 15 divisores positivos.
Para ser un cuadrado, un divisor de 400 deber tener la forma 2 5 con {0, 2, 4} y {0, 2}. As pues, hay 3 2 = 6
divisores de 400 que son adems cuadrados.
Arguyendo de la manera mostrada en el ejemplo 186, se obtiene el siguiente teorema.
187 Teorema Tenga el entero positivo n la factorizacin en primos
a

n = pa11 pa22 pk k ,
en donde los primos pi son distintos y las enteros ai son 1. Si d(n) denota el nmero de divisores positivos de n, entonces
d(n) = (a1 + 1)(a2 + 1) (ak + 1).
188 Ejemplo (AHSME 1977) Cuntos caminosconsistiendo de una sucesin de segmentos verticales u horizontales y cada
segmento uniendo un par adyacente de letrasen la figura 3.1 deletrean CONT EST ?

C
C O
C O N
C O N T
C O N T E
O N T E S

C
O C
N O C
T N O C
E T N O C
S E T N O C
T S T E N O C

Figura 3.1: Problem 188.

C
C O
C O N
C O N T
C O N T E
O N T E S

C
O
N
T
E
S
T

Figura 3.2: Problem 188.

Resolucin: Divdase el diagrama como en la figura 3.2. Ya que cada uno de los caminos requeridos debe utilizar la T de
abajo a la derecha, se contarn los caminos que comienzan con esta T hasta llegar a la C. Ya que hay seis filas ms a las que
se podr ir y en cada etapa se puede ir o hacia arriba o hacia la derecha, existen pues 26 = 64 caminos. La otra porcin del
diagrama contribuir 64 caminos ms. Pero la columna del medio es compartida por ambas porciones, as que hay un total de
64 + 64 1 = 127 caminos.
189 Ejemplo Se escriben los enteros del 1 al 1000 en sucesin. Hllese la suma de todos los dgitos.

Resolucin: Al escribir los enteros del 000 al 999 (con tres dgitos), se utilizan 3 1000 = 3000 dgitos. Cada uno de los 10
dgitos est representado de una manera igual y uniforme, as que cada dgito se utiliza 300 veces. La suma de los dgitos del
000 al 999 es pues
(0 + 1 + 2 + 3 + 4 + 5 + 6 + 7 + 8 + 9)(300) = 13500.
Por lo tanto, la suma de los dgitos al escribir del 1 al 1000 es 13500 + 1 = 13501.
Aliter: Pareando los enteros del 0 al 999 de la siguiente manera
(0, 999), (1, 998), (2, 997), (3, 996), . . . , (499, 500),
se ve que cada par tiene suma de dgitos 27 y que hay 500 pares. Aadiendo 1 por la suma de los dgitos de 1000, el total
requerido es pues
27 500 + 1 = 13501.

Las reglas de la multiplicacin y la suma

29

190 Ejemplo Determine cuntos enteros positivos de 3-dgitos pueden escribirse en notacin decimal que no tengan al 0 en su

expansin. Hllese tambin la suma de estos nmeros de 3-dgitos.


Resolucin: Hay 9 9 9 = 729 enteros positivos de 3-dgitos no poseyendo al 0 en su expansin decimal. Si 100x + 10y + z es
uno de estos enteros, entonces para cada seleccin fija de una variable, hay 9 9 = 81 selecciones de las otras dos variables.
Luego, la suma requerida es
81(1 + 2 + + 9)100 + 81(1 + 2 + + 9)10 + 81(1 + 2 + + 9)1 = 404595.
191 Ejemplo Determine cuntos enteros positivos de 3-dgitos pueden escribirse en notacin decimal poseyendo al menos un

0 en su expansin. Hllese tambin la suma de estos nmeros de 3-dgitos.


Resolucin: Utilizando el ejemplo 190, hay 900 729 = 171 tales enteros. La suma de todos los enteros de tres dgito es
100 + 101 + + 998 + 999.
Para obtener esta suma, observe que hay 900 sumandos y que la suma no cambia al cambiar el orden de los sumandos:
S
S
2S

dando S =

=
=
=
=

100
999
1099
900(1099),

+
+
+

101
998
1099

+
+
+

+
+
+

999
100
1099

900(1099)
= 494550. La suma requerida es pues 494550 404595 = 89955.
2

192 Ejemplo Todos los enteros positivos se escriben en sucesin

123456789101112131415161718192021222324 . . .
Qu dgito ocupa el 206790avo lugar?
Resolucin: Observemos que
1 9 + 2 90 + 3 900 + 4 9000 = 38819
y que
1 9 + 2 90 + 3 900 + 4 9000 + 5 90000 = 488819.
Por lo tanto, el dgito buscado est entre los nmeros de cinco dgitos. Si 5x + 38819 206790, entonces x 33595 (el entero
x es cunto nos adentramos en los nmeros de cinco dgitos). As pues, para llegar hasta el 206790avo dgito debemos de ir
hasta el 33595avo nmero de cinco dgitos, es decir 43594 (el primero es 10000). Luego, hasta el dgito final de 43594 (el 4 de
las unidades) hemos utilizado 38819 + 5 33595 = 206794 dgitos. Luego, el 4 ocupa la posicin 206794ava, el 9 la 206793ava,
el 5 la 206792ava, el 3 la 206791ava y el 4 la 206790ava. El dgito requerido es el 4.
193 Ejemplo Cuntos enteros del 1 al 1000000 tienen al menos un 1 en su expansin decimal?

Resolucin: Hay
8
8 9 = 72
8 9 9 = 648
8 9 9 9 = 5832
8 9 9 9 9 = 52488
8 9 9 9 9 9 = 472392

enteros positivos de 1-dgito,


enteros positivos de 2-dgitos,
enteros positivos de 3-dgitos,
enteros positivos de 4-dgitos,
enteros positivos de 5-dgitos,
enteros positivos de 6-dgitos,

30

Captulo 3

no poseyendo el dgito 1. As pues hay


8 + 72 + 648 + 5832 + 52488 + 472392 = 531440
entre el 1 y el 999999 no poseyendo el dgito 1. Luego hay 999999 531440 = 468559 poseyendo al menos un 1 y as 468559 +
1 = 468560 enteros entre el 1 y el 1000000 teniendo al menos un 1 en su expansin decimal.
Aliter: Analizaremos los enteros del 0 al 999999. Al resultado final le sumaremos 1, ya que 1000000 tiene un 1 en su
expansin.
Dividamos este conjunto de un milln de enteros como sigue: en 100000 decenas
{0, 1, 2, . . . , 9}
{10, 11, 12, . . . , 19}
..
.
{999990, 999991, . . . , 999999}.

En 10000 centenas

{0, 1, 2, . . . , 99}

{100, 101, 102, . . . , 199}


..
.
etc., hasta llegar a diez 100000enas

{999900, 999901, . . . , 999999},


{0, 1, 2, . . . , 99999}

{100000, 100001, 100002, . . . , 199999}


..
.
{900000, 900001, . . . , 999999}.

En la primera decena hay solamente un nmero, el 1, que tiene un 1 en su numeracin decimal. En la segunda decena, los diez
enteros tienen un 1 en su numeracin decimal.
En la primera centena, cada decena, excepto la segunda, contendr exactamente un entero que tiene un 1 en su expansin.
La segunda decena, claro est, tiene sus diez enteros con 1s en sus expansiones. En consecuencia, la primera centena tiene
10 + 9 1
enteros que tienen el 1 en sus expansiones.
En el primer millar, cada centena excepto la segunda tendr exactamente 10 + 9 1 enteros con el 1 en su expansin. La
segunda centena, que consiste de los enteros 100, 101, . . . 199 tendr sus 100 enteros con el 1 en su expansin. As pues, el
primer millar tendr exactamente
100 + 9(10 + 9 1) = 102 + 9 10 + 9

enteros con el 1 en su expansin.


En la primera decena de millar, cada millar excepto el segundo, tendr exactamente 102 + 9 10 + 9 enteros con el 1 en su
expansin. El segundo millar tendr sus 103 enteros con el 1 en su expansin. Luego, en la primera decena de millar hay
103 + 9(102 + 9 10 + 9) = 103 + 9 102 + 92 10 + 93
enteros con el 1 en su expansin.
Un razonamiento semejante nos lleva a concluir que en la primera centena de millar hay 104 + 9(103 + 9 102 + 92 10 +
3
9 ) = 104 + 9 103 + 92 102 + 93 10 + 94 enteros con el 1 en su expansin y en los primeros millones hay
105 + 9 104 + 92 103 + 93 102 + 94 10 + 95 =

106 96
= 468559
10 9

enteros con el 1 en su expansin. Esto quiere decir que en los enteros del 0 al 999999 hay 468559 enteros con el 1 en su
expansin y en los enteros del 1 al 1000000 hay 468559 + 1 = 468560 enteros con el 1 en su expansin.

Tarea

31

Tarea
194 Problema Se marcan n puntos, 1, 2, . . . , n sobre una circunferencia, que colocamos
a igual distancia unos de los otros. Si el punto marcado 15 est directamente opuesto al
marcado 49, cuntos puntos hay en total?
195 Problema Cuntos de los factores de 295 hay que sean mayores que 1, 000, 000?

201 Problema Cuntos enteros entre 1 y 3012 son divisibles por 5 o por 7 pero no por
ambos nmeros?

202 Problema Escribir la versin de cuatro conjuntos del principio de inclusin-

exclusin.

196 Problema Se escribe la sucesin de enteros positivos.

1, 2, 3, 4, 5, 6, 7, 8, 9, 10, 11, 12, 13, 14, 15, 16, 17, 18, 19, 20, . . .

203 Problema Cuntos nmeros primos hay entre 1 y 100?

Qu dgito ocupa la posicin 3000-ava?


204 Problema Sean x, y, z nmeros reales. Demostrar que
8 9 2

197 Problema Cuntos divisores positivos tiene 2 3 5 ? Cul es la suma de estos

max(x, y) = x + y mn(x, y)

divisores?
198 Problema Para escribir un libro se utilizaron 1890 dgitos. Cuntas pginas tiene

y que

el libro?
max(x, y, z) = x + y + z mn(x, y) mn(y, z) mn(z, x) + mn(x, y, z).
199 Problema Hallar d(1260), (1260) y (1260).

Qu relacin nota entre estas frmulas y el principio de inclusin-exclusin?


200 Problema Los enteros del 1 al 1000 se escriben en orden sobre un crculo. Comen-

zando con 1, cada quinceavo nmero es marcado (esto es, 1, 16, 31, etc.). Este proceso
se repite hasta que se marque un nmero por segunda vez. Cuntos nmeros sin marcar
quedan?

205 Problema Cuntos enteros entre 1 y 1000000 no son ni cuadrados, ni cubos, ni


cuartas, ni quintas potencias?

3.2. Mtodos combinatorios


La gran mayora de los problemas de conteo pertenecen a una de cuatro categoras, que explicaremos mediante el siguiente
ejemplo.
206 Ejemplo Considrese el conjunto {a, b, c, d}. Seleccinese dos letras de entre estas cuatro. Dependiendo de la interpreta-

cin se obtendr una de las siguientes cuatro respuestas.

Permutaciones con repeticin. El orden en que se listan las letras importa y se permite repetir letras. En este caso hay
4 4 = 16 selecciones posibles:
aa ab ac ad
ba bb bc bd
ca cb cc cd
da db dc dd
Permutaciones sin repeticin. El orden en que se listan las letras importa y no se permite repetir letras. En este caso
hay 4 3 = 12 selecciones posibles:
ab ac ad
ba
bc bd
ca cb
cd
da db dc
Combinaciones con repeticin. El orden en que se listan las letras no importa y se permite repetir letras. En este caso
43
hay
+ 4 = 10 selecciones posibles:
2
aa ab ac ad
bb bc bd
cc cd
dd

32

Captulo 3
Combinaciones sin repeticin. El orden en que se listan las letras no importa y no se permite repetir letras. En este caso
43
hay
= 6 selecciones posibles:
2
ab ac ad
bc bd
cd

Consideraremos ahora ejemplos de cada situacin.

3.2.1.

Permutaciones sin repeticin

207 Definicin Definimos el smbolo ! (factorial), como sigue: 0! = 1, y para entero n 1,

n! = 1 2 3 n.
n! se lee n factorial.
208 Ejemplo Se tiene

1!
2!
3!
4!
5!

=
=
=
=
=

1,
1 2 = 2,
1 2 3 = 6,
1 2 3 4 = 24,
1 2 3 4 5 = 120.

209 Definicin Sean x1 , x2 , . . . , xn n objetos distintos . Una permutacin de estos objetos es simplemente un rearreglo de ellos.
210 Ejemplo Hay 24 permutaciones de las letras de la palabra MAT H:

MAT H
AMT H
TAMH
HAT M

MAHT
AMHT
TAHM
HAMT

MTAH
AT MH
T MAH
HTAM

MT HA
AT HM
T MHA
HT MA

MHTA
AHT M
T HMA
HMTA

MHAT
AHMT
T HAM
HMAT

211 Teorema Sean x1 , x2 , . . . , xn n objetos distintos. Hay n! permutaciones de ellos.

Demostracin: La primera posicin de puede elegir de n maneras, la segunda en n 1 maneras, la tercera en


n 2, etc. Esto da
n(n 1)(n 2) 2 1 = n!.

212 Ejemplo Un librero tiene 5 libros alemanes, 7 libros espaoles y 8 libros franceses. Cada libro es diferente de cada otro.
Cuntos arreglos hay de estos libros?
Cuntos arreglos hay de estos libros si los libros deben estar
agrupados por lengua?
Cuntos arreglos hay de estos libros si todos los libros fran-

Resolucin:

ceses deben estar juntos?


Cuntos arreglos hay de estos libros si ningn libro francs
est junto a otro libro francs?

Mtodos combinatorios

33

Se permutan 5 + 7 + 8 = 20 objetos. Luego, el nmero de arreglos buscados es 20! = 2432902008176640000.


Agrpese los libros por lengua lo que garantizar que los libros de una misma lengua estarn juntos. Se permutan los 3
grupos en 3! maneras. Se permutan los libros alemanes en
5! maneras, los libros espaoles en 7! maneras y los libros
franceses en 8! maneras. Luego el nmero total de maneras es
3!5!7!8! = 146313216000.
Alniense los libros alemanes y los libros espaoles primero.
Al alinear estos 5 + 7 = 12 libros, se crean 12 + 1 = 13 espacios (incluyendo el espacio frente al primer libro, los espacios
entre los libros y el espacio luego del ltimo libro). Para asegurar que todos los libros franceses yagan uno al lado del
otro, pegamos los libros franceses y ponemos este bulto en
uno de los espacios. Ahora, los libros franceses se permutan
en 8! maneras y los libros no-franceses en 12! maneras. Luego

3.2.2.

el total number of permutaciones es


(13)8!12! = 251073478656000.
Alniense los libros alemanes y los libros espaoles primero.
Al alinear estos 5 + 7 = 12 libros, se crean 12 + 1 = 13 espacios (incluyendo el espacio frente al primer libro, los espacios
entre los libros y el espacio luego del ltimo libro). Para asegurar que ningn libro francs yaga junto a otro libro francs,
los ponemos en estos espacios. El primer libro francs puede
ponerse en cualquiera de 13 espacios, el segundo, en cualquiera de 12, etc., el octavo libro francs puede ponerse en algn
de los restantes 6 espacios. Ahora, los libros no-franceses pueden ser permutados en 12! maneras. As el de permutaciones
es
(13)(12)(11)(10)(9)(8)(7)(6)12!,
which es 24856274386944000.

Permutaciones con repeticin

Consideramos ahora permutaciones con objetos repetidos.


213 Ejemplo En cuntas maneras se pueden permutar las letras de la palabra

MASSACHUSET T S

Resolucin: Pngase subndices a las repeticiones, formando


MA1 S1 S2 A2CHUS3 ET1 T2 S4 .
Hay ahora 13 objetos distintos, los que pueden ser permutados en 13! maneras diferentes, gracias al teorema 211. En cada una
de estas 13! permutaciones, A1 A2 puede permutarse en 2! maneras, S1 S2 S3 S4 puede permutarse en 4! maneras y T1 T2 puede
permutarse en 2! maneras. As pues, el sobre-estimado de 13! es corregido y la cuenta final es
13!
= 64864800.
2!4!2!
Razonando de manera anloga al ejemplo 213, se podr demostrar el siguiente teorema.
214 Teorema Si hay k tipos de objetos: n1 del tipo 1; n2 del tipo 2; etc. Entonces el nmero de maneras en que estos n1 + n2 +

+ nk objetos pueden permutarse es

(n1 + n2 + + nk )!
.
n1 !n2 ! nk !

215 Ejemplo En cuntas maneras se pueden permutar las letras de la palabra MASSACHUSET T S en tal manera que la

partcula MASS est siempre junta, con las letras en este orden?
Resolucin: La partcula MASS se puede considerar como un bloque de una letra, agregando luego las 9 letras A, C, H, U,
S, E, T, T, S. En A, C, H, U, S, E, T, T, S hay cuatro Ss y dos T s, dando un total de permutaciones de
10!
= 907200.
2!2!
216 Ejemplo De cuntas maneras se puede escribir el 9 como la suma de tres enteros positivos? Se tendr cuenta del orden,

as pues 1 + 7 + 1 se cuenta aparte de 7 + 1 + 1, etc.

34

Captulo 3

Resolucin: Primero buscaremos soluciones del tipo


a + b + c = 9, 1 a b c 7
y veremos las permutaciones de cada tro. Se tiene
(a, b, c)

Nmero de permutaciones
3!
=3
2!
3! = 6
3! = 6
3!
=3
2!
3!
=3
2!
3! = 6
3!
=1
3!

(1, 1, 7)
(1, 2, 6)
(1, 3, 5)
(1, 4, 4)
(2, 2, 5)
(2, 3, 4)
(3, 3, 3)
El nmero deseado es as

3 + 6 + 6 + 3 + 3 + 6 + 1 = 28.
217 Ejemplo En cuntas maneras pueden arreglarse las letras de la palabra MURMUR de tal manera que letras idnticas no
sean adyacentes?

Resolucin: Si comenzramos con, dgase, MU entonces las R se pueden disponer de la manera siguiente:
M

U R

U R

R ,
R .

En el primer caso hay 2! = 2 de poner las M y la U restantes, en el segundo hay 2! = 2 y en el tercero hay solamente 1!. As, si
se comenzase con MU se tendra 2 + 2 + 1 = 5 arreglos posibles. En general, se puede elegir la primera letra de 3 maneras y
la segunda en 2 maneras. As pues el nmero de number of maneras pedidas es 3 2 5 = 30.

3.2.3.

Combinaciones sin repeticin


 

218 Definicin Sean n, k enteros no negativos, con 0 k n. El smbolo

n
(lase n tomando k) se define como
k

 

n!
n (n 1) (n 2) (n k + 1)
n
=
=
.
k!(n k)!
123k
k

Obsrvese que en esta ltima fraccin hay k factores tanto en el numerador como en el denominador. Obsrvese tambin los siguientes valores de frontera
 

 

n
n
=
= 1,
0
n

 

n
n
=
= n.
1
n1

Mtodos combinatorios

35

219 Ejemplo Se tiene

 

6
3 
11
2
12
7 
110
109
110
0

654
= 20,
123
11 10
= 55,
12
12 11 10 9 8 7 6
= 792,
1234567

=
=
=
=

110,

1.

Ya que n (n k) = k, se satisface para enteros n, k, 0 k n, la siguiente identidad de simetra


 

n!
n!
n
n
=
=
=
k!(n k)! (n k)!(n (n k))!
k
nk

sta se puede interpretar como sigue: el nmero de maneras de sacar k boletos de un sombrero que tiene n boletos
distintos es l mismo nmero de maneras de elegir n k boletos que permanezcan dentro del sombrero.
220 Ejemplo




11
11
=
= 55,
9
2

12
12
=
= 792.
5
7

221 Definicin Considrese n distintos. Una k-combinacin es una seleccin de k, (0 k n) objetos de entre los n sin tomar

en cuenta se orden.

222 Ejemplo Las 2-combinaciones de la lista {X,Y, Z,W } son

XY, XZ, XW,Y Z,YW,W Z.


223 Ejemplo Las 3-combinaciones de la lista {X,Y, Z,W } son

XY Z, XYW, XZW,YW Z.
224 Teorema Considrese
n objetos distintos y sea k un entero satisfaciendo 0 k n. El nmero de k-combinaciones de
 

estos n objetos es

n
.
k

Demostracin: Tmese cualesquiera k objetos. Si se tuviese en cuenta el order, entonces se producira una lista
con
n(n 1)(n 2) (n k + 1)
entradas, ya que hay n maneras de elegir el primer objeto, n 1 maneras de elegir el segundo, etc. Esta seleccin
particular de k objetos se puede permutar en k! maneras. Luego, el nmero total de k-combinaciones es
 

n(n 1)(n 2) (n k + 1)
n
=
.
k!
k

36

Captulo 3

225 Ejemplo Cuntos subconjuntos de tres elementos tiene el conjunto {a, b, c, d, f }?

Resolucin: Vea que aqu el orden carece de importancia. Lo que queremos es el nmero de maneras de seleccionar tres
elementos de cinco, el cual es
 
5
543
=
= 10.
3
123
226 Ejemplo De cuntas maneras podemos seleccionar un comit de tres personas de entre diez profesores?


Resolucin:

10
= 120
3

227 Ejemplo De cuntas maneras podemos seleccionar un comit de siete personas con un presidente de entre veinte perso-

nas?
Resolucin: Primero
As el nmero
 se selecciona al presidente de 20 maneras. Luego a los otros 6 de entre los 19
 restantes.

19
20
= 542640. O tambin podemos seleccionar a las 7 personas primero, de
maneras, y luego al
de maneras es 20
6
7


presidente de entre los 7, de 7 maneras. As el nmero de maneras es 7

20
= 542640, que concuerda con lo anterior.
7

228 Ejemplo De cuntas maneras podemos seleccionar un comit de siete personas con un presidente y un secretario de entre

veinte personas? El secretario no sirve de presidente.




Resolucin: De 20 19

18
20
= 76
= 3255840 maneras.
5
7

229 Ejemplo Se toman tres enteros diferentes del conjunto {1, 2, . . . , 20}. En cuntas formas se pueden tomar de tal manera
que su suma sea divisible por 3?

Resolucin: En {1, 2, . . . , 20} hay


6
7
7

nmeros que dejan residuo 0 al ser divididos por 3,


nmeros que dejan residuo 1 al ser divididos por 3,
nmeros que dejan residuo 2 al ser divididos por 3.

La suma de tres enteros ser divisible por 3 cuando (a) los tres enteros son divisibles por 3; (b) uno de los enteros es divisible
por 3, uno deja residuo 1 y el tercero deja residuo 2 al ser divididos por 3; (c) los tres dejan residuo 1 al ser divididos por 3;
(d) los tres dejan residuo 2 al ser divididos por 3. Luego el nmero de maneras es
 

   

6
6
+
3
1

7
1

 

 

7
7
7
+
+
= 384.
1
3
3

Mtodos combinatorios

37

230 Ejemplo Para contar el nmero de caminos de longitud mnima de A a B en la figura 3.3 obsrvese que cualquier camino de
longitud mnima deber consistir de 6 movimientos horizontales y de 3 verticales, para un total de 6 + 3 = 9 movidas. De estas
9 desplazamientos,
una vez se hallan elegido las 6 que sern horizontales, las 3 verticales quedan completamente determinadas.
 
9
Hay as
= 84 caminos.
6
231 Ejemplo Para contar el nmero de caminos
 de
 longitud mnima de A a B en la figura 3.4 que pasan por el punto O, se

cuenta el nmero de pasos de A a O (que son


 

5
3

= 20 en nmero) y el nmero de caminos que hay de O a B ( que son


  

4
5
= 4 en nmero). El nmero total de caminos lo es pues
3
3

4
= (20)(4) = 80.
3

B
O

Figura 3.3: Example 230.

3.2.4.

Figura 3.4: Example 231.

Combinaciones con repeticin

232 Teorema (De Moivre) Sea n un entero positivo. El nmero de soluciones enteras positivas de

x1 + x2 + + xr = n


es

n1
.
r1

Demostracin: Escrbase n como


n = 1 + 1 + + 1 + 1,
en donde hay n 1s y n 1 +s. Para descomponer a n en r sumandos slo se necesita escoger r 1 +s del total
de n 1, lo cual demuestra el teorema.
233 Ejemplo En cuntas maneras se puede escribir 9 como la suma de tres enteros estrictamente positivos? Aqu 1 + 7 + 1 se

tomar como diferente de 7 + 1 + 1.


Resolucin: Este es el ejemplo 216. Se buscan soluciones ntegras de
a + b + c = 9, a > 0, b > 0, c > 0.
Por el teorema 232 esto es

 

91
8
=
= 28.
31
2

38

Captulo 3

234 Ejemplo En cuntas maneras se puede escribir 100 como la suma de cuatro enteros estrictamente positivos?

Resolucin: Se buscan soluciones ntegras de


a + b + c + d = 100,
que gracias al teorema 232 son en total

99
= 156849.
3

235 Corolario Sea n un entero positivo. El nmero de soluciones enteras no negativas de

y1 + y2 + + yr = n
es

n+r1
.
r1

Demostracin: Pngase xr 1 = yr . Entonces xr 1. La ecuacin


x1 1 + x2 1 + + xr 1 = n
es equivalente a
x1 + x2 + + xr = n + r,
que por el teorema 232, tiene

n+r1
r1

soluciones.
236 Ejemplo Hllese el nmero de cuartetos (a, b, c, d) de enteros que satisfagan

a + b + c + d = 100, a 30, b > 21, c 1, d 1.


Resolucin: Pngase a + 29 = a, b + 20 = b. Entonces se desea el nmero de soluciones enteras estrictamente positivas de
a + 29 + b + 21 + c + d = 100,
o sea, de
a + b + c + d = 50.
Por el teorema 232 este nmero es

49
= 18424.
3

237 Ejemplo En cuntas maneras se puede escribir 1024 como el producto de tres enteros positivos?

Resolucin: Vase que 1024 = 210 . Se necesita una descomposicin de la forma 210 = 2a 2b 2c , esto es , se necesitan soluciones
ntegras de
a + b + c = 10, a 0, b 0, c 0.


10 + 3 1
12
=
= 66 en nmero.
Por el teorema 235 stas son
31
2

Principio de inclusin-exclusin

39

3.3. Principio de inclusin-exclusin


La regla de la adicin 183 da la cardinalidad de la reunin de conjuntos disjuntos. En esta seccin se mostrar como deducir
la cardinalidad de la reunin de conjuntos no necesariamente disjuntos.
El principio de inclusin-exclusin es atribudo tanto a Sylvester como a Poincar.
238 Teorema (Inclusin-exclusin para dos conjuntos)

card (A B) = card (A) + card (B) card (A B)


Demostracin: En el diagrama de Venn 3.5, sea R1 el nmero de elementos simultneamente en ambos conjuntos
(i.e., en A B), sea R2 el nmero de elementos en A pero no en B (i.e., en A \ B) y por R3 el nmero de elementos
en B pero no en A (i.e., en B \ A). Se tiene R1 + R2 + R3 = card (A B), lo cual demuestra el teorema.
C
R4
R6

A
R2

R1

R3

R2

Figura 3.5: Inclusin-exclusin para dos conjuntos.

R3
R5

R7
R1

Figura 3.6: Inclusin-exclusin para tres conjuntos.

239 Ejemplo De 40 personas, 28 fuman y 16 mascan tabaco. Adems, se sabe que 10 tanto fuman como mascan tabaco.
Cuntas personas ni fuman ni mascan tabaco?

Resolucin: Si X es un conjunto finito, denotaremos por card (X) su cardinalidad, esto es, el nmero de elementos que hay
en el conjunto. Sea A el conjunto de personas que fuman y B el conjunto de personas que mascan tabaco. Como card (A B) =
card (A) + card (B) card (A B) = 28 + 16 10 = 34, hay 34 personas o que fuman o que mascan tabaco. Por lo tanto, el
nmero de personas que ni fuma ni masca tabaco es 40 34 = 6.
240 Ejemplo Cuatrocientos nios forman un crculo y los numeramos 1, 2, . . . , 400. Sea k, 1 k 400 un entero fijo. Marca-

mos cada k nios detenindonos cuando marcamos a un nio por segunda vez. Por ejemplo, si k = 6, comenzamos marcando
los nins 6, 12, 18, . . . , 396. Luego nos toca marcar al nio 2, pues el sexto luego 396 es el 2. Seguimos marcando a los nios
8, 14, 20, . . . , 398. Nos toca ahora marcar a los nios 4, 10, 16, . . . , 400. El prximo nio a marcar es el sexto, que lo marcamos
pues por segunda vez. Notamos que dejamos sin marcar a los nios 1, 3, 5, 7, 9, 11, . . . 399esto es, los enteros de la forma
6k 1, 6k + 3 entre 1 y 400. Para cuntos valores de k sern marcados todos los nios al menos una vez?
Resolucin: Vemos que si k tiene un factor mayor que 1 en comn con 400, entonces no marcamos a todos los nios. As
pues, las ks requeridas son aquellas ks entre 1 y 400 inclusive que son relativamente primas a 400. Ahora bien, 400 = 24 52 .
Para contar las ks que no tienen factores primos en comn con 400, contaremos las que s tienen factores en comn con
400 y las restaremos a 400. Sea A el conjunto los mltiplos de 2 en {1, 2, 3, . . . , 400} y B el conjunto de mltiplos de 5 en
{1, 2, 3, . . . , 400}. Por inclusin- exclusin card (A B) = card (A) + card (B) card (A B). Ahora bien,

card (A) =

400
400
400
= 200, card (B) =
= 80, card (A B) =
= 40.
2
5
10

Luego card (A B) = 240 enteros en {1, 2, . . . 400} no son relativamente primos a 400 y 400 240 = 160 lo son. As pues, slo
160 ks provocan que todos los nios sean marcados.

40

Captulo 3

Sea n = pa11 pa22 pas s , donde las ps son primos distintos. Si (n) denota el nmero de enteros k, 1 k n relativamente
primos a n, entonces por inclusin-exclusin se puede demostrar que

(n) = (pa11 p1a1 1 )(pa22 p2a2 1 ) (pas s psas 1 ).


241 Teorema (Inclusin-exclusin en tres conjuntos)

card (A B C) = card (A) + card (B) + card (C)


card (A B) card (B C) card (C A)
+card (A B C)
Demostracin: Combinando la propiedad asociativa y distributiva,
card (A B C) =
=
=
=

=
=

card (A (B C))
card (A) + card (B C) card (A (B C))
card (A) + card (B C) card ((A B) (A C))
card (A) + card (B) + card (C) card (B C)
card (A B) card (A C)
+card ((A B) (A C))
card (A) + card (B) + card (C) card (B C)
(card (A B) + card (A C) card (A B C))
card (A) + card (B) + card (C)
card (A B) card (B C) card (C A)
+card (A B C) .

Esto demuestra el teorema. Vase tambin la figura 3.6.

242 Ejemplo De 200 polticos entrevistados en la legislatura, 75 usan cocana, 85 usan herona y 100 utilizan barbitricos.

Entre los 200, 30 usan cocana y herona, 50 usan herona y barbitricos y 40 utilizan cocana y barbitricos. Finalmente, 10
indulgen en el uso de las tres substancias. Cuntos de estos 200 polticos no usan drogas?
Resolucin: Sean A, B,C el conjunto de polticos entre los 200 que utilizan cocana, herona y barbitricos, respectivamente. Se nos es dado que card (A) = 75, card (B) = 85, card (C) = 100, card (A B) = 30, card (B C) = 50, card (C A) =
40, card (A B C) = 10. Por el principio de inclusin-exclusin
card (A B C) = 75 + 85 + 100 30 50 40 + 10 = 150
polticos utilizan al menos una droga. Luego, 200 150 = 50 no utilizan ninguna droga.
243 Ejemplo Cuntos nmeros entre 1 y 600 inclusive no son divisibles ni por 3, ni por 5, ni por 7?

Resolucin: Dentese por Ak aquellos enteros en el intervalo [1; 600] que son divisibles por k = 3, 5, 7. Entonces
600

3
600

5
600

7
600

15
600

21
600

35
600

105

card (A3 )

200,

card (A5 )

120,

card (A7 )

85,

card (A15 )

40

card (A21 )

28

card (A35 )

17

card (A105 ) =

Principio de inclusin-exclusin

41

Por inclusin-exclusin hay 200 + 120 + 85 40 28 17 + 5 = 325 enteros en [1; 600] divisibles por al menos uno de los
enteros en {3, 5, 7}. Los no divisibles por los enteros en {3, 57} son 600 325 = 275 en total.
C

sin 9

9550

14266

3
A

14266
14406

9550

9550

14266

sin 7

Figura 3.7: Ejemplo 244.

sin 8

Figura 3.8: Ejemplo 245.

244 Ejemplo En un grupo de 30 personas , 8 hablan ingls , 12 hablan castellano y 10 hablan francs. Se sabe que 5 hablan
ingls y castellano , 5 castellano y francs, y 7 ingls y francs. Tres personas hablan los tres idiomas. Cuntas personas no
hablan ninguno de estos idiomas?

Resolucin: Sea A el conjunto de los anglfonos, B el conjunto de los hispanfonos y C el conjunto de los francfonos.
Llenamos sucesivamente los diagramas de Venn en la figura 3.7. En la interseccin de los tres ponemos 8. En la regin comn
de A y B que no ha sido llenada ponemos 5 2 = 3. En la regin comn entre A y C que no ha sido llenada ponemos 5 3 = 2.
En la regin comn de B y C que no ha sido llenada ponemos 7 3 = 4. En la parte restante de A ponemos 8 2 3 2 = 1,
en la parte restante de B ponemos 12 4 3 2 = 3, y en la parte restante de C ponemos 10 2 3 4 = 1. Las regiones
disjuntas cuentan 1 + 2 + 3 + 4 + 1 + 2 + 3 = 16 personas. Fuera de estos tres crculos hay 30 16 = 14.
245 Ejemplo Considrese el conjunto de enteros naturales de cinco dgitos escritos en notacin decimal.
1. Cuntos hay?

7. Cuntos tienen exactamente cuatro 9s?

2. Cuntos no tienen un 9 en su expansin decimal?

8. Cuntos tienen exactamente cinco 9s?

3. Cuntos tienen al menos un 9 en su expansin decimal?

9. Cuntos no tienen ni un 8 ni un 9 en su expansin decimal?

4. Cuntos tienen exactamente un 9?


5. Cuntos tienen exactamente dos 9s?
6. Cuntos tienen exactamente tres 9s?

10. Cuntos no tienen ni un 7, ni un 8, ni un 9 en su expansin


decimal?
11. Cuntos tienen o bien un 7, o un 8, o un 9 en su expansin
decimal?

Resolucin:
1. Hay 9 selecciones posibles para el primer dgito y 10 selecciones posibles para los dgitos restantes. El nmero de selecciones es as 9 104 = 90000.
2. Hay 8 selecciones posibles para el primer dgito y 9 selecciones posibles para los dgitos restantes. El nmero de selecciones es as 8 94 = 52488.
3. Esto es la diferencia 90000 52488 = 37512.
4. Se condiciona en el primer dgito. Si el primer dgito es un 9
entonces los otros cuatro dgitos restantes debern de ser distintos de 9, lo que da 94 = 6561 nmeros. Si el primer dgito
no es un 9, entonces
hay 8 selecciones para este primer dgi4
to. Adems hay
= 4 maneras de seleccionar donde el 9
1

estar, y hay 93 maneras de llenar los 3 espacios restantes.


En este caso hay 8 4 93 = 23328 tales nmeros. En total hay
6561 + 23328 = 29889 enteros de cinco dgitos con exactamente un 9 en expansin decimal.
5. Se condiciona en el primer dgito. Si el primer dgito es un
9 uno de los cuatro restantes
ser un 9, y seleccin de po4
= 4 maneras. Los otros tres
sicin se puede lograr en
1
dgitos restantes debern de ser distintos de 9, lo que da
4 93 = 2916 tales nmeros. Si el primer dgito no es un 9,
entonces
hay 8 selecciones para este primer dgito. Adems
4
hay
= 6 maneras de seleccionar donde los otros dos 9s
2

42

Captulo 3
estarn, y hay 92 maneras de llenar los dos espacios restantes.
En este caso hay 8 6 92 = 3888 tales nmeros. En total hay
2916 + 3888 = 6804 enteros de cinco dgitos con exactamente
dos 9s en su expansin decimal.
6. De nuevo, condicionamos en el primer dgito. Si el primer dgito es un 9 entonces dos de los restantes cuatro
sern 9s, y la
4
seleccin de posicin se puede lograr en
= 6 maneras.
2
Los otros dos dgitos restantes debern de ser distintos de 9,
lo que da 6 92 = 486 tales nmeros. Si el primer dgito no
es un 9, entonces
hay 8 selecciones para este primer dgito.
4
Adems hay
= 4 maneras de seleccionar donde los tres
3
9s estarn, y se tiene 9 maneras de llenar los espacios restantes. En este caso hay 8 4 9 = 288 tales nmeros. En total hay
486 + 288 = 774 enteros de cinco dgitos con exactamente tres
9s en expansin decimal.
7. Si el primer dgito es un 9 entonces tres de los restantes cuatro
sern 9s, y la seleccin de posicin se puede lograr en
4
= 4 maneras. Los otros restantes dgitos debern de ser
3

distintos de 9, lo que da 4 9 = 36 tales nmeros. Si el primer


dgito no es un 9, entonces
hay 8 selecciones para este primer
4
= 4 maneras de seleccionar donde
dgito. Adems hay
4
los cuatro 9s estarn, as llenando todos estos espacios. En
este caso hay 8 1 = 8 tales nmeros. En total hay 36 + 8 = 44
enteros de cinco dgitos con exactamente tres 9s en expansin
decimal.
8. Obviamente existe solamente 1 entero tal.

Obsrvese que 37512 = 29889 + 6804 +


774 + 44 + 1.
9. Hay 7 selecciones para el primer dgito y 8 selecciones para
los restantes 4 dgitos, lo que da 7 84 = 28672 tales enteros.
10. Hay 6 selecciones para el primer dgito y 7 selecciones para
los restantes 4 dgitos, lo que da 6 74 = 14406 tales enteros.
11. Se utiliza inclusin-exclusin. De la figura 3.8, los nmeros
dentro de los crculos suma a 85854. Luego el nmero deseado es 90000 85854 = 4146.

246 Ejemplo

Cuntas soluciones enteras tiene la ecuacin


a + b + c + d = 100,
que satisfacen las siguientes restricciones:
1 a 10, b 0, c 2, 20 d 30?


Resolucin: Se utiliza inclusin-exclusin. Hay

80
= 82160 soluciones ntegras
3

a + b + c + d = 100, a 1, b 0, c 2, d 20.
Sea A el conjunto de soluciones
a 11, b 0, c 2, d 20
y B el conjunto de soluciones


Entonces card (A) =

a 1, b 0, c 2, d 31.

70
69
59
, card (B) =
, card (A B) =
y as
3
3
3


70
69
59
card (A B) =
+

= 74625.
3
3
3
El nmero total de soluciones
a + b + c + d = 100
con
1 a 10, b 0, c 2, 20 d 30
es as

Tarea

80
70
69
59

+
= 7535.
3
3
3
3

Tarea
247 Problema De cuntas maneras diferentes puede un estudiante adivinar un examen

43
mentos, demuestre que

completo de verdadero/falso que tenga dieciocho preguntas?

  
2n =

248 Problema En contando el nmero total de subconjuntos de un conjunto de n ele-

 

n
n
n
n
n
+
+
++
+
.
0
1
2
n1
n

Captulo

Sumas y recurrencias
4.1. Progresiones aritmticas
Consideremos el siguiente problema.
249 Ejemplo Si la sucesin de trminos 6, 10, 14, 18, 22, . . . sigue la misma ley de formacin, hallar los trminos 10mo , 50avo , 100avo .

Puede hallar el n-simo trmino?


Solucin: Observemos que cada trmino se obtiene sumndole 4 al trmino anterior. As:
10 = 6 + 4, 14 = 10 + 4, 18 = 14 + 4, 22 = 18 + 4, . . .
Pero an podemos ir ms lejos. Podemos expresar cada trmino en trminos del primero. Luego
6
10
14
18
22

=
=
=
=
=

6+04
6+14
6+24
6+34
6+44

primer termino
segundo termino
tercer termino
cuarto termino
quinto termino.

Si el patrn de formacin es respetado para los trminos subsiguientes entonces deberamos tener: dcimo trmino = 6 + 9 4 =
42, cincuentavo trmino = 6 + 49 4 = 202 y cienavo trmino = 6 + 99 4 = 402. De igual manera deducimos que el ensimo
trmino es = 6 + 4(n 1).
Una progresin como la del ejemplo previo, en donde la diferencia de trminos consecutivos es constante se llama progresin aritmtica. As
18, 12, 6, 0, 6, 12, . . .
es una progresin aritmtica de diferencia comn 6, mientras que 1, 3, 7, 97, . . . no lo es, ya que trminos sucesivos no guardan
diferencia constante.
En general, si comenzamos con un nmero arbitrario, digamos a y si guardamos una diferencia comn de d, entonces
obtenemos la progresin aritmtica a, a + d, a + 2d, a + 3d, . . . , con trmino en la posicin n igual a a + (n 1)d.
250 Ejemplo Hallar el 35avo trmino de una progresin aritmtica cuyo 27avo trmino es 186 y cuyo 45avo trmino es 312.

Solucin: Tratemos de expresar la data que sabemos en trminos del primer trmino y la diferencia comn. Como no se nos da
el primer trmino, llammosle a y a la diferencia comn llammosla d. As el primer trmino es a, el segundo a + d, el tercero
a + d + d = a + 2d, el cuarto a + 2d + d = a + 3d, etc. As el 27avo trmino debe ser a + 26d y el 45avo trmino debe ser a + 44d.
Pero la data del problema estipula que a + 26d = 186 y a + 44d = 312. De aqu (a + 44d) (a + 26d) = 312 186 = 126, i.e.,
44

Progresiones aritmticas

45

18d = 126 o d = 7. Pero entonces 186 = a + 26d = a + 26 7 = a + 182, de donde a = 4. Finalmente el 35avo trmino, o sea,
a + 34d est dado por a + 34d = 4 + 34(7) = 242.
Veremos ahora como sumar progresiones aritmticas.
251 Ejemplo Sumar la progresin aritmtica

7 + 15 + 23 + + 807.
Solucin: Vemos que los trminos estn en progresin aritmtica: 7, 7+ 81, 7+ 82, . . . , 7+ 8100. Observe que si S = 7+ 15+
23+ +807, entonces tambin S = 807+799+791+ +7. As: 2S = (7+807)+(15+799)+(23+791)+ +(807+7) =
101 814 = 82214. Finalmente, S = 41107.
252 Ejemplo Sumar 5/2, 1, 1/2, . . . hasta 19 trminos.

Solucin: La diferencia comn es 3/2. Luego el primer trmino es 5/2 = 5/2 + 0(3/2), el segundo 1 = 5/2 + 1(3/2), el
tercero 1/2 = 5/2 + 2(3/2), . . . , el diecinueveavo trmino 5/2 + 18(3/2) = 49/2. As, la suma que queremos es
S = 5/2 + 1 1/2 49/2.
Operando como en los ejemplos anteriores,
2S

= (5/2 49/2) + (1 46/2) + (1/2 43/2) + + (49/2 + 5/2)


= 44/2 44/2 44/2 44/2
= 19(44/2)
= 418.

Colegimos luego que S = 209.


La siguiente frmula ocurre con regularidad y el lector har bien en aprender a deducirla. Si
An = 1 + 2 + 3 + + n
entonces tambin
An = n + (n 1) + + 1.
Sumando estas dos cantidades,
=
=
=
=

An
An
2An

1
n
(n + 1)
n(n + 1),

+
2
+ (n 1)
+ (n + 1)

+
+
+

+
n
+
1
+ (n + 1)

puesto que hay n sumandos. De esto colegimos


1+2++n =

n(n + 1)
.
2

(4.1)

253 Ejemplo Hallar el valor de la suma

1 2 + 3 4 + 5 6 + + 99 100.
Solucin: Observe que 1 = 1 2 = 3 4 = = 99 100. Luego agrupando la suma en cincuenta pares que suman 1,
tenemos
1 2 + 3 4 + 5 6 + + 99 100 = 50.

254 Ejemplo Hallar la suma

12 22 + 32 42 + 52 62 + + 992 1002 .

46

Captulo 4

Solucin: Como x2 (x + 1)2 = 2x 1, tenemos


(12 22 ) + (32 42 ) + (52 62 ) + + (992 1002 ) = (3 + 7 + 11 + + 199) = 5050.

Tarea
255 Problema Hallar los trminos 14 y 110 de la progresin 43, 42, 41, . . ..

269 Problema (AHSME 1994) Sumar la serie

1
2
20 + 20 + 20 + + 40.
5
5

256 Problema Hallar los trminos 20 y 310 de la progresin 43, 40, 37, . . ..

270 Problema (AIME 1984) Hallar el valor de a2 + a4 + a6 + + a98 si a1 , a2 , a3 , . . .


es una progresin aritmtica con diferencia comn 1 y con a1 +a2 +a3 + +a98 = 137.

257 Problema Hallar los trminos 12 y 1090 de la progresin 0,6, 1,2, 1,8, . . ..

258 Problema Hallar los trminos 13 y 150 de la progresin a 2d, a d, a, . . ..

271 Problema Demostrar que

2
3
1995
1
+
+
++
1996 1996 1996
1996

259 Problema Hallar los trminos 10 y 51 de la progresin x y, x + y, x + 3y, . . ..

es un entero.

260 Problema Sumar 64, 96, 128, . . . hasta cuarenta trminos.

272 Problema (AHSME 1991) Sea Tn = 1 + 2 + 3 + + n y

261 Problema Sumar 1/2, 1/2 x, 1/2 2x, . . . hasta treinta trminos.

Pn =
262 Problema Sumar x y, x + y, x + 3y, . . . hasta diez trminos.

T2
T3
T4
Tn

.
T2 1 T3 1 T4 1
Tn 1

Hallar P1991 .
263 Problema Hallar el trmino 15 de una progresin aritmtica cuyo trmino 14 es 9

y cuyo trmino 16 es 90.

273 Problema Considere la tabla:

1=1
2+3+4 = 1+8

264 Problema Hallar el trmino 22 de una progresin aritmtica cuyo trmino 11 es

1 y cuyo trmino 16 es 55.

5 + 6 + 7 + 8 + 9 = 8 + 27
10 + 11 + 12 + 13 + 14 + 15 + 16 = 27 + 64

265 Problema Hallar el nmero de trminos y la diferencia comn de una serie aritm-

tica cuya suma es 30, cuyo primer trmino es 9 y cuyo ltimo trmino es 14.

Descubra el patrn de formacin, conjeture una ley general y demustrela.

274 Problema Los enteros naturales impares se agrupan en parntesis de la siguiente

266 Problema Sumar

1 + 2 + 3 + + 100.

manera:
(1)
(3, 5)

267 Problema Demostrar que

(7, 9, 11)
n2 (n2 + 1)
1 + 2 + 3 + + (n 1) + n =
.
2
2

(13, 15, 17, 19)

(21, 23, 25, 27, 29)


...............................

268 Problema Demostrar que

1 + 3 + 5 + + 2n 1 = n2 .

Halle la suma del sexto, sptimo, y octavo grupos. Conjeture y demuestre una frmula
para la suma del ensimo parntesis.

4.2. Progresiones geomtricas


Consideremos ahora la progresin 2, 6, 18, 54, 162, . . .. Notamos que cada trmino es el triple del anterior. Una progresin
de este tipo se llama progresin geomtrica. Decimos que 3 es la razn comn de esta progresin geomtrica. Vemos que el
primer trmino est dado por 2(3)0 , el segundo por 2(3)1 , el tercero por 2(3)2 , el cuarto por 2(3)3 , el quinto por 2(3)4 , etc. As
pues, el cincuentavo trmino est dado por 2(3)49 , el 100-avo es 2(3)99 y el ensimo trmino est dado por 2(3)n1 .
275 Ejemplo Hallar el dcimo trmino de la progresin geomtrica

1/2, 1, 2, . . . .

Progresiones geomtricas

47

1
1
Solucin: Vemos que la razn comn es 2. Luego el primer trmino est dado por (2)0 , el segundo est dado por (2)1 ,
2
2
1
1
1
el tercero por (2)2 , el cuarto por (2)3 , etc. El patrn indica pues que el dcimo trmino est dado por (2)9 .
2
2
2
En general si una progresin geomtrica tiene primer trmino a y razn comn r, entonces va de la siguiente manera:
a, ar, ar2 , ar3 , . . . y el ensimo trmino est dado por arn1 .
276 Ejemplo Hallar el segundo trmino de una progresin geomtrica con cuarto trmino 24 y sptimo trmino 192.

Solucin: No conocemos el primer trmino, llammosle a. Tampoco conocemos la razn comn, llammosle r. Luego el primer
trmino est dado por a, el segundo por ar, el tercero por ar2 , el cuarto por ar3 y siguiendo el patrn, el sptimo trmino es ar6 .
La data es que 24 = ar3 y 192 = ar6 . De aqu tenemos
r3 =

ar6 192
=
= 8,
ar3
24

de donde r = 2. Luego a(2)3 = 24 y as a = 3. Luego el segundo trmino est dado por ar = 6.


277 Ejemplo Hallar la suma de la serie geomtrica

1 + 2 + 4 + + 1024.
Solucin: Pongamos
S = 1 + 2 + 4 + + 1024.
Entonces
2S = 2 + 4 + 8 + + 1024 + 2048.
As
S = 2S S = (2 + 4 + 8 + 2048) (1 + 2 + 4 + + 1024) = 2048 1 = 2047.
278 Ejemplo Hallar la suma de la serie geomtrica

x=
Solucin: Tenemos

1 1
1
1
+ + + + 99 .
3 32 33
3

1
1
1
1
1
x = 2 + 3 + + 99 + 100 .
3
3
3
3
3

Luego
2
x
3

1
x x
3
1 1
1
1
= ( + 2 + 3 + + 99 )
3 3
3
3
1
1
1
1
( 2 + 3 + + 99 + 100 )
3
3
3
3
1
1

.
=
3 3100

De esto colegimos
x=

1
1

.
2 2 399

279 Ejemplo Sumar

a = 1 + 2 4 + 3 42 + + 10 49 .

48

Captulo 4

Solucin: Tenemos
4a = 4 + 2 42 + 3 43 + + 9 49 + 10 410 .
Luego 4a a nos da
3a = 1 4 42 43 49 + 10 410 .
Al sumar esta ltima serie geomtrica,
a=

10 410 410 1

.
3
9

280 Ejemplo Hallar la suma

Sn = 1 + 1/2 + 1/4 + + 1/2n .


Interpretar el resultado cuando n crece indefinidamente.
Solucin: Tenemos
1
Sn Sn = (1 + 1/2 + 1/4 + + 1/2n ) (1/2 + 1/4 + + 1/2n + 1/2n+1 ) = 1 1/2n .
2
As
Sn = 2 1/2n .
Calculamos ahora los siguientes valores:
S1
S2
S3
S4
S5
S6
S10

= 2 1/20
= 2 1/2
= 2 1/22
= 2 1/23
= 2 1/24
= 2 1/25
= 2 1/29

=
=
=
=
=
=
=

1
1,5
1,875
1,875
1,9375
1,96875
1,998046875

Vemos que a medida que tomamos ms trminos de la serie, nos acercamos cada vez ms a 2.
Supongamos que los procedimientos que utilizamos para sumar series geomtricas finitas son vlidos para las infinitas.
Entonces podremos decir que
S = 1 + 1/2 + 1/22 + 1/23 +
hasta infinitos suma a 2, ya que
1
S S = (1 + 1/2 + 1/22 + 1/23 + ) (1/2 + 1/22 + 1/23 + 1/24 + )) = 1
2
puesto que los trminos luego del primero encuentran su opuesto en la serie de la derecha.
La manipulacin formal que utilizamos supra debe manejarse con sumo cuidado. Por ejemplo,
S = 2 + 22 + 23 +
es obviamente infinitamente grande, pues cada trmino es mayor que 1 y esto incrementa el valor de la serie cada vez ms. Sin
embargo, al operar formalmente como lo hicimos anteriormente obtenemos
S = 2S S = (22 + 23 + 24 + ) (2 + 22 + 23 + ) = 2,
i.e., la suma de nmeros positivos da un resultado negativo, lo cual es un disparate maysculo.
Qu sucede entonces? Entra ahora pues el concepto de convergencia. Diremos que la serie geomtrica
a + ar + ar2 +
converge hacia un valor finito S si cada suma parcial
Sn = a + ar + ar2 + + arn1

Tarea

49

se acerca ms y ms a S a medida que n aumenta. Como vimos en los ejemplos anteriores, esto suceder cuando |r| < 1. Si las
sumas parciales no se acercan a un valor finito definitivo cuando n aumenta, entonces decimos que la serie geomtrica diverge.
Del ejemplo anterior se vislumbra que esto sucede cuando |r| 1.
La teora de series geomtricas convergentes puede utilizarse para convertir un decimal peridico a una fraccin.
281 Ejemplo Hallar la fraccin que representa

x = 0,122222222222222 . . .
Solucin: Observemos que
x=
Pero

1
2
2
2
+ 2 + 3 + 4 + .
10 10
10
10

2
2
2
1
2
+
+
+ =
= .
102 103 104
90 45

S=
As

0,1222222222 . . . =

1
1
11
+
= .
10 45 90

282 Ejemplo Hallar la fraccin que representa

x = 1,304040404040 . . . .
Solucin: Tenemos
x = 1+

30
40
40
40
+ 4 + 6 + 8 + .
2
10
10
10
10

Ahora bien, la serie geomtrica infinita


S=

40
40
40
+
+
+
104 106 108

suma a
S=
As
x = 1+

2
40
=
.
9900 495

30
2
1291
+
=
.
102 495
990

283 Ejemplo Una mosca est en el origen (0, 0) y viaja sobre el plano una pulgada hacia el este, 1/2 pulgada hacia el norte,

1/4 de pulgada hacia el oeste, 1/8 de pulgada hacia el sur, 1/16 de pulgada hacia el este, etc. Si la mosca hiciese esto ad
infinitum, donde terminara?
Solucin: Sea (X,Y ) el punto donde la mosca terminara. Vemos que
X = 1

1
1
4
1
+

+ =
4 16 64
5

y
Y=
Luego la mosca termina en el punto (4/5, 2/5).

Tarea

1 1
1
1
2
+

+ = .
2 8 32 128
5

50

Captulo 4

284 Problema Hallar los trminos 13, 22 y ensimo de la progresin geomtrica

4. Qu valor tendr
1+1+1+

3/2, 3/8, 3/32, . . .


hasta infinito? Discuta.
285 Problema Si el trmino 15 de una progresin geomtrica es 100 y el trmino 20

es 125, hallar el segundo trmino.


5. Qu valor tendr
11+11+

286 Problema Hallar la suma de las siguientes series geomtricas:

1.

hasta infinito?

1/2 1/4 + 1/8 + 1/512 1/1024.


2.

289 Problema Hallar

6 + 18 + 54 + . . .

hasta 20 trminos.
3.

k
.
2k

k=1

1/10 + 1/100 + 1/1000 + . . .


hasta 10 trminos.

290 Problema Hallar todos los nmeros complejos x, y tales que x, x+2y, 2x+y formen

una progresin aritmtica a la vez que (y + 1)2 , xy + 5, (x + 1)2 formen una progresin
geomtrica.

287 Problema Hallar la suma de la serie

3 + 5 4 + 7 42 + 9 43 + + 99 448 .
288 Problema

291 Problema Los lados de un tringulo rectngulo forman una progresin geomtrica.
Halle las tangentes de los ngulos agudos.

1. Qu valor tendr
1/2 + 1/4 + 1/8 +

292 Problema Sean a, b las races de la ecuacin x2 3x + A = 0 y sean c, d las races de la ecuacin x2 12x + B = 0. Se sabe que a, b, c, d forman, en este orden, una
progresin geomtrica creciente. Halle A y B.

hasta infinito?
2. Qu valor tendr
1/10 + 1/10 + 1/100 +
hasta infinito?

293 Problema Los nmeros a1 , a2 , . . . , an forman una progresin geomtrica. Si

3. Qu valor tendr
9/10 + 9/100 + 9/1000 +

s = a1 + a2 + + an ; T =

hasta infinito? Discuta por qu


1 = 0,9999999999999999999 . . .

1
1
1
+
++ ,
a1
a2
an

hallar su producto P = a1 a2 an en trminos de s y T.

4.3. Cancelacin telescpica


A veces podemos sumar una serie
a1 + a2 + a3 + + an

si podemos encontrar una sucesin {vk } con ak = vk vk1 . Entonces

a1 + a2 + a3 + + an = v1 v0 + v2 v1 + + vn1 vn2 + vn vn1 = vn v0 .


Si tal sucesin {vk } existe, diremos entonces que la serie a1 + a2 + + an es una serie telscopica.
294 Ejemplo Simplificar

1+
Solucin: Sumando cada fraccin tenemos:

1
1
1
1
1+
1+
1+
.
2
3
4
99
3 4 5
100

,
2 3 4
99

lo que simplifica a 100/2 = 50.


295 Ejemplo Dado que

es un entero, hllelo.

(log2 3) (log3 4) (log4 5) (log511 512)

Cancelacin telescpica

51

Solucin: Poniendo todo en una base comn, digamos a > 0, a 6= 1:


(log2 3) (log3 4) (log4 5) (log511 512) =

loga 3 loga 4 loga 5


loga 512 loga 512

=
.
loga 2 loga 3 loga 4
loga 511
loga 2

Pero
loga 512
= log2 512 = log2 29 = 9,
loga 2
de donde obtenemos el resultado.
296 Ejemplo Simplificar

99

(2 + 1) 22 + 1 22 + 1 22 + 1 22 + 1 .
Solucin: Utilizando la identidad x2 y2 = (x y)(x + y) y llamando P al producto:
(2 1)P

99

99

99

99

= (2 1) (2 + 1) 22 + 1 22 + 1 22 + 1 22 + 1

=
=

22 1 22 + 1 22 + 1 22 + 1 22 + 1
22 1 22 + 1 22 + 1 22 + 1
22 1 22 + 1 22 + 1 22 + 1

=
..
.
=
=

..
.
99
99
(22 1)(22 + 1)
100
22 1,

de donde
P = 22

100

1.

297 Ejemplo Simplificar

S = log tan 1 + log tan 2 + log tan 3 + + log tan 89 .


Solucin: Observe que (90 k) + k = 90 . Luego, sumando el trmino k-simo con el 90 k-simo obtenemos que la suma
dada es
S = log(tan 1 )(tan 89 ) + log(tan 2 )(tan 88 )
+ log(tan 3 )(tan 87 ) + + log(tan 44 )(tan 46 ) + log tan 45 .
Como tan k = 1/ tan(90 k) , tenemos
S = log 1 + log 1 + + log 1 + log tan 45 .
Finalmente, como tan 45 = 1, colegimos
S = log 1 + log 1 + + log 1 = 0.

298 Ejemplo Hallar el valor exacto del producto

P = cos

2
4
cos
cos .
7
7
7

52

Captulo 4

Solucin: Multiplicando a uno y otro lado por sin

y haciendo uso de la identidad sin 2x = 2 sin x cos x obtenemos


7

2
4
sin P = (sin cos ) cos
cos
7
7
7
7
7
2
2
4
1
(sin
cos ) cos
=
2
7
7
7
1
4
4
=
(sin
cos )
4
7
7
8
1
sin .
=
8
7
Como sin

8
= sin , deducimos que
7
7

1
P= .
8

299 Ejemplo Demostrar que

1 3 5
9999
1

<
.
2 4 6
10000 100
Solucin: Pongamos
A=

9999
1 3 5

2 4 6
10000

B=

2 4 6
10000

.
3 5 7
10001

Es claro que x2 1 < x2 para todo nmero real x. De esto deducimos


x1
x
<
.
x
x+1
Por tanto

<
<
<
..
.

1/2
3/4
5/6
..
.
9999/10000

2/3
4/5
6/7
..
.

< 10000/10001

Como todas estas desigualdades son de nmeros positivos, podemos multiplicar una y otra columna para obtener
9999
2 4 6
10000
1 3 5

<
,
2 4 6
10000 3 5 7
10001
o A < B. Luego A2 = A A < A B. Pero entrelazando los factores de A y B,
1 2 3 4 5 6 7
9999 10000
1

=
,
2 3 4 5 6 7 8
10000 10001 10001

y por consiguiente, A2 < A B = 1/10001. De aqu A < 1/ 10001 < 1/100.


Para nuestro siguiente ejemplo necesitaremos la siguiente definicin. El smbolo n! (lase n factorial) significa
AB =

n! = 1 2 3 n.
Por ejemplo 1! = 1, 2! = 1 2 = 2, 3! = 1 2 3 = 6, 4! = 1 2 3 4 = 24. Observe que (k + 1)! = (k + 1)k!. Tendremos por
convencin 0! = 1.
300 Ejemplo Sumar

1 1! + 2 2! + 3 3! + + 99 99!.

Cancelacin telescpica

53

Solucin: De (k + 1)! = (k + 1)k! = k k! + k! deducimos (k + 1)! k! = k k!. As


1 1!
2 2!
3 3!
..
.

=
=
=
..
.

98 98
99 99!

= 99! 98!
= 100! 99!

2! 1!
3! 2!
4! 3!
..
.

Sumando una y otra columna,


1 1! + 2 2! + 3 3! + + 99 99! = 100! 1! = 100! 1.
301 Ejemplo Hallar una forma cerrada para la suma

An = 1 + 2 + + n.
Solucin: Observemos que
k2 (k 1)2 = 2k 1.
Luego
12 02
22 12
32 22
..
.

=
=
=
..
.

211
221
231
..
.

n2 (n 1)2

2n1

Sumando una y otra columna,


n2 02 = 2(1 + 2 + 3 + + n) n.
Resolviendo para la suma,
1 + 2 + 3 + + n = n2 /2 + n/2 =

n(n + 1)
.
2

302 Ejemplo Hallar la suma

12 + 22 + 32 + + n2 .
Solucin: Observemos que
k3 (k 1)3 = 3k2 3k + 1.
Luego
13 03
23 13
33 23
..
.

=
=
=
..
.

3 12 3 1 + 1
3 22 3 2 + 1
3 32 3 3 + 1
..
.

n3 (n 1)3

3 n2 3 n + 1

Sumando una y otra columna,


n3 03 = 3(12 + 22 + 32 + + n2 ) 3(1 + 2 + 3 + + n) + n.
Pero por el ejercicio anterior se tiene
n3 03 = 3(12 + 22 + 32 + + n2 )

3
n(n + 1) + n.
2

54

Captulo 4

Resolviendo para la suma,


12 + 22 + 32 + + n2 =

n3 1
n
+ n(n + 1) .
3 2
3

Simplificando obtenemos
12 + 22 + 32 + + n2 =

n(n + 1)(2n + 1)
.
6

303 Ejemplo Sumar la serie

1
1
1
1
+
+
++
.
12 23 34
99 100
Solucin: Observe que
1
1
1
=
.
k(k + 1) k k + 1
Luego

Sumando una y otra columna obtenemos

1
12
1
23
1
34
..
.
1
99 100

=
=
=
..
.
=

1 1

1 2
1 1

2 3
1 1

3 4
..
.
1
1

99 100

1
1
1
1
1
99
+
+
++
= 1
=
.
12 23 34
99 100
100 100

304 Ejemplo Sumar

1
1
1
1
+
+
++
.
1 4 4 7 7 10
31 34
Solucin: Observe que
1
1
1
1
1
=

.
(3n + 1) (3n + 4) 3 3n + 1 3 3n + 4

Luego

Sumando una y otra columna obtenemos

1
14
1
47
1
7 10
1
10 13
..
.
1
34 37

=
=
=
=
..
.
=

1
1

3 12
1
1

12 21
1
1

21 30
1
1

30 39
..
.
1
1

102 111

1
1
1
1
1
12
1
+
+
++
=
= .
1 4 4 7 7 10
31 34 3 111 37

Tarea

55

305 Ejemplo Sumar

1
1
1
1
+
+
++
.
1 4 7 4 7 10 7 10 13
25 28 31
Solucin: Observe que
1
1
1
1
1
=

.
(3n + 1) (3n + 4) (3n + 7) 6 (3n + 1)(3n + 4) 6 (3n + 4)(3n + 7)
Luego

Sumando una y otra columna obtenemos

1
147
1
4 7 10
1
7 10 13
..
.
1
25 28 31

=
=
=
..
.
=

1
1

614 647
1
1

6 4 7 6 7 10
1
1

6 7 10 6 10 13
..
.
1
1

6 25 28 6 28 31

1
1
1
1
1
9
1
+
+
++
=

=
.
1 4 7 4 7 10 7 10 13
25 28 31 6 1 4 6 28 31 217
306 Ejemplo Sumar

1 2 + 2 3 + 3 4 + + 99 100.
Solucin: Observemos que
1
1
k(k + 1) = (k)(k + 1)(k + 2) (k 1)(k)(k + 1).
3
3
Por lo tanto
12

23

34
..
.

=
..
.

99 100

1
1
123 012
3
3
1
1
234 123
3
3
1
1
345 234
3
3
..
.
1
1
99 100 101 98 99 100
3
3

Sumando una y otra columna,


1 2 + 2 3 + 3 4 + + 99 100 =

1
1
99 100 101 0 1 2 = 333300.
3
3

Tarea
307 Problema Hallar una frmula para

Dn = 1 2 + 3 4 + + (1)n1 n.

309 Problema Si

a + ib = 1 + 2i + 3i2 + 4i3 + + 1995i1994 + 1996i1995 ,


con a y b nmeros reales, hallar a y b.

308 Problema Simplificar

(1 + i)2004
.
(1 i)2000

56

Captulo 4

310 Problema Simplificar

22

32

42

311 Problema Simplificar

1023
X

log2 1 +

1
k

1
992

319 Problema Demostrar que

csc 2 + csc 4 + csc 8 + + csc 2n = cot 1 cot 2n .


320 Problema Sea 0 < x < 1. Demostrar que

k=2

x2

1 x2n+1

x
.
1x

n=1

312 Problema Hallar el valor exacto de

1
1
1
1
+
+
++
.
log2 1996! log3 1996! log4 1996!
log1996 1996!

321 Problema Demostrar que

313 Problema (AHSME 1996) La sucesin

tan

1, 2, 1, 2, 2, 1, 2, 2, 2, 1, 2, 2, 2, 2, 1, 2, 2, 2, 2, 2, 1, 2, . . .

+ 2 tan 99 + 22 tan 98 + + 298 tan 2 = cot 100 .


2100
2
2
2
22

consiste de 1s separados por bloques de 2s, con n bloques de 2s en el n-simo bloque.


Hallar la suma de los primeros 1234 trminos de esta sucesin.

322 Problema Demostrar que

314 Problema (AIME 1985) Calcular el producto x1 x2 x8 si x1 = 97 y xn =

n
X

n/xn1 , n > 1.

1
k
n2 + n
= 2
.
2 n +n+1
k4 + k2 + 1

k=1

315 Problema (AIME 1993) Durante una campaa poltica reciente, un candidato hizo

una trayectoria que presumimos yace en el plano. En el primer da l viaj hacia el este,
en el segundo, l viaj hacia el norte, en el tercero hacia el oeste, en el cuarto hacia el
sur, en el quinto hacia el este, etc. Si el candidato viaj n2 /2 millas en el n-simo da, a
cuantas millas estaba l de su punto de partida en el 40avo da?

323 Problema Evale el radical

316 Problema Demostrar que

13 + 23 + 33 + + n3 =

n(n + 1) 2
2

arctan
3

1/3
.

324 Problema Demostrar que

317 Problema Simplificar


3

1 2 4 + 2 4 8 + 3 6 12 +
1 3 9 + 2 6 18 + 3 9 27 +

2 1 3 1 4 1
100 1

.
23 + 1 33 + 1 43 + 1
1003 + 1

= .
4
1 + n + n2

n=0

325 Problema Demostrar que

318 Problema Sean a1 , a2 , . . . , an nmeros arbitrarios. Demostrar que

a1 + a2 (1 + a1 ) + a3 (1 + a1 )(1 + a2 ) + a4 (1 + a1 )(1 + a2 )(1 + a3 )


+ + an1 (1 + a1 )(1 + a2 )(1 + a3 ) (1 + an2 )
= (1 + a1 )(1 + a2 )(1 + a3 ) (1 + an ) 1.

1
1
1
1
1998 < 1 + + + + +
< 1999.
3
1000000
2
4

4.4. Recursiones y ecuaciones funcionales


Veremos ahora mtodos para hallar formas cerradas de ciertas recursiones. En general aplicaremos las tcnicas de la seccin
anterior.
326 Ejemplo Sea x0 = 7 y xn = 2xn1 , n 1. Hallar una frmula cerrada para xn .

Solucin: Tenemos
x0
x1
x2
x3
..
.

=
=
=
=
..
.

7
2x0
2x1
2x2
..
.

xn

2xn1

Multiplicando una y otra columna,


x0 x1 xn = 7 2n x1 x2 xn1 .

Recursiones y ecuaciones funcionales

57

Cancelando factores comunes,


xn = 7 2n .

327 Ejemplo Sea x0 = 7 y xn = xn1 + n, n 1. Hallar una frmula cerrada para xn .

Solucin: Tenemos
x0
x1
x2
x3
..
.

=
=
=
=
..
.

7
x0 + 1
x1 + 2
x2 + 3
..
.

xn

xn1 + n

Sumando una y otra columna,


x0 + x1 + x2 + + xn = 7 + x0 + x2 + + xn1 + (1 + 2 + 3 + + n).
Cancelando y simplificando,
xn = 7 +

n(n + 1)
.
2

328 Ejemplo Sea x0 = 7 y xn = 2xn1 + 1, n 1. Hallar una frmula cerrada para xn .

Solucin: Tenemos
x0
x1
x2
x3
..
.

=
=
=
=
..
.

7
2x0 + 1
2x1 + 1
2x2 + 1
..
.

xn1
xn

=
=

2xn2 + 1
2xn1 + 1

Aqu no nos funcionan los mtodos anteriores, as que nos valdremos del siguiente artificio. Multipliquemos a la k-sima fila
por 2nk , obteniendo
2n x0
= 2n 7
n1
2 x1 = 2n x0 + 2n1
2n2 x2 = 2n1 x1 + 2n2
2n3 x3 = 2n2 x2 + 2n3
.. ..
..
. .
.
22 xn2
2xn1
xn

=
=
=

23 xn3 + 22
22 xn2 + 2
2xn1 + 1

Sumando una y otra columna y cancelando,


xn = 7 2n + (1 + 2 + 22 + + 2n1 ) = 7 2n + 2n 1 = 2n+3 1.
Aliter: Pongamos un = xn + 1 = 2xn1 + 2 = 2(xn1 + 1) = 2un1 . Luego la recursin un = 2un1 la resolvemos como en
nuestro primer ejemplo, obteniendo de esta forma un = 2n u0 = 2n (x0 + 1) = 2n 8 = 2n+3 . Finalmente xn = un 1 = 2n+3 1.
329 Ejemplo Una sucesin satisface u0 = 3, u2n+1 = un , n 1. Halle una forma cerrada para ella.

58

Captulo 4
1/2

Solucin: Pongamos vn = log un . Entonces vn = log un = log un1 =


n

o sea, log un = (log u0 )/2n . Luego un = 31/2 .

1
vn1
log un1 =
. Como vn = vn1 /2, tenemos vn = v0 /2n ,
2
2

330 Ejemplo Halle una forma cerrada para

a0 = 5, a j+1 = a2j + 2a j , j 0.
Solucin: Tenemos
a j+1 + 1 = a2j + 2a j + 1 = (a j + 1)2 .
j

Pongamos v j+1 = a j+1 + 1. Entonces v j+1 = a j+1 + 1 = (a j + 1)2 = v2j . De aqu v j+1 = v20 , o sea
j

a j+1 = v j+1 1 = v20 1 = (a0 + 1)2 1 = 62 1.

331 Ejemplo Una escalera tiene n escalones. Un duende puede subir la escalera de escaln en escaln o saltndose un escaln.

Hallar una recursin para el nmero de maneras en que el duende puede subir la escalera.
Solucin: Sea un el nmero de maneras en que puede el duende subir una escalera de n escalones. El duende puede llegar al
ltimo escaln o bien desde el penltimo o bien desde el antepenltimo escaln. Luego
un = un1 + un2 .
Es claro que u1 = 1, u2 = 2.
332 Ejemplo Si f (x) = f0 (x) =

1
y fn (x) = f ( fn1 (x)) para n > 0, hallar f1996 (1/3).
1x

Solucin: Observe que


f1 (x) = f ( f0 (x)) =

1
x1
=
,
1
x
1 1x

f2 (x) = f ( f1 (x)) =

1
=x
1 x1
x

y
f3 (x) =

1
= f0 (x).
1x

Luego esta recursin es cclica de orden 3. Esto implica que


f0 (x) = f3 (x) = f6 (x) = ,
f1 (x) = f4 (x) = f7 (x) =
y
f2 (x) = f5 (x) = f8 (x) = .
Como 1996 = 1995 + 1 deja residuo 1 al ser dividido por 3,
f1996 (1/3) = f1 (1/3) = 4.

333 Ejemplo Hallar todas las funciones que satisfacen

f (x + y) + f (x y) = 4x2 + 4y2 .

Tarea

59

Solucin: Tomando y = 0, obtenemos f (x) + f (x) = 4x2 o f (x) = 2x2 . Veamos que f (x) = 2x2 satisface la ecuacin funcional:
f (x + y) + f (x y) = 2(x + y)2 + 2(x y)2 = 2x2 + 4xy + 2y2 + 2x2 4xy + 2y2 = 4x2 + 4y2 .

334 Ejemplo (AHSME 1979) La funcin f satisface

f (x + y) = f (x) + f (y) xy + 1
para todos los nmeros reales x, y. Si f (1) = 1, hallar todos los enteros n 6= 1 tales que f (n) = n.
335 Ejemplo (AHSME 1981) La funcin f no est definida para x = 0, pero si x 6= 0 satisface

f (x) + 2 f

1
x

= 3x.

Para cuntos valores de x se cumple f (x) = f (x)?

Tarea
336 Problema Sea x1 = 1, xn+1 = xn2 xn + 1, n > 0. Demostrar que

341 Problema Demostrar que existe una funcin f nica del conjunto R+ de los reales
positivos a R+ tal que

1
= 1.
xn

f ( f (x)) = 6x f (x), f (x) > 0 x > 0.

n=1

342 Problema Si u0 = 1/3 y un+1 = 2u2n 1, hallar una frmula para un .


337 Problema (AIME 1994) La funcin f tiene la propiedad que para todo nmero real

x,
f (x) + f (x 1) = x2 .

343 Problema Una sucesin a1 , a2 , . . . satisface a1 = 2 y

am+n = 4mn am an m, n.

Si f (19) = 94, hallar el residuo cuando f (94) se divide por 1000.

Hallar el valor mnimo de n para el cual an tiene al menos 3000 dgitos.


338 Problema Hallar una forma cerrada para
344 Problema Sea k un entero no-negativo fijo y supngase que

x0 = 1; xn = xn1 + n2 , n > 0.

339 Problema Sea f (x) = f0 (x) =

f (2x) = 2k1
1 x2 , fn (x) = f ( fn1 (x)), n > 0. Hallar

f99 (4).

340 Problema Sea f una funcin con las siguientes propiedades:

1)
2)
3)
4)
5)

f (n) est definida para todo entero positivo n;


f (n) es un entero;
f (2) = 2;
f (mn) = f (m) f (n) para todo m y n;
f (m) > f (n) si m > n.
Demostrar que f (n) = n.

1
f (x) + f (x + ) .
2

Demustrese que
f (3x) = 3k1

2
1
f (x) + f (x + ) + f (x + ) .
3
3

345 Problema Hallar todas las funciones f que satisfagan

f (x)2 f

1x
1+x

= 64x.

Captulo

Polinomios y ecuaciones
5.1. Ecuaciones
A continuacin veremos como resolver algunas ecuaciones. La mayora de ellas son ecuaciones cuadrticas disfrazadas.
Para resolver ecuaciones cuadrticas, el mtodo ms eficiente es quizs la complecin del cuadrado. sta es preferible a la
frmula cuadrtica ya que crea malicia para identificar patrones. Por ejemplo, para resolver
x2 6x + 3 = 0,
escribimos

x2 6x + 3

= x2 6x + 9
6 2
= (x 3)2
(

6)
= (x 3 + 6)(x 3 6).

De aqu x = 3 6. De manera semejante, para resolver 2x2 + 6x + 5 = 0 escribimos


2x2 + 6x + 5

9 1
2x2 + 6x + +
2 2

1
3
= ( 2x + )2 (i )2
2
2

3
3
1
1
= ( 2x + i )( 2x + + i ).
2
2
2
2
=

1
3
Luego, x = i .
2
2
346 Ejemplo Resolver

9 + x4 = 10x2 .
Solucin: Observe que
x4 10x2 + 9 = (x2 9)(x2 1).
1
Luego x = y x = 1.
3
347 Ejemplo Resolver

9x 3x+1 4 = 0.
Solucin: Observe que 9x 3x+1 4 = (3x 4)(3x + 1). Como no existe ningn nmero real con 3x + 1 = 0, este factor se
descarta. As 3x 4 = 0 nos da x = log3 4.
60

Ecuaciones

61

348 Ejemplo Resolver

(x 5)(x 7)(x + 6)(x + 4) = 504.


Solucin: Reordenemos los factores y multipliquemos para obtener
(x 5)(x 7)(x + 6)(x + 4) = (x 5)(x + 4)(x 7)(x + 6) = (x2 x 20)(x2 x 42).
Pongamos y = x2 x. As (y 20)(y 42) = 504 o y2 62y + 336 = (y 6)(y 56) = 0. Luego y = 6, 56, lo que implica
x2 x = 6
y
x2 x = 56.
De aqu x = 2, 4, 7, 8.
349 Ejemplo Resolver

12x4 56x3 + 89x2 56x + 12 = 0.


Solucin: Reordenando
12x4 + 12 56(x3 + x) + 89x2 = 0.

(5.1)

Dividiendo por x ,
1
1
) 56(x + ) + 89 = 0.
2
x
x
Pongamos u = x + 1/x. Luego u2 2 = x2 + 1/x2 . Usando esto, (1) se convierte 12(u2 2) 56u + 89 = 0, de donde u =
5/2, 13/6. Por lo tanto
1 5
x+ =
x 2
y
1 13
x+ = .
x
6
Concluimos que x = 1/2, 2, 2/3, 3/2.
12(x2 +

350 Ejemplo Hallar las soluciones reales de

x2 5x + 2
Solucin: Observe que
2

x2 5x + 3 + 2

x2 5x + 3 15 = 0.

1/2

+ 5)(u1/2 3) = 0. Luego u = 9 (descartamos u1/2 + 5 = 0, por

1/2

Poniendo u = x 5x + 3 obtenemos u + 2u 15 = (u
qu?). Por lo tanto x2 5x + 3 = 9 o x = 1, 6.
351 Ejemplo Resolver

x2 5x + 3 = 12.

3x2 4x + 34

3x2 4x 11 = 9.

(5.2)

(3x2 4x + 34) (3x2 4x 11) = 45.

(5.3)

Solucin: Tenemos idnticamente


Dividiendo cada miembro de (3) por los miembros correspondientes de (2) obtenemos
p

3x2 4x + 34 +

Sumando (2) y (4)


5
de donde x = , 3.
3

3x2 4x 11 = 5.

3x2 4x + 34 = 7,

(5.4)

62

Captulo 5

352 Ejemplo Resolver la ecuacin

Solucin: Pngase u =

3
14 + x + 3 14 x = 4.

3
14 + x, v = 3 14 x. Entonces
64 = (u + v)3 = u3 + v3 + 3uv(u + v) = 14 + x + 14 x + 12(196 x2 )1/3 ,

de donde
3 = (196 x2 )1/3 ,
que al resolver nos da x = 13.
353 Ejemplo Halle el valor exacto de cos 2 /5.

Solucin: Usando la identidad


par de veces obtenemos

cos(u v) = cos u cos v sin u sin v

cos 2 = 2 cos2 1

(5.5)

cos 3 = 4 cos3 3 cos .

(5.6)

Pongamos x = cos 2 /5. Como cos 6 /5 = cos 4 /5, gracias a las dos identidades (5) y (6), vemos que x satisface la ecuacin
4x3 2x2 3x + 1 = 0,
o sea,
(x 1)(4x2 + 2x 1) = 0.
Como x = cos 2 /5 6= 1, y cos 2 /5 > 0, x es la raz positiva de la ecuacin cuadrtica 4x2 + 2x 1 = 0, es decir

51
2
cos
=
.
5
4

354 Ejemplo Cuntas soluciones reales tiene la ecuacin

sin x =

x
?
100

Solucin: Vemos que x = 0 es una solucin. Adems si x > 0 es una solucin, x < 0 lo es tambin. As pues, slo contaremos
las soluciones positivas.
Para que x sea una solucin, se debe tener |x| = 100| sin x| 100. Por lo tanto podemos restrinjir x al intervalo (0, 100].
Dividamos este intervalo en subintervalos de longitud 2 (con un ltimo intervalo ms corto):
(0, 100] = (0, 2 ] (2 , 4 ] (4 , 6 ] (28 , 30 ] (30 , 100].

De las grficas de y = sin x, y = x/100 vemos que en el intervalo (0, 2 ] existe slo una solucin. En cada intervalo de la forma
(2 k, 2(k + 1) ], k = 1, 2, . . . , 14 existen dos soluciones. El intervalo (30 , 100] tiene una onda completa de longitud (ya que
31 < 100) en la cual hay dos soluciones. Por consiguiente existen 1 + 2 14 + 2 = 31 As pues, hay 31 soluciones positivas y
por ende 31 soluciones negativas. Como 0 es tambin una solucin, el total de soluciones reales es por lo tanto 31 + 31 + 1 = 63.

355 Ejemplo Resolver el sistema de ecuaciones

x+y+u
y+u+v
u+v+x
v+x+y

= 4,
= 5,
= 0,
= 8.

Tarea

63

Solucin: Sumando las cuatro ecuaciones y diviendo por 3,


x + y + u + v = 3.
Esto implica
4+v
5 + x
0+y
8 + u

=
=
=
=

3,
3,
3,
3.

De aqu x = 2, y = 3, u = 5, v = 7.
356 Ejemplo Resolver el sistema de ecuaciones

(x + y)(x + z) = 30,
(y + z)(y + x) = 15,
(z + x)(z + y) = 18.
Solucin: Pongamos u = y + z, v = z + x, w = x + y. Entonces el sistema se convierte en
vw = 30, wu = 15, uv = 18.

(5.7)

Multiplicando todas estas ecuaciones obtenemos u2 v2 w2 = 8100, esto es, uvw = 90. Combinando este resultado con cada una
de estas ecuaciones en (7), obtenemos u = 3, v = 6, w = 5, o u = 3, v = 6, w = 5. Luego
y+z
z+x
x+y

=
=
=

3,
y+z
6, o z + x
5,
x+y

= 3,
= 6,
= 5,

de donde x = 4, y = 1, z = 2 o x = 4, y = 1, z = 2..

Tarea
357 Problema Resolver

363 Problema Resolver la ecuacin

x
+3
a

a
b 6a
= + .
x
a
b

358 Problema Resuelva

log1/3

cos x +




5
5
+ log1/3 cos x
= 2.
6
6

364 Problema Cuntas soluciones reales tiene la ecuacin

sin x = ln x?

(x 7)(x 3)(x + 5)(x + 1) = 1680.


365 Problema Resolver la ecuacin

359 Problema Resuelva

x4 + x3 4x2 + x + 1 = 0.

|x + 1| |x| + 3|x 1| 2|x 2| = x + 2.

360 Problema Resolver la ecuacin


366 Problema Hallar las races reales de

|x 3|(x

2 8x+15)/(x2)

= 1.

361 Problema Resolver la ecuacin

= 3log9 4 .

362 Problema Resolver la ecuacin

x+34 x1+
x + 8 6 x 1 = 1.

367 Problema Resolver la ecuacin

0,5 logx (x2 x)

sin2 x

cos2 x

+52

= 7.

6x4 25x3 + 12x2 + 25x + 6 = 0.


368 Problema Una progresin geomtrica de nmeros reales satisface que la suma de
sus primeros cuatro trminos es 15 y la suma de los cuadrados de estos trminos es 85.
Hallar esta progresin.

64

Captulo 5

369 Problema Resolver la ecuacin

379 Problema Resolver el sistema

2x + y + z + u = 1

x(2x + 1)(x 2)(2x 3) = 63.

x + 2y + z + u = 12
370 Problema Hallar el valor de

x + y + 2z + u = 5

30 31 32 33 + 1.
371 Problema Si la ecuacin

xx

x.

x + y + z + 2u = 1

..

380 Problema Resolver el sistema de ecuaciones

=2
x2 + x + y = 8,

hiciere sentido, hallar el valor de x.

y2 + 2xy + z = 168,
372 Problema Si la ecuacin

z2 + 2yz + 2xz = 12480.

x+

x+

x+ = 2

381 Problema Hallar las races reales de la ecuacin

hiciere sentido, hallar el valor de x.

373 Problema Resolver la ecuacin

q
x+2

p
x++2

x+2

{z

x + 2 3x = x.

n radicales

x + x2 1 x x2 1
+
= 98.

2
x x 1 x + x2 1

382 Problema Resolver la ecuacin

1
374 Problema Sean a,b, c constantes reales con abc 6= 0. Resolver el sistema de ecua-

1+

ciones

= x.

1
1

1+

x2 (y z)2 = a2 ,

1+

y2 (z x)2 = b2 ,

.
.
.

1+ 1x

Aqu la expresin de la fraccin se repite n veces.

z2 (x y)2 = c2 .

383 Problema Resolver el sistema

375 Problema Resolver el sistema

x+2+y+3+

log2 x + log4 y + log4 z = 2,

(x + 2)(y + 3) = 39,

(x + 2)2 + (y + 3)2 + (x + 2)(y + 3) = 741.

log3 x + log9 y + log9 z = 2,


log4 x + log16 y + log16 z = 2.

384 Problema Resolver el sistema de ecuaciones


376 Problema Resuelva el sistema

x4 + y4 = 82,

x3 + 3x2 y + y3 = 8,
3

x y = 2.

2x 2x y + xy = 1.
385 Problema Resolver el sistema de ecuaciones
377 Problema Encuentre una solucin real para la ecuacin
2

10

(x 9x 1) + 99x

10

x1 x2 = 1, x2 x3 = 2, . . . , x100 x101 = 100, x101 x1 = 101.

= 10x (x 1).
386 Problema Resuelva para x

378 Problema Resolver el sistema de ecuaciones

x + x + 11 +
x + x 11 = 4.

x2 yz = 3,
387 Problema Dos estudiantes trataron de resolver la ecuacin cuadrtica x2 +bx+c =

y2 zx = 4,

0. Maguer ambos estudiantes ejecutaron todos los pasos correctamente, el primero copi
mal el coeficiente b y obtuvo las soluciones x = 6, 1. El segundo copi mal c y obtuvo
las soluciones x = 2, 3. Cules son las soluciones correctas?

z xy = 5.

5.2. Polinomios
Recordemos que un polinomio es una expresin de la forma
p(x) = a0 + a1 x + a2 x2 + + an xn .

Polinomios

65

Aqu los coeficientes ak de p(x) pueden ser cualquier nmero complejo. Si las ak s pertenecen exclusivamente al conjunto de
los nmeros enteros diremos que p(x) Z[x]. Si las ak s son nmeros reales entonces escribiremos p(x) R[x]. Finalmente,
escribiremos p(x) C[x] si las ak s son nmeros complejos.
388 Ejemplo Hallar la suma de todos los coeficientes obtenidos luego de expandir y simplificar el producto

(1 x2 + x4 )109 (2 6x + 5x9 )1996 .


Solucin: Pongamos
p(x) = (1 x2 + x4 )109 (2 6x + 5x9 )1996 .
Vemos que p(x) un polinomio de grado 4 109 + 9 1996 = 18400. As pues, p(x) es tambin la expresin
p(x) = a0 + a1 x + a2 x2 + + a18400 x18400 .
Vemos entonces que la suma de los coeficientes de p(x) es
p(1) = a0 + a1 + a2 + + a18400 ,
que tambin es p(1) = (1 12 + 14 )109 (2 6 + 5)1996 = 1. As pues, la suma deseada es igual a 1.
389 Ejemplo Pngase

(1 + x4 + x8 )100 = a0 + a1 x + a2 x2 + + a800 x800 .


Hallar:
(A) a0 + a1 + a2 + a3 + + a800 .
(B) a0 + a2 + a4 + a6 + + a800 .
(C) a1 + a3 + a5 + a7 + + a799 .
(D) a0 + a4 + a8 + a12 + + a800 .
(E) a1 + a5 + a9 + a13 + + a797 .
Solucin: Pongamos
p(x) = (1 + x4 + x8 )100 = a0 + a1 x + a2 x2 + + a800 x800 .
Entonces
(A)
a0 + a1 + a2 + a3 + + a800 = p(1) = 3100 .
(B)
a0 + a2 + a4 + a6 + + a800 =

p(1) + p(1)
= 3100 .
2

(C)
a1 + a3 + a5 + a7 + + a799 =

p(1) p(1)
= 0.
2

(D)
a0 + a4 + a8 + a12 + + a800 =

p(1) + p(1) + p(i) + p(i)


= 2 3100 .
4

(E)
p(1) p(1) ip(i) + ip(i)
= 0.
4
Otra propiedad de los polinomios que es a menudo til es el algoritmo de divisin: si dividimos p(x) por a(x) obtendremos
polinomios q(x), r(x) con
p(x) = a(x)q(x) + r(x).
a1 + a5 + a9 + a13 + + a797 =

Aqu 0 grado r(x) < grado a(x). Por ejemplo, al dividir x5 + x4 + 1 por x2 + 1 obtenemos
x5 + x4 + 1 = (x3 + x2 x 1)(x2 + 1) + x + 2,
de donde el cociente es q(x) = x3 + x2 x 1 y el residuo es r(x) = x + 2.

66

Captulo 5

390 Ejemplo Hallar el residuo cuando (x + 3)5 + (x + 2)8 + (5x + 9)1997 se divide por x + 2.

Solucin: Como estamos dividiendo por un polinomio de grado 1, el residuo es un polinomio de grado 0, es decir, una constante.
As pues, existe un polinomio q(x) y una constante r con
(x + 3)5 + (x + 2)8 + (5x + 9)1997 = q(x)(x + 2) + r
Si ponemos x = 2 obtenemos
0 = (2 + 3)5 + (2 + 2)8 + (5(2) + 9)1997 = q(2)(2 + 2) + r = r,
de donde el residuo es r = 0.
391 Ejemplo Un polinomio deja residuo 2 cuando se divide por x 1 y residuo 4 cuando se divide por x + 2. Hallar el
residuo cuando este polinomio se divide por x2 + x 2.

Solucin: De la informacin dada existen polinomios q1 (x), q2 (x) con p(x) = q1 (x)(x 1) 2 y p(x) = q2 (x)(x + 2) 4. Luego
p(1) = 2 y p(2) = 4. Ahora bien, como x2 + x 2 = (x 1)(x + 2) es un polinomio de grado 2, el residuo r(x) al dividir
p(x) por x2 + x 1 es de grado 1 o menor, es decir r(x) = ax + b para constantes a, b que debemos determinar. Por el algoritmo
de divisin
p(x) = q(x)(x2 + x 1) + ax + b.
Luego
2 = p(1) = a + b
y
4 = p(2) = 2a + b.
De estas ecuaciones vemos que a = 2/3, b = 8/3. Luego el residuo es r(x) = 2x/3 8/3.
392 Ejemplo Sea f (x) = x4 + x3 + x2 + x + 1. Hallar el residuo cuando f (x5 ) se divide por f (x).

Solucin: Observe que f (x)(x 1) = x5 1 y


f (x5 ) = x20 + x15 + x10 + x5 + 1 = (x20 1) + (x15 1) + (x10 1) + (x5 1) + 5.
Cada sumando en parntesis es divisible por x5 1 y por ende por f (x). Luego el residuo es 5.
El algoritmo de divisin nos ayuda a demostrar el siguiente resultado, a menudo conocido como el Teorema del factor.
393 Teorema Teorema del factor El polinomio p(x) es divisible por x a si y slo si p(a) = 0.

Demostracin Como x a es un polinomio de grado 1, el residuo al dividir p(x) por x a es un polinomio de grado 0, es decir,
una constante. As
p(x) = q(x)(x a) + r.
De aqu p(a) = q(a)(a a) + r = r. El teorema se deduce de esto.
394 Ejemplo Si p(x) un polinomio cbico con p(1) = 1, p(2) = 2, p(3) = 3, p(4) = 5. Hallar p(6).

Solucin: Pongamos g(x) = p(x) x. Entonces g(x) es un polinomio de grado 3 y g(1) = g(2) = g(3) = 0. Luego g(x) = c(x
1)(x2)(x3) para alguna constante c que debemos determinar. Pero g(4) = c(41)(42)(43) = 6c y g(4) = p(4)4 = 1,
de donde c = 1/6. Finalmente
(6 1)(6 2)(6 3)
+ 6 = 16.
p(6) = g(6) + 6 =
6

Polinomios

67

395 Ejemplo El polinomio p(x) tiene coeficientes enteros y p(x) = 7 para cuatro valores enteros diferentes de x. Demostrar

que p(x) 6= 14 para ningn entero x.


Solucin: El polinomio g(x) = p(x) 7 se anula para cuatro enteros diferentes a, b, c, d. Luego, por el Teorema del factor,
g(x) = (x a)(x b)(x c)(x d)q(x),
para algn polinomio q(x) con coeficientes enteros. Supongamos que p(t) = 14 para algn entero t. Entonces g(t) = p(t) 7 =
14 7 = 7. De aqu
7 = g(t) = (t a)(t b)(t c)(t d)q(t),
esto es, hemos factorizado a 7 como el producto de al menos cuatro factores enteros distintos, lo que es imposible, pues 7 es a
lo sumo 7(1)1 el producto de tres enteros distintos. De esta contradiccin colegimos que no existe tal entero t.
396 Ejemplo Hallar un polinomio cbico p(x) que se anule cuando x = 1, 2, 3 y que satisfaga p(4) = 666.

Solucin: El polinomio debe tener la forma p(x) = a(x 1)(x 2)(x 3), donde a es una constante. Como 666 = p(4) =
a(4 1)(4 2)(4 3) = 6a, a = 111. Luego el polinomio deseado es p(x) = 111(x 1)(x 2)(x 3).
397 Ejemplo Hallar un polinomio cbico p(x) con p(1) = 1, p(2) = 2, p(3) = 3, p(4) = 5.

Solucin: Utilizaremos el siguiente mtodo debido a Lagrange. Sea


p(x) = a(x) + 2b(x) + 3c(x) + 5d(x),
donde a(x), b(x), c(x), d(x) son polinomios cbicos con las siguientes propiedades: a(1) = 1 y a(x) se anula para x = 2, 3, 4;
b(2) = 1 y b(x) se anula cuando x = 1, 3, 4; c(3) = 1 y c(3) = 1 se anula para x = 1, 2, 4 y d(4) = 1, d(x) anulndose cuando
x = 1, 2, 3.
Utilizando el mtodo del problema anterior hallamos
a(x) =

b(x) =

(x 2)(x 3)(x 4)
,
6

(x 1)(x 3)(x 4)
,
2

c(x) =

(x 1)(x 2)(x 4)
2

y
d(x) =

(x 1)(x 2)(x 3)
.
6

As
1
p(x) = (x 2)(x 3)(x 4) + (x 1)(x 3)(x 4)
6
3
5
(x 1)(x 2)(x 4) + (x 1)(x 2)(x 3).
2
6
El lector podr verificar que este polinomio cumple con las condiciones estipuladas.
Por ltimo discutiremos las frmulas de Vite y las identidades de Newton-Girard. Para introducir el tpico consideremos
primero el siguiente ejemplo.
398 Ejemplo Hallar el producto

(x + 1)(x 2)(x + 4)(x 5)(x + 6).

68

Captulo 5

Solucin: Vemos que el producto es un polinomio de grado 5. Para obtener el coeficiente de x5 tomamos una x de cada binomio.
As pues el coeficiente de x5 es 1. Para formar el trmino de x4 tomamos una x de cuatro de los binomios y una constante del
binomio restante. As pues, el coeficiente de x4 es
1 2 + 4 5 + 6 = 4.
Para formar el trmino de x3 tomamos tres x de tres de los binomios y dos constantes de los dos binomios restantes. As el
coeficiente de x3 es
(1)(2) + (1)(4) + (1)(5) + (1)(6) + (2)(4) + (2)(5) + (2)(6)
+(4)(5) + (4)(6) + (5)(6) = 33.
De manera semejante, el coeficiente de x2 es
(1)(2)(4) + (1)(2)(5) + (1)(2)(6) + (1)(4)(5) + (1)(4)(6) + (2)(4)(5)
+(2)(4)(6) + (4)(5)(6) = 134
y el coeficiente de x es
(1)(2)(4)(5) + (1)(2)(4)(6) + (1)(2)(5)(6) + (1)(4)(5)(6) + (2)(4)(5)(6) = 172.
Finalmente, el trmino constante es (1)(2)(4)(5)(6) = 240. El producto pedido es entonces
x5 + 4x4 33x3 134x2 + 172x + 240.
Del ejemplo anterior vemos que cada trmino tiene un peso de 5, pues de cada uno de los cinco binomios o bien tomamos
el trmino de x o bien tomamos la constante.
Si a0 6= 0 y
a0 xn + a1 xn1 + a2 xn2 + + an1 x + an
es un polinomio con races 1 , 2 , . . . , n entonces podemos escribir

a0 xn + a1 xn1 + a2 xn2 + + an1 x + an = a0 (x 1 )(x 2 )(x 3 ) (x n1 )(x n ).


De esto deducimos las frmulas de Vite:

a1 X
=
k ,
a0
n

k=1

a2
=
a0

a3
=
a0

a4
=
a0

j k ,

1 j<kn

j k l ,

1 j<k<ln

j k l s ,

1 j<k<l<sn

..........
..........
...........
n an

(1)

a0

= 1 2 n .

399 Ejemplo Hallar la suma de las races, la suma de las races tomadas de dos en dos, la suma de los cuadrados de las races

y la suma de los recprocos de las races de la ecuacin


2x3 x + 2 = 0.

Polinomios

69

Solucin: Sean a, b, c las races de 2x3 x + 2 = 0. Por las frmulas de Vite la suma de las races es
0
a+b+c = = 0
2
y la suma de las races tomadas de dos en dos es
ab + ac + bc =

1
.
2

Para hallar a2 + b2 + c2 recurrimos a la siguiente identidad


a2 + b2 + c2 = (a + b + c)2 2(ab + ac + bc).
Luego
a2 + b2 + c2 = 02 2(1/2) = 1.
Finalmente, como abc = 2/2 = 1, vemos que
1 1 1 ab + ac + bc 1/2
+ + =
=
= 1/2.
a b c
abc
1

400 Ejemplo Sean , , las races de x3 x2 + 1 = 0. Hallar

1
1
1
+
+ .
2 2 2
Solucin: De x3 x2 + 1 = 0 deducimos 1/x2 = 1 x. Luego
1
1
1
+ 2 + 2 = (1 ) + (1 ) + (1 ) = 3 ( + + ) = 3 1 = 2.
2

Conjunto con las frmulas de Vite tenemos las identidades de Newton-Girard para las sumas de potencias sk = 1k + 2k +
+ nk de las races:
a0 s1 + a1 = 0,
a0 s2 + a1 s1 + 2a2 = 0,
a0 s3 + a1 s2 + a2 s1 + 3a3 = 0,
etc..
401 Ejemplo Si a, b , c son las races de x3 x2 + 2 = 0, hallar

a2 + b2 + c2
a3 + b3 + c3
y
a4 + b4 + c4 .
Solucin: Primero observamos que
a2 + b2 + c2 = (a + b + c)2 2(ab + ac + bc) = 12 2(0) = 1.
Como x3 = x2 2, obtenemos
a3 + b3 + c3 = a2 2 + b2 2 + c2 2 = a2 + b2 + c2 6 = 1 6 = 5.
Finalmente, de x3 = x2 2 obtenemos x4 = x3 2x, de donde
a4 + b4 + c4 = a3 2a + b3 2b + c3 2c = a3 + b3 + c3 2(a + b + c) = 5 2(1) = 7.

70

Captulo 5

402 Ejemplo (USAMO 1973) Determine todas las soluciones, reales o complejas del sistema de ecuaciones

x + y + z = 3,
x2 + y2 + z2 = 3,
x3 + y3 + z3 = 3.
Solucin: Sean x, y, z las races del polinomio
p(t) = (t x)(t y)(t z) = t 3 (x + y + z)t 2 + (xy + yz + zx)t xyz.
Ahora bien, xy + yz + zx = (x + y + z)2 /2 (x2 + y2 + z2 )/2 = 9/2 3/2 = 3 y de
x3 + y3 + z3 3xyz = (x + y + z)(x2 + y2 + z2 xy yz zx)
se desprende que xyz = 1. Luego
p(t) = t 3 3t 2 + 3t 1 = (t 1)3 .
Luego x = y = z = 1 es la nica solucin del sistema anterior.

Tarea
403 Problema Sea

410 Problema Suponga que


2 n

2n

xn + a1 xn1 + a2 xn2 + + an = (x + r1 )(x + r2 ) (x + rn )

(1 + x + x ) = a0 + a1 x + + a2n x .

donde r1 , r2 , . . . , rn son nmeros reales. Demuestre que

Hallar
a0 + a2 + a4 + + a2n .

404 Problema Demostrar que las tres races de x3 1 = 0 son = 1/2 + i 3/2, 2 =

3
2
1/2 i 3/2 y = 1. Demostrar que + + 1 = 0.

(n 1)a21 2na2 .
411 Problema Si 1 , 2 , . . . , 100 son las races de

x100 10x + 10 = 0,

405 Problema Sea

(1 + x2 + x4 )100 = a0 + a1 x + + a400 x400 .

hallar la suma

100
1100 + 2100 + + 100
.

Hallar
a0 + a3 + a6 + + a399 .

412 Problema Hallar un polinomio p(x) de grado 4 con p(1) = 1, p(2) =


2, p(3) = 4, p(4) = 5, p(5) = 8.

406 Problema El polinomio p(x) satisface p(x) = p(x). Cuando p(x) es dividido

por x 3 el residuo es 6. Cul es el residuo cuando p(x) es dividido por x2 9?

413 Problema Sean , , las races de x3 x 1 = 0. Halle

1
1
1
+ 3+ 3
3

407 Problema La ecuacin x4 16x3 + 94x2 + px + q = 0 tiene dos races dobles. Ha-

llar p + q.
y
408 Problema (USAMO 1984) El producto de dos de las races de

x4 18x3 + kx2 + 200x 1984 = 0

414 Problema Los nmeros reales , safisfacen

3 3 2 + 5 17 = 0,

es 32. Determine el valor de k.

3 3 2 + 5 + 11 = 0.

409 Problema Si p(x) es un polinomio de grado n tal que p(k) = 1/k, k = 1, 2, . . . , n +

1, hallar el valor de p(n + 2).

5 + 5 + 5.

Demuestre que + = 2.

Captulo

Geometra
6.1. Razones de reas de tringulos
415 Definicin Dado un ABC, denotaremos las longitudes de sus lados por a = BC, b = CA, c = AB. Las longitudes de sus

tres medianas las denotaremos por ma , mb , mc . Las longitudes de sus tres alturas las denotaremos por ha , hb , hc . Por ltimo,
las longitudes de sus tres bisectores angulares las denotaremos por ta , tb , tc . El rea de ABC se denotar por [ABC] y su
semipermetro por p.

Vemos que
[ABC] =

aha bhb chc


=
=
.
2
2
2

El concepto de rea es uno de los ms intuitivos pero a su vez, uno de los ms poderosos en la resolucin de problemas de
concurso. Comenzaremos pues con el siguiente resultado.
416 Teorema Si las rectas AB y PQ se intersecan en M entonces

PM
[ABP]
=
.
[ABQ] QM
Demostracin: Se observan cuatro casos, como en las figuras 6.1 a 6.4. Sin prdida de generalidad presmase
que las reas involucradas no son degeneradas. As
[ABP] [AMP] [AMQ]
AB PM AM
PM
[ABP]
=

=
.
[ABQ] [AMP] [AMQ] [ABQ]
AM QM AB
QM

bM

Q
Figura 6.1: Teorema 416.

Figura 6.2: Teorema 416.

Figura 6.3: Teorema 416.

71

Figura 6.4: Teorema 416.

72

Captulo 6

417 Ejemplo Sea P un punto en el interior del ABC. Las semirrectas AP, BP, CP intersecan a los lados BC, CA, AB en los

puntos A , B , C respectivamente, como en la figura 6.5. Demostrar que

PA PB PC
+
+
= 1.
AA BB CC
Resolucin: Se tiene
1=

[ABP] + [BCP] + [CAP] [ABP] [BCP] [CAP] PC PA PB


=
+
+
=
+
+
,
[ABC]
[ABC] [ABC] [ABC] CC AA BB

gracias al teorema 416.


b

K
b

Figura 6.5: Ejemplo 417.

bQ
b

B
b
F

Figura 6.6: Ejemplo 418.

bM

Figura 6.7: Ejemplo 419.

418 Ejemplo Tres lneas rectas, cada una de ellas paralelas a los lados del ABC, concurren en el punto M (figura 6.6). Si las
reas de los
2 resultantes dentro del ABC son [EKM] = R, [MQF] = S y [PMN] = T , demustrese que
trestringulos
[ABC] = ( R + S + T ) .

Resolucin: Obsrvese que EKM MQF PMN. Entonces


EM 2
S
MF 2
T
PN 2
R
=
,
=
,
=
.
2
2
[ABC]
AC
[ABC]
AC
[ABC]
AC2
Luego

R
AC, MF =
[ABC]

EM =

S
AC, PN =
[ABC]

T
AC.
[ABC]

A causa del paralelismo de rectas, EM = AP, y MF = NC. Esto conlleva a


EM + PN + MF = AP + PN + NC = AC.
De esta ltima igualdad se desprende que

R
AC +
[ABC]

S
AC +
[ABC]

T
AC = AC,
[ABC]

de donde se colige, al cancelar AC y resolver para [ABC] que

[ABC] = ( R + S + T )2 .
419 Ejemplo En el ABC, A , B , C son puntos en BC, CA, AB respectivamente tales que

BA CB 1
AC
= = = .

C B AC BA 3
K, L, M son las intersecciones de las rectas AA y CC ; BB y AA ; CC y BB , respectivamente. Si el rea del ABC es 1,
encuentre el rea del KLM.

bA
b

Tarea

73

Resolucin: Observe que


1 = [ABC] = [ABL] + [LBC] + [ALC].
Si [ABL] = s, por el teorema 416, se tiene [LBC] =
s+

s
y [ALC] = 3s. As,
3

3
s
+ 3s = 1 = s = .
3
13

Con razonamientos semejantes se obtiene [BCM] = [CAK] =

3
. Luego
13

[KLM] = [ABC] [ABL] [BCM] [CAK] =

4
.
13

420 Ejemplo ABCD es un cuadriltero convexo tal que DA y CB se intersecan en K; AB y DC se intersecan en L; AC y KL se

intersecan en G; DB y KL se intersecan en F. Demostrar que


KG
KF
=
.
FL
GL
Resolucin: Aplicaciones sucesivas del teorema 416 producen
KF
[DBK] [DBK] [KBL] CD AK [ACD] [ACK] [ACK] KG
=
=

=
=
.
FL
[DBL]
[KBL] [DBL] CL AD
[ACL] [ACD]
[ACL]
GL
b

D
b

b b

G
L

Figura 6.8: Ejemplo 420

Tarea
421 Problema (AIME 1992) Los puntos A , B ,C estn en los lados BC, CA y AB res-

pectivamente, del ABC. Dado que AA , BB y CC , concurren en O y que la suma


BO
CO
AO
+
+
= 92,
OA
OB
OC

422 Problema El ABC tiene lados que 13, 14 y 15 unidades. El A BC est dentro
del ABC con lados paralelos al ABC y a 2 unidades de distancia de los lados de ste.
Hallar el rea del A BC .

hllese el producto
AO BO CO

.
OA OB OC

6.2. Teoremas de Ceva y de Menelao


423 Definicin Una recta que pasa por el vrtice de un tringulo se llama ceviana de este vrtice. La ceviana es propia si no

coincide con un lado del tringulo.

74

Captulo 6

Adoptaremos la convencin de marcar la interseccin de la ceviana con el lado opuesto a su vrtice con una
prima , as AA , BB , CC son cevianas. Vase la figura 6.9.

424 Teorema (Teorema de Ceva) Las tres cevianas AA , BB y CC de un ABC concurren si y slo si

AC BA CB

= +1.
C B AC B A
Demostracin: = Usando el teorema 416 se obtiene


AC BA CB [APC] [APB] [PCB]

= 1.
=
C B AC BA
[PCB] [APC] [ABP]

Presmase que AA y BB se intersecan en P. nase CP y extindase hasta AB, intersecndola en C . Como


AA , BB y CC son concurrentes, por la mitad del teorema ya demostrada se tiene
AC BA CB

= +1.
C B AC B A
Pero por hiptesis

AC BA CB

= +1.
C B AC B A

Esto significa que

AC
AC
=
,
C B C B

de donde se colige que C = C .


C

b
b

M
b

Figura 6.10: Cevianas.

Figura 6.9: Cevianas.

Figura 6.11: Puntos menelaicos.

Como el teorema 416 no depende de la posicin de los puntos involucrados, la demostracin anterior es
vlida aun cuando el punto P yace fuera del tringulo ABC.
425 Definicin A un punto P que yaga sobre la recta determinada por un lado del ABC se le llama punto menelaico1 de este

lado. Si el punto no es un vrtice del tringulo entonces es un punto menelaico propio.

426 Teorema (Teorema de Menelao) Sean L, M, N puntos menelaicos de los lados BC, CA y AB del ABC. L, M, N son

colineales si y solamente si

BL CM AN

= 1.
LC MA NB
Demostracin: = Sean X,Y puntos arbitrarios de la recta LMN. Entonces


BL CM AN [BXY ] [CXY ] [AXY ]

= 1.

=

LC MA NB

Que no puntos melenudos.

[CXY ] [AXY ] [BXY ]

Teoremas de Ceva y de Menelao

75

Presmase ahora que la recta MN corta a AC en L . Por la mitad del teorema ya demostrada se tiene
BL CM AN

= 1.
LC MA NB
Pero por hiptesis
BL CM AN

= 1.
LC MA NB
As

BL
BL
=
,

L C LC

de donde L = L .
427 Ejemplo (IMO 1982) Las diagonales AC y CE de un hexgono regular ABCDEF son dividas interiormente en los puntos

M y N, respectivamente, de tal manera que


AM CN
=
= r.
AC
CE
Determnese r si B, M, N son colineales.
Resolucin: nase BE intersecando AC en P. Aplcando el teorema de Menelao al CPE y a la recta BMN, se tiene
1r
CM
;
=
MP r 21

PB = AB cos ABP =

Adems

de donde se colige que

AB BE
PB 1
=
=
= .
2
4
BE
4

1r
EN
=
,
NC
r

3
2 2r 1 1 r

= 1 = r =
.
2r 1 4
r
3
B

M
D

P
b

E
F
428 Teorema [Invariancia bajo perspectiva] Sean L1 y L2 dos rectas distintas sobre el plano. Si A, B, C, D son puntos distintos
sobre L1 y si A , B , C , D son puntos distintos sobre L2 y si las rectas AA , BB , CC , DD son concurrentes, entonces
AC BD AC B D
=
.
CB DA C B D A
Demostracin: Si AA , BB , CC , DD se intersecan en O y si P es el punto de interseccin de L1 y L2 (si L1 ||L2 ,
entonces P es el punto en infinito). Vase la figure 6.12.
Aplicando el teorema de Menelao a ABA , A PA, B PB, BPB intersecndose con las rectas CC , DD , CC ,
y DD respectivamente,
AC PC A O

= 1,
CP c A OA

76

Captulo 6
A D PD AO

= 1,
D P DA OA
BD PD B O

= 1.
DP D B OB
En multiplicando estas cuatro igualdades se colige
AC A D BC BD
= 1,
C A DA CB D B
de donde

AC BD AC B D
=
.
CB DA C B D A

O
b

C
b

b
b

bD

C D

Figura 6.12: Teorema 428.

b
b

Figura 6.13: Teorema 429.

Figura 6.14: Ejemplo ??.

429 Teorema [Teorema de Simson] Si P es un punto en el circuncrculo del ABC. Sean D, E, F los pies de las perpendiculares

de P a BC, CA y AB respectivamente. Entonces C, E, F son colineares.

Demostracin: Para demostrar que D, E, F son colineares, se necesita demostrar que


AF BD CE

= 1.
FB DC EA
Ntese que AF = PA cos PAF, FB = PB cos PBF, BD = PB cos PBD, DC = PC cos PCD, CE = PC cos PCE,
EA = PA cos PAE. Por lo tanto,
(cos PAC)(cos PBD)(cos PCE)
AF BD CE

=
.
FB DC EA (cos PBF)(cos PCD)(cos PAE)
Ahora bien, PAF = PCD, PBD = PAE, PCE = PBF, de donde se consigue el resultado.
430 Ejemplo Refirase a la figura . D, E, F son, respectivamente, los pies de las perpendiculares de A a BC, B a CA y de C a
AB. Trcense las rectas perpendiculares de D a AB, CA, BE, CF y sean P, Q, M, N los respectivos pies de las perpendiculares.
Demustrese que P, Q, M, N son colineares.

Demostracin: El cuadriltero BDHF es cclico y la recta de Simson de D pasa por P, M, N. En otras palabras
P, M, N son colineares. Arguyendo de manera semejante se demuestra que Q, M, N son colineares.

Puntos y rectas notables de un tringulo

77

431 Ejemplo Sea H el ortocentro, O el circuncentro y R el circunradio del ABC. Sea D la reflexin de A a travs de la recta

BC, E la reflexin de B a travs de CA y F aquella de C a travs de AB. Demustrese que D, E y F son colineares si y slo si
OH = 2R.
Demostracin: Sea PQR el tringulo cuyo tringulo de medianas es ABC (esto es A es el punto medio
de QR, B el de RP y C el de PQ). Desde O, trcense rectas perpendiculares a QR, RP y PQ, cuyos pies de las
perpendiculares son D , E , F respectivamente. Por el teorema de Simson, D , E , F son colineares si y slo si O
yace en el circuncrculo del PQR. Ntese que el circuncentro del PQR is el ortocentro del ABC, esto es, H.
As, O yace en el circuncrculo del PQR si y slo si OH es igual al PQR que es 2R.

432 Teorema (Teorema de Ptolomeo) Dados cuatro puntos cualesquiera en posicin general (esto es, ninguno de ellos coin-

cide, y no tres de ellos son colineares),


AB CD + AD BC AC BD.
La igualdad se cumple si y slo si el cuadriltero ABCD es cclico.
Demostracin: Sean L, M, N, respectivamente, los pies de las perpendiculares de D a BC, CA, AB. Como
CLD = CMD = 90 , los puntos L, C, D, M son cocclicos. La figura muestra uno de los casos posibles. De
todas maneras se tiene
CD AB
LM = CD sin BCA =
,
2R
en donde R denota el circunradio del ABC. De la misma guisa,
MN =

AD BC
,
2R

LN =

BD AC
.
2R

En virtud de la desigualdad del tringulo, LM + MN LN. Utilizando las expresiones para LM, MN y LN se
obtiene
CD AB AD BC BD AC
+

.
2R
2R
2R
Esto demuestra la desigualdad deseada. La igualdad es satisfecha si y solamente si L, M, N son colineares y
gracias al teorema de Simson esto sucede si y solamente si D yace en el circuncrculo del ABC.

6.3. Puntos y rectas notables de un tringulo


433 Ejemplo Dos crculos se intersecan en A y B. El punto P viaja alrededor de uno de los crculos. Las rectas PA y PB se

extienden de tal manera que corten al otro crculo en C y D, respectivamente. Demustrese que la longitud de la cuerda CD es
independiente de P.
Resolucin: Es suficiente demostrar que CAD es constante, como en la figura 6.15. Pero esto se desprende de
CAD = APB + ADB
y de que estos ltimos dos ngulos son constantes.

434 Ejemplo Tres crculos congruentes pasan por un punto comn P, intersecndose en los puntos P, U, V y W como en la

figura 6.16. Demustrese que P es el ortocentro del UVW .

78

Captulo 6

b
b
b

D
Figura 6.15: Ejemplo 433.

Resolucin: Sean A, B,C los centros de los crculos, vase la figura 6.17. Note que AVCP y BWCP son rombos y por tanto,
paralelogramos. Luego AV ||BW y as, AVW B es un paralelogramo.
Adems PU es una cuerda comn de los crculos con centro en A y B y as, perpendicular a AB. Luego pues, lo es tambin
a VW . De manera semejante se demuestra que PV UW y PW UV , de donde se obtiene el resultado.
D
b

P
b
b

P
b
b

B
bW

F
b

Figura 6.16: Ejemplo 434.

Figura 6.17: Ejemplo 434.

Figura 6.18: Ejemplo 435.

435 Ejemplo Sean A, B,C puntos colineales. Constryanse crculos con dimetros en AB, AC y BC. Sea D un punto en el arco
tal que AD AC y sea EF la tangente comn a los arcos AB
y BC.
Demustrese que BEFD es un rectngulo. (Figura 6.18.)
AC

Resolucin: Reljese la condicin AC BD y permtase a D recorrer el semicrculo AC. Sea Sea E la interseccin de AD
con el semicrculo AB, y sea F la interseccin de DC con el semicrculo BC. Sean M y N los puntos medios de AB y BC
respectivamente. Como ADC = AE B = BF C = 90 , BE DF es un rectngulo. Se tiene
E BD = BE F

F BD = BF E .

El ME B es issceles, as MBE = ME B. De igual manera, NBF es issceles, as NBF = NF B. Por lo tanto


MBD = ME F y NBD = NF E . When MBD = ME F = 90 , ME F = NF E = 90 , o en otras palabras,
E F es tangente a ambos semicrculos AB y BC, y E = E , F = F . Se concluye que BEFD es un rectngulo.
436 Ejemplo Se construye, exteriormente, cuadrados en los lados AB y BC del tringulo ABC, siendo P y Q los centros
de los respectivos cuadrados, como en la figura 6.19. Sea M el punto medio del lado AB. Demustrese que PM = QM y que
PM QM.

Resolucin: Sean U y V las esquinas de los cuadrados opuestas a A y B, respectivamente. Ntese que una rotacin de 90
en torno a C toma al ACV al UCB. Luego PV y UB son iguales y perpendiculares. En PUB vemos que we see PM es
la mitad de larga y paralela a UB. De igual manera, MQ es la mitad de larga y paralela a PV . Por lo tanto, PM y MQ son
iguales y perpendiculares.

bU

Tarea

79

C
b

b
b

Figura 6.19: Ejemplo 436.

Tarea
437 Problema Se construye, exteriormente, cuadrados en cada lado del cuadrilte-

ro ABCD siendo P, Q, R, S los centros de los respectivos cuadrados. Demustrese que

PR = QS y que PR QS.

Qb


Apendice

Indicaciones y respuestas
8 Presmase que AC BC y colquese D en el segmento de recta AC de tal manera que AD = BD. Luego ADB es issceles en D y se
tendr A = B, contradiccin.
9 Si

a entonces 2 , lo que implica que (1 ) 0, desigualdad imposible al ser 0 < < 1.

10 Se tiene 1

1
< < 1. En cuadrando,
102000
1

Ya que

1
1
+
< 0, se tiene
102000 104000
1

11 Se tiene

2
1
+
< 2.
102000 104000

1
1
1
1
< 1 2000 2000 + 4000 < 2 .
102000
10
10
10

x4 + 2x2 + 2x + 2

=
=

(x2 + ax + b)(x2 + cx + d)
x4 + (a + c)x3 + (d + b + ac)x2 + (ad + bc)x + bd.

As
bd = 2, ad + bc = 2, d + b + bc = 2, a + c = 2.
Presmase que a, b, c, d son ntegros. Entonces como bd = 2, b y d deben ser de paridad opuesta, el uno par y el otro non. Luego d + b sera
non, y como d + b + bc = 2, bc debera ser non, lo que hace tanto a b como a c nones, de donde d es par. As pues ad es par y ad + bc = 2 no
puede ser, ya que ad es par y bc non, contradiccin.
20 Decompngase el conjunto en los n pares
{1, 2}, {2, 3}, . . . , {2n 1, 2n}.
Como siempre se tomarn dos enteros consecutivos, estos sern relativamente primos.
21 Cada entero es de la forma 2a m en donde a 0 es entero y m es impar. Como solamente hay n enteros impares en el conjunto {1, 2, . . . , 2n},
de los n + 1 enteros tomados, dos tendrn la misma parte impar, digamos 2a m y 2b m. Luego si a < b se tendr que 2a m dividir a 2b m. Se
dar otra solucin en el problema 47
22 Hay n residuos posibles diferentes cuando se divide a un entero por n, as entre n + 1 enteros diferentes habr dos dejando el mismo
residuo al dividirse por n. Su diferencia ser divisible por n.
23 20
24 Divdase al cuadrado en 4 subcuadrados congruentes, con lados paralelos al cuadrado
original. Dos de los cinco puntos caern en un

subcuadrado. Pero como la distancia mxima en un subcuadrado es su diagonal de 2/2 unidades de longitud, el resultado se cumple.
38 Hay 27 sumas distintas. Las sumas 1 y 27 se obtienen de manera nica (en 100 y 999). Cada una de las otras 25 sumas aparece al menos
tres veces. Luego si se sacan 27 + 25 + 1 = 53 boletas, al menos 3 tendrn la misma suma.

80

Indicaciones y respuestas

81

45 Sea P(n) la aseveracin: (1 + 2)2n + (1 2)2n es par y (1 + 2)2n (1 2)2n = b 2 para algn b N. Si n = 1, entonces vemos
que

(1 + 2)2 + (1 2)2 = 6,
que es par y que

(1 + 2)2 (1 2)2 = 4 2.

As P(1) cierta es. Presmase que P(n 1) es cierta para n > 1, i.e., presmase que

(1 + 2)2(n1) + (1 2)2(n1) = 2N
para algn entero N y que

(1 + 2)2(n1) (1 2)2(n1) = a 2

para algn entero positivo a.


Considrese ahora

(1 + 2)2n + (1 2)2n = (1 + 2)2 (1 + 2)2n2 + (1 2)2 (1 2)2n2 .

Esto es

(3 + 2 2)(1 + 2)2n2 + (3 2 2)(1 2)2n2 .

En utilizando P(n 1), esto simplifica a


12N + 2 2a 2 = 2(6N + 2a),

un entero par y de manera semejante

(1 + 2)2n (1 2)2n = 3a 2 + 2 2(2N) = (3a + 4N) 2,


estableciendo la veracidad de P(n).
46 Para n = 1, la aseveracin es cierta, ya que k2 1 = (k 1)(k + 1) es el producto de dos enteros pares consecutivos, y por lo tanto divisible
n
n
n+1
n+1
n
por 8. Hgase la hiptesis que 2n+2 |k2 1, y demostremos que 2n+3 |k2 1. Como k2 1 = (k2 1)(k2 + 1), vemos que 2n+2 divide
a(k2n 1), de donde el problema se reduce a demostrar que 2|(k2n + 1). Pero esto es obvio ya que el impar k2n torna a k2n + 1 par.
47 Este es el problema 47. Aqu se dar una solucin por induccin. Ntese que la solucin utilizando el principio de las pichoneras es ms
sucinta.
Supngase que de entre los 2n nmeros 1, 2, . . . , 2n, con n 2 , se han encontrado n + 1 nmeros tales que ninguno de ellos es divisible
por cualquier otro. Dentese este conjunto de n + 1 nmeros por Mn+1 . Se demostrar que si as occurriese sera posible seleccionar de entre
los 2n 2 nmeros 1, 2, . . . , 2n 2, un conjunto conteniendo n nmeros tales que ninguno de los n nmeros sea divisible por cualquier otro.
Se observan los siguientes casos:
1. Mn+1 no contiene al nmero 2n 1 ni al nmero 2n.
2. Mn+1 contiene a 2n 1 pero no a 2n.
3. Mn+1 contiene a 2n pero no a 2n 1.
4. Mn+1 contiene tanto a 2n 1 como a 2n.
Veamos
1. Qutese un nmero arbitrario del conjunto Mn+1 . Entonces quedan n nmeros ninguno de los cuales es mayor que 2n 2. Ninguno de
stos es divisible por cualquier otro.
2. Qutese el nmero 2n 1 del conjunto Mn+1 . Efectivamente, de nuevo, entre los n nmeros restantes, ninguno es mayor que 2n 2 y
ninguno de ellos es divisible por otro cualquiera.
3. Qutese el nmero 2n del conjunto Mn+1 ; el resultado es el mismo que en los casos 1 y 2.
4. Antes que todo, obsrvese que el nmero n no puede pertenecer al conjunto Mn+1 ; en caso contrario, el conjunto Mn+1 contendra a
dos los nmeros n y 2n; y 2n es divisible entre n. Qutese ahora los dos nmeros 2n 1 y 2n del conjunto Mn+1 . Dentese por Mn1
al conjunto de los n 1 nmeros que quedan. A continuacin agrguese el nmero n al conjunto Mn1 , obteniendo de este modo un
conjunto de n nmeros, ninguno de los cuales es mayor que 2n 2. Falta demostrar que de estos n nmeros, ninguno ser divisible
por cualquier otro. Como el conjunto Mn+1 no contuvo dos nmeros de los cuales uno fuera divisible por el otro, el conjunto Mn1
tampoco contendr tales nmeros. Por lo tanto, es slo necesario el demostrar que no existe dos nmeros tales, an cuando se agrega
el nmero n al conjunto Mn1 . Para hacerlo, basta demostrar (I) Que ningn nmero en Mn1 es divisible por n y (II) Que n no es
divisible por nmero alguno en Mn1 . La primera proposicin se deduce del hecho de que de los nmeros en Mn1 , ninguno es mayor
que 2n 2. La segunda se deduce del hecho de que 2n no es divisible por nmero alguno de en Mn1 . As se ha demostrado que si la
proposicin es falsa para los 2(n 1) nmeros 1, 2, . . . , 2n 2. De aqu que, si la proposicin es verdadera para los 2(n 1) nmeros
1, 2, . . . , 2n 2, tambin debe ser verdadera para los 2n nmeros 1, 2, . . . , 2n. La proposicin es verdadera para los dos nmeros 1 y 2;
de aqu que es verdadera para todos los conjuntos de 2n nmeros 1, 2, . . . , 2n, donde n es un nmero natural.

82

Anexo A

48 Para n = 1, tenemos
0 1 = f0 f1 = 12 (1)1 = f12 (1)1 ,

de donde la asercin es cierta para n = 1. Supongamos que n > 1, y que la asercin es cierta para n, esto es
fn1 fn+1 = fn2 + (1)n .
Usando fn+2 = fn+1 + fn , y por la hiptesis de induccin, fn2 = fn1 fn+1 (1)n . Esto significa que
fn fn+2

=
=
=
=
=

fn ( fn+1 + fn )
fn fn+1 + fn2
fn fn+1 + fn1 fn+1 (1)n
fn+1 ( fn + fn1 ) + (1)n+1
fn+1 fn+1 + (1)n+1 ,

de donde se colige el resultado.


49 Utilizarase induccin robusta. Obsrvese que 8 = 3 + 5, 9 = 3 + 3 + 3, 10 = 5 + 5, de donde se puede pagar 8, 9, o 10 pesos con las
susodichas monedas. Presmase que se puede pagar n 3, n 2, y n 1 pesos, esto es, que 3x + 5y = k tiene soluciones no negativas para
k = n 3, n 2 y n 1. Demostrarase que tambin se pueden obtener soluciones para 3x + 5y = k con k = n, n + 1 y n + 2. Ahora,
3x + 5y = n 3 = 3(x + 1) + 5y = n,
3x1 + 5y1 = n 2 = 3(x1 + 1) + 5y1 = n + 1,
3x2 + 5y2 = n 1 = 3(x2 + 1) + 5y2 = n + 2,
y as si las cantidades n3, n2, n1 se pueden pagar, tambin se puede pagar las cantidades n, n+1, n+2. La aseveracin queda demostrada
por induccin robusta.
1
1+2
=
. Presmase que para k > 1
4 2(1 + 1)




1
1
1
k+2
.
1
1
1
=
4
9
2(k + 1)
(k + 1)2

50 El resultado es inmediato para n = 1 ya que 1

Por la hiptesis de induccin,




1
1
1
1
1
1
4
9
(k + 2)2

=
=
=
=

k+2
1
1
2(k + 1)
(k + 2)2
k2 + 4k + 3
k+2
2(k + 1) (k + 2)2

(k + 1)(k + 3)
k+2
2(k + 1)
(k + 2)2
k+3
,
2(k + 2)

estableciendo el resultado para k + 1.


51 Para n = 0 esto es cierto, ya que 03 + 12 + 23 = 9. Presmase que k3 + (k + 1)3 + (k + 2)3 = 9N, en donde N es un entero. Se demostrar
que (k + 1)3 + (k + 2)3 + (k + 3)3 es tambin un mltiplo de 9. Pero
(k + 1)3 + (k + 2)3 + (k + 3)3 = 9N + (k + 3)3 k3 ,
en virtud de la hiptesis de induccin. Esto es
9N + (k + 3)3 k3 = 9N + (k3 + 9k2 + 27k + 27) k3 = 9N + 9k2 + 27k + 27 = 9(N + k2 + 3k + 3),
mltiplo de 9, como se quera demostrar.
52 El enunciado es obvio para n = 1. Si
1+

1
1
1
++ 2 < 2
n
22
n

para n > 1 entonces


1+

1
1
1
1
++
< 2 +
=.
n (n + 1)2
22
(n + 1)2

Indicaciones y respuestas

83

Se demostrar que para n > 1


2

1
1
1
< 2
+
.
2
n (n + 1)
n+1

Pero esto de inmediato resulta de la desigualdad


n(n + 1) < (n + 1)2 =



1
1
1 1
1
1
1

<

=
.
1

=
2
n
n(n
+
1)
n
n
n
+
1
n
+
1
(n + 1)

53 Primero demostramos que todos los elementos de C poseen la misma paridad. De cierto, sea x C y sean A (con suma de elementos a)
y B (con suma de elementos b) dos subconjuntos con n elementos cada uno verificando
C \ {x} = A B;

A B = ;

a = b.

De manera semejante, sea y C y sean A (con suma de elementos a ) y B (con suma de elementos b ) dos subconjuntos con n elementos
cada uno verificando
C \ {x} = A B ;
A B = ;
a = b .

Si c es la suma de todos los elementos en C entonces c = x + a + b = x + 2a y tambin c = y + a + b = y + 2a . As y x = 2(a a) y x, y


tienen la misma paridad.
Ahora demostraremos la igualdad de todos los elementos por induccin, donde n se mantendr fijo y se inducir en el mximo de los
elementos de
C = {c1 , c2 , . . . , c2n+1 }.
Si

max

1i2n+1

= 1 entonces c1 = c2 = . . . = c2n+1 = 1, por ser todos los elementos estrictamente positivos. Presumse pues que el enunciado

es cierto cuando

max

1i2n+1

ci = t > 1. Sea
F = { f1 , f2 , . . . , f2n+1 }

un conjunto de enteros positivos no nulos verificando la propiedad del enunciado con

max

1i2n+1

fi = t + 1. . Tenemos dos casos: o bien todos

los elementos de F son pares o bien todos nones.


Si todos los elementos de F son pares, aplica la hiptesis de induccin a { f1 /2, f2 /2, . . . , f2n+1 /2} porque
max

1i2n+1

fi /2 = (t + 1)/2 < t

y al ser todas las fi /2 idnticas tambin lo sern las fi .


Si todos los elementos de F son nones, aplica la hiptesis de induccin a {( f1 + 1)/2, ( f2 + 1)/2, . . . , ( f2n+1 + 1)/2} porque
max ( fi + 1)/2 = (t + 2)/2 < t

1i2n+1

y al ser todas las ( fi + 1)/2 idnticas tambin lo sern las fi .


54 Razonamos por induccin sobre m + n. Como
n a1 + a2 + + an < mn,

se sigue que m > 1. De igual manera se demuestra que n > 1. Para m + n = 4 se tiene m = n = 2, y las igualdades posibles son 1 + 1 = 1 + 1
y 1 + 2 = 2 + 1, de donde se colige el resultado. Supngase que el resultado es cierto para k = m + n 4 y considrese
a1 + a2 + + an = b1 + b2 + + bn < mn,
donde m + n = k + 1. Sin perdida de generalidad se puede presumir que a1 es la mayor de todas las ai y que b1 es la mayor de todas las bi . Si
a1 = b1 no cabr nada que demostrar, pues se podrn suprimir estos trminos y se lograr una suma restante idntica. Si a1 > b1 entonces
(a1 b1 ) + a2 + + an = b2 + + bn ,
y

mn
= n(m 1).
m
Como n + (m 1) = k, podemos aplicar la hptesis de induccin, obteniendo el resultado.
b2 + + bn < mn

84

Anexo A

55 Las figuras A.1, A.2, y A.3 proveen una descomposicin en 4, 6, y 10 subtringulos, respectivamente. As pues, dado n, se puede construir
o bien n + 3 tringulos o n + 5 tringulos. Obsrvese ahora que toda n 6 puede ser escrita de la manera 3x + 5y = n, lo que se puede
demostrar con otra induccin, a la manera del problema 49.

Figura A.1: 4 subtringulos equilteros.

Figura A.2: 6 subtringulos equilteros.

Figura A.3: 10 subtringulos equilteros.

56 Si s fuese una potencia de 2 entonces no hay nada que demostrar. Si s yace estrictamente entre dos potencias de 2, digamos 2r1 < s < 2r ,
entonces s < 2r < 2s < 2r+1 , por lo que el intervalo [s; 2s] contiene a 2r , una potencia de 2.
61 Si la tuviere entonces
a2m+1 a2m = a2m a2m1

m = 2, 3, 4, . . . .

Como claramente se tiene am+1 + am > am + am1 se deber tener


am+1 am < am am1
para m 2. Esto se puede escribir como

a2 a1 > a3 a2 > a4 a3 > . . .

una sucesin infinita de enteros positivos estrictamente decreciente, lo que es imposible.


65 Sea O el centro del disco, y sean A1 , A2 , . . . A7 los siete puntos en cuestin. Si ninguno de ellos es el centro del disco, entonces el menor
entre los ngulos Ai OA j es estrictamente inferior a 60 . Sean A y B los puntos correspondientes a este ngulo. Por la ley de los cosenos de
Al-Kashi
AB2 = OA2 + OB2 2OA OB cos AOB = AB < 1,
lo que es una contradiccin.

81 Ponga x = 123456789. Entonces x2 (x + 2)(x 2) = 4.


82 52
84 Pista:
22225555 + 55552222 = (22225555 + 45555 ) + (55552222 42222 ) (45555 42222 ).
160 63
166 Pista: Demuestre primero que 320 1 mod 100.
179 2
180 $0,73
194 Los puntos 16, 17, . . . , 48 son 33 en total y estn del mismo lado del dimetro que une al punto 15 con el 49. Para cada uno de estos hay
un punto correspondiente y opuesto en la circunferencia. As pues hay un total de 2 33 + 2 = 68 puntos en total.
195 Los factores de 295 son 1, 2, 22 , . . . , 295 . Observemos que 210 = 1024 y por lo tanto 220 = 1048576. Luego 219 = 524288 < 1000000 <
1048576 = 220 . Por lo tanto, son los factores 220 , 221 , . . . 295 los mayores de 1000000. Estos constituyen un total de 95 20 + 1 = 76 factores.
196 En utilizando
91 = 9
90 2 = 180
900 3 = 2700

enteros de 1-dgito,
enteros de 2-dgitos,
enteros de 3-dgitos,

Indicaciones y respuestas

85

un total de 9 + 180 + 2700 = 2889 dgitos han sido utilizados, as pues el 3000-avo dgito debe de pertenecer a los enteros positivos de
4-dgitos. Quedan pues 3000 2889 = 111 dgitos para ser usados y como 111 = 4 27 + 3, el 3000-avo dgito es el tercer dgito del 28-avo
entero positivo de 4-dgitos, esto es, el tercer dgito de 4027, es decir, el 2.
197 Presumiremos conocido el que los enteros naturales se pueden factorizar en factores primos de una manera nica. Entonces pues, al
expandir el producto
(1 + 2 + 22 + + 28 )(1 + 3 + 32 + + 39 )(1 + 5 + 52 )

obtenemos todos los factores de 28 39 52 y slo factores de este nmero. As pues, hay tantos factores como trminos en el producto. Por lo
tanto, hay (1 + 8)(1 + 9)(1 + 3) = 320 factores.
La suma de los divisores la obtenemos sumando las tres series geomtricas anteriores:
29 1 310 1 53 1

= 467689684.
21 31 51

En general, si n = pa11 pa22 pas s , donde las ps son primos distintos y si d(n), (n) denotan, el respectivamente, el nmero de divisores
positivos de n y la suma de los divisores positivos de n, el razonamiento anterior nos dice que
d(n) = (a1 + 1)(a2 + 1) (as + 1)
y

(n) =

p1a1 +1 1 p2a2 +1 1
pas +1 1

s
.
p1 1
p2 1
ps 1

198 Para escribir las primeras nueve pginas, se utilizaron nueve dgitos. Para escribir las 99 10 + 1 = 90 pginas entre la 10 y 99 inclusas,
se utilizaron 2 90 = 180 dgitos. Hasta ahora hemos utilizado 189 dgitos. Si el libro llegase hasta la pgina 999, las 999 100 + 1 = 900
pginas de tres dgitos utilizaran 3 900 = 2700 dgitos, que es mucho ms que la cantidad de dgitos prescrita. As pues, el nmero de
pginas es un nmero de tres dgitos. Nos quedan 1890 189 = 1701 dgitos que usar, que nos dan para 1701/3 = 567 pginas ms. As pues,
contamos 567 pginas a partir de la 100. Esto quiere decir que el libro tiene 666 pginas.
200 800
269 3030
270 93
272

5973
1993

308 4
309 a = 998 = b
310 2400
311 9
312 1
314 384
315 580
317 Pista: x3 1 = (x 1)(x2 x + 1)
319 Pista: Demuestre primero que csc 2x = cot x cot 2x.
320 Pista: Observe que
1
1
y
=

.
1 y 1 y2
1 y2

86

Anexo A

324 Pista: De la identidad


tan x tan y =

tan x tan y
1 + tan x tan y

deduzca que
arctan a arctan b = arctan
325 Pista: De la identidad

deduzca

k+1 k =

ab
.
1 + ab

1
,
k+1+ k

1
2 k + 1 2 k < < 2 k 2 k 1.
k

Haciendo k = 2, 3, . . . , n, luego

n
X

1
< 2 n 2.
2 n+12 2 <
k
k=2

Como 2 2 < 3 y n + 1 > n se sigue que

X 1

< 2 n 1.
2 n2 <
k
n

k=1

Poniendo n = 1000000 se obtiene el resultado.


342 Pista: Poner un = cos vn .
345 Se tiene
( f (x))2 f
de donde

Substituya x por

1x
1+x


= 64x,

 
 2
1x
( f (x))4 f
) = 642 x2
1+x
1x
. Entonces
1+x


f

1x
1+x

2


f (x) = 64

Divida (I) por (II),


f (x)3 = 64x2

(I)


1x
.
1+x

(II)


1+x
,
1x

de donde se destila el resultado.


362 Pista: cos2 x = 1 sin2 x
370 Observe que

(x 1)x(x + 1)(x + 2) + 1 = (x2 + x)(x2 + x 2) + 1 = (x2 + x)2 2(x2 + x) + 1 = (x2 + x 1)2 = x2 + x 1.


As

30 31 32 33 + 1 = 312 + 31 1 = 991.

371 Se tiene
xx

x.

..

= 2 = x2 = 2 = x =

2,

ya que x es positivo.
372 Se tiene

q
x+

x+

x + = 2 = x + 2 = 2 = x + 2 = 4 = x = 2.

376 Pista: Ponga y = mx y divida las ecuaciones as obtenidas. Resuelva para m.


377 Pista: Escriba la ecuacin como
(x2 9x 1)10 10x9 (x2 9x 1) + 9x10 = 0.

Indicaciones y respuestas

87

383 Pista: Ponga u = x + 2, v = y + 3. Divida una ecuacin por la otra.


384 Pista: Ponga u = x + y, v = x y.
421 Pngase
KA = [BCO]

KB = [CAO]

KC = [ABO].

Entonces

[ABO]
[CAO]
[ABO] + [CAO]
KB + KC
AO
=
=
=
=
.
OA
[BOA ] [OAC] [BOA ] + [OAC]
KA
De manera semejante se puede demostrar que
BO
K + KC
= A
OB
KB

CO
K + KB
= A
.
OC
KC

Se deduce que
92 =

KB + KC KA + KC KA + KB
+
+
= KA2 KB + KA2 KC + KB2 KA + KB2 KC + KC2 KA + KC2 KB = 92KA KB KC .
KA
KB
KC

As
KA2 KB + KA2 KC + KB2 KA + KB2 KC + KC2 KA + KC2 KB + 2KA KB KC
AO BO CO
KB + KC KA + KC KA + KB

=
= 94.
OA OB OC
KA
KB
KC
KA KB KC
422 Gracias a la frmula de Hern, [ABC] = 84. ABC A BC y por lo tanto [A BC ] = 84k2 donde k es la constante de semejanza. El
rea del A BC tambin puede obtenerse al substraer l rea de los tres trapezoides de altura 2 formados al unir los vrtices correspondientes
de los tringulos. As pues
1
84k2 = 84 41(k + 1) = k = ,
2
84
= 21.
al descartarse una raz extraa. El rea buscada es entonces
4
437 Sea M el punto medio de BD. Del ejemplo 436, MP y MS son iguales y perpendiculares, y tambin MQ y MR son iguales y perpendiculares. Un giro de 90 toma al PMR al QMS. Por lo tanto, PR y QS son iguales y perpendiculares.

Вам также может понравиться